◆ わからない問題はここに書いてね 22 ◆

このエントリーをはてなブックマークに追加
1132人目のともよちゃん
   / ̄   ̄ ヽ
  / ,,w━━━.、)   / ̄ ̄ ̄ ̄ ̄ ̄ ̄ ̄ ̄ ̄ ̄ ̄ ̄ ̄ ̄ ̄ ̄
  ! .fw/f_」」_|_|_i_)  | ここは分からない問題について質問するさくらちゃんスレですわ
  ヽ|:::(6||f;j' ,fj'||)  | スレッドや業務連絡,記号の書き方例は >>2-13 の中に。
 ∠|::i:!::|:|、_ワノ:i、 <  『質問です』って名前で質問して頂けるとみつけやすいですわ
  .|::|< |::|ヽーノ`l:i;ヽ \_________________
  .ノ:ノ' i:::l `只´|:|i)::)
 (::(:i  |:::|ノ ) j:j|:(

    (⌒, -- 、⌒)     / ̄ ̄ ̄ ̄ ̄ ̄ ̄ ̄ ̄ ̄ ̄ ̄ ̄ ̄ ̄ ̄
  _  Y      Y  _ < 知ってるか?『132人目の素数さん』ってのはなぁ。
 ミ \| ・  . ・| / 彡 | 132個目の素数が743(ななしさん)だからなんやで
    @ゝ.  ^  ノ@    | どや?また一つ利口になったやろー
                \________________

【前のスレッド】
◆ わからない問題はここに書いてね 21 ◆
http://cheese.2ch.net/test/read.cgi/math/1011689052/
2132人目のともよちゃん:02/02/01 12:59
3132人目のともよちゃん:02/02/01 13:00
【関連スレッド】
雑談はここに書け!【2】
http://cheese.2ch.net/test/read.cgi/math/989344065/l50
くだらねぇ問題はここへ書け ver.3.1415926
http://cheese.2ch.net/test/read.cgi/math/1004559257/l50
厨房問題必ず答えます!3(お化けスレ)
http://cheese.2ch.net/test/read.cgi/math/1009032097/l50
4132人目のともよちゃん:02/02/01 13:01
【掲示板での数学記号の書き方例】
■数の表記
 ●スカラー:a,b,c,...,z, A,B,C,...,Z, α,β,γ,...,ω, Α,Β,Γ,...,Ω, ... (← ギリシャ文字はその読み方で変換可.)
 ●ベクトル:V=[V[1],V[2],...], |V>, V↑, vector(V) (← 混同しない場合はスカラーと同じ記号でいい.通常は縦ベクトルとして扱う.)
 ●テンソル(上下付き1成分表示):T^[i,j,k...]_[p,q,r,...], T[i,j,k,...;p,q,r,...]
 ●行列(1成分表示):M[i,j], I[i,j]=δ_[i,j]
 ●行列(全成分表示):M=[[M[1,1],M[2,1],...],[M[1,2],M[2,2],...],...], I=[[1,0,0,...],[0,1,0,...],...] (← 列(または行ごと)に表示する.)

■演算・符号の表記
 ●足し算:a+b
 ●引き算:a-b
 ●掛け算:a*b, ab (← 通常は"*"を使い,"x"は使わない.)
 ●割り算・分数:a/b, a/(b+c), a/(bc) (← 通常は"/"を使い,"÷"は使わない.)
 ●複号:a±b=a士b, a干b (← "±"は「きごう」で変換可.他に漢字の"士""干"なども利用できる.)
 ●内積・外積・3重積:a・b=(a,b), axb=a∧b=[a,b], a・(bxc)=(axb)・c=det([a,b,c]), ax(bxc)

■関数・数列の表記
 ●関数:f(x), f[x]
 ●数列:a(n), a[n], a_n
 ●平方根:√(a+b)=(a+b)^(1/2) (← "√"は「るーと」で変換可.)
 ●指数・指数関数:a^b, x^(n+1), exp(x+y)=e^(x+y) (← "^"を使う."exp"はeの指数.)
 ●対数・対数関数:log_{a}(b), log(x/2)=log_{10}(x/2), ln(x/2)=log_{e}(x/2) (← 底を省略する場合,"log"は常用対数,"ln"は自然対数.)
 ●三角比・三角関数:sin(a), cos(x+y), tan(x/2)
 ●行列式・トレース:|A|=det(A), tr(A)
 ●絶対値:|x|
 ●ガウス記号:[x] (← 関数の変数表示などと混同しないように注意.)
 ●共役複素数:z~
 ●転置行列・随伴行列:M† (← "†"は「きごう」で変換可.)
 ●階乗:n!=n*(n-1)*(n-2)*...*2*1, n!!=n*(n-2)*(n-4)*...
 ●順列・組合せ:P[n,k]=nPk, C[n.k]=nCk, Π[n,k]=nΠk, H[n,k]=nHk (← "Π"は「ぱい」で変換可.)

■微積分・極限の表記
 ●微分・偏微分:dy/dx=y', ∂y/∂x=y,x, D^(n)f(x) (← "∂"は「きごう」で変換可.)
 ●ベクトル微分:∇f=grad(f), ∇・A=div(A),∇xA=rot(A), (∇^2)f=Δf (← "∇"は「きごう」,"Δ"は「でるた」で変換可.)
 ●積分:∫[0,1]f(x)dx=F(x)|_[x=0,1], ∫[y=0,x]f(x,y)dy, ∬_[D]f(x,y)dxdy, 点[C]f(r)dl (← "∫"は「いんてぐらる」,"∬"は「きごう」で変換可.)
 ●数列和・数列積:Σ_[k=1,n]a(k), Π_[k=1,n]a(k) (← "Σ"は「しぐま」,"Π"は「ぱい」で変換可.)
 ●極限:lim_[x→∞]f(x) (← "∞"は「むげんだい」で変換可.)

■その他
 ●図形:"△"は「さんかく」,"∠"は「かく」,"⊥"は「すいちょく」,"≡"は「ごうどう」,"∽"は「きごう」で変換可.
 ●論理・集合:"⇔⇒∀∃∧∨¬∈∋⊆⊇⊂⊃∪∩"は「きごう」で変換可.
 ●等号・不等号:"≠≒≦≧≪≫"は「きごう」で変換可.

※ ここで挙げた表記法は1例であり,標準的な表記法からそうでないものまで含まれているので,後者の場合使う時にあらかじめことわっておいたほうがいい.
※ 関数等の変数表示や式の括弧は,括弧()だけでなく[]{}を適当に組み合わせると見やすい場合がある.
※ 上記のほとんどの数学記号や上記以外の数学記号は大体「きごう」で順次変換できる.
5132人目のともよちゃん:02/02/01 13:01
【業務連絡】
■900を超えたら新スレに移行準備.
■旧スレ側 → 終了宣言,新スレへの誘導.
■新スレ側 → 開始宣言と目次,旧スレのリンク,掲示板での数学記号の書き方例,
  業務連絡・その他,旧スレ側の残り問題の移動.
■数学板の要望スレで数学板の注意書き(リンク先)の変更依頼.
■単独の質問スレは,このスレか「くだらんスレ」に誘導して下さい.
■誤って過去スレに新たに書き込まれた質問は,最新スレに誘導して下さい.
【数学板削除依頼スレ】
http://teri.2ch.net/test/read.cgi/saku/986384122/ (レス削除)
http://teri.2ch.net/test/read.cgi/saku/987829968/ (スレッド削除)
【ローカルルール等リンク先更新総合スレッド】
http://teri.2ch.net/test/read.cgi/accuse/992178408/
★__________________________.
|              │
│ はにゃ〜ん     |
| γ∞γ~  \    |
│人w/ 从从) )   │
│ ヽ | |┬ イ |〃  │
│ `wハ~ . ノ)    │
│  / \`「 .     │
| 数学板さくらスレ  |
|_________________________│


〃二二ヽ
| |77777〉
| | ゚д゚ノ|  サクラチャンノハタケイヨウデスワ
|⊂   つ

6132人目のともよちゃん:02/02/01 13:02
━━━━━━━━━━━━━━━━━━━━━━━━━━━━━━━━━━━━━

                   移転完了しましたわ (o^-')b
              ◆ わからない問題はここに書いてね 22 ◆
         いよいよ始まりますわ♪ それではみなさま心置きなくどうぞ

━━━━━━━━━━━━━━━━━━━━━━━━━━━━━━━━━━━━━
7132人目の素数さん:02/02/01 13:11
教えてください!
15進数で
11cの1.5乗は何ですか?
解き方も教えてください。
8ネット屋 ◆.t4dJfuU :02/02/01 13:36
>>7
11c[15]=1*15*15+1*15+12=252 =7*6^2
(7*6^2)^(3/2)=7*6^3 Sqrt(7)=1512 Sqrt(7)
1512 Sqrt(7) = 6ac Sqrt(7) [15]
9132人目の素数さん:02/02/01 13:50
15進数で1.5というのは、1+5/15=4/3 になるんじゃないのか?
10ネット屋 ◆.t4dJfuU :02/02/01 13:52
>>9 うっ、鋭いかも。1.5![15]=3/2[15]か!
んじゃ、答えは12ba.6[15]かな?
11ネット屋 ◆.t4dJfuU :02/02/01 14:01
>>10 うっ、うそだ。うつだ(w
12ネット屋 ◆.t4dJfuU :02/02/01 14:46
難しいねー。11c x 6^(3/7) x 7^(3/14) [15]
もっとキレイになるのだろうか?
13132人目の素数さん:02/02/01 16:21
おながいします


ラプラス変換で部分分数展開をして解け

       3
F(S)= ―――――――
     S(S^2+a^2)
>>7
win付属の関数電卓によれば
252^(4/3) [10]
= 711.ac8c878e77b…[15]
桁落ちの可能性もあるので下位は保証できません。

>>12
3/14はどこから出てきた?
15940:02/02/01 20:00
前スレ
>>948-949
解けました。どうもありがとうございます。
あと何故e^(-x)を両辺にかけるという発想がでるのか
教えてもらえればうれしいです。
16132人目の素数さん:02/02/01 20:39
1,11,111,1111,11111,111111,1111111,・・・・・・・

上の行列が全て素数である事を証明したいのですが。
17111=3*37:02/02/01 20:41
1816:02/02/01 20:43
あそっか。どうも
19132人目の素数さん:02/02/01 20:47
放物線y=x^2-x+3に点(1,-1)から引いた二つの接線と
放物線で囲まれた面積を求めよ。

曲線y=x^3-xと、その上の点(2,6)とで囲まれた部分の面積を求めよ。

面積苦手です。お願いします。
20お願いします。:02/02/01 21:02
「赤球5個、青球4個、白球3個が袋の中に入っている。この袋の中から同時に4個取り出す時、
取り出した球がどの色も含む確立を求めよ。」という問題で次のように考えましたが、上手くいきませんでした。御指摘願います。

 取り出し方は12C4=495通りで、起こり方は同様に確からしい。

 A:取り出す球がどの色も含む

 と、すると、n(A)=5C1+4C1+3C1+9C1=540>分母(=495)?
>>16
むしろ
1111111111111111111

11111111111111111111111
が素数だというほうが、不思議な気がする。
22aa:02/02/01 21:18
>>20
赤2、青1、白1

赤1、青2、白1

赤1、青1、白2
のときでしょ。
あとは考えて。
23 ◆FHB7Ku.g :02/02/01 21:21
>>19
y=x^2-x+3
y'=2x-1
接線をy=(2t-1)(x-t)+t^2-t+3とおく。
x=1,y=-1を代入してt=-1,3
よってS=∫[-1,1](x+1)^2dx+∫[1,3](x-3)^2dx=16/3・・・答

2個目の問題は意味が・・・
2420:02/02/01 21:21
>>22
 何故、n(A)=5C1+4C1+3C1+9C1ではダメなんでしょうか?
>>24
>n(A)=5C1+4C1+3C1+9C1ではダメなんでしょうか?
じゃあ、なんでこうなると思うのか教えて。
(ところで、「+」は「×」のまちがいだろうな?)
>>20
赤が2個:5C2×4C1×3C1=120
青が2個:5C1×4C2×3C1=90
白が2個:5C1×4C1×3C2=60
合計:120+90+60=270通り

確率は270/495=6/11

が正解。
1色1個ずつ選んでから残りを1つ選ぶと、重複があるでしょ。
27 ◆FHB7Ku.g :02/02/01 21:31
>>24
赤赤青白のとき、5C2*4C1*3C1

赤青青白のとき、5C1*4C2*3C1

赤青白白のとき、5C1*4C1*3C2

よって(5C2*4C1*3C1+5C1*4C2*3C1+5C1*4C1*3C2)/12C4通りかな?


28aa:02/02/01 21:33
>>20くんは
赤A,青A、白Aを取った後に赤Bをとる。
みたいに考えてるのかもしんないが、
これでいくと
赤B、青A,白Aを取った後に赤Aをとる。
っていうのも(重複されて)計算しちゃうでしょ、ということを
>>26さんはいいたいのです。
2920:02/02/01 21:34
>>25
 分かった!例えば、
 
 (赤1、青1、白1、赤2)と(赤2、青1、白1、赤1)とがダブってるって事ですよね?(右の数字はそれぞれの区別)
30質問です:02/02/01 21:42
R^3の部分空間(1)〜(3)の基底を求めよ。
(1)平面 3x-2y+5z=0
(2)直線 x=2t,y=-t,z=4t (−∞<t<∞)
(3)(a,b,c)の形のベクトル全体。ただしb=a+c
3120:02/02/01 21:44
>>26-28
 そういう事に、引っ掛からなくするためには、どう勉強してけばいいですか?
32 ◆FHB7Ku.g :02/02/01 21:52
>>20
各色から1個ずつ取ったあとで、最後の色を選ぶ、という考えなら
重複を考えて

5C1*4C1*3C1*(4C1/2)+5C1*4C1*3C1*(3C1/2)+5C1*4C1*3C1*(2C1/2)
と計算すればよいと思います。
33質問です:02/02/01 21:55
質量mの雨の粒子が落ち始めてからt病後の速度をvとすると
m(dv/dt)=mg-cv (m,g,cは正の定数)
が成り立つ。この時次の問いに答えよ。

1)この微分方程式を1階線形とみて、初期条件「t=0のときv=0」
 を満たす解を求めよ。
2)この微分方程式を変数分離形とみて、(1)と同じ初期条件を満たす解
 を求めよ。

どんな斉次微分方程式?にすれば良いかもわからないです。
お願いします。
3426:02/02/01 21:57
>>31
場合の数を数えるような問題では、
とにかく勝手な思い込みをやめること。
もっと言うと、「簡単な式で書けるにちがいない」という思い込みをやめる。
むしろ、「なるべく細かく場合分けして考えよう」という方向が結果近道。
簡単なそれっぽい式を探そうとするのは、結局自分で自分をだますことに
しかならないと思う。
35 ◆FHB7Ku.g :02/02/01 22:04
>>33
リア厨なので意味わかんないですが....まず両辺をmで割って
v'+(c/m)v=g・・・ア
[v*{e^(c/m)v}]'=[e^{(c/m)v}]*{v'+(c/m)v}だから

アの左辺=[v*{e^(c/m)v}]'/[e^{(c/m)v}]

よって[v*{e^(c/m)v}]'=g*e^{(c/m)v}
あとはこれを両辺で積分すれば答でると思います。
三国無双?
37 ◆FHB7Ku.g :02/02/01 22:11
y'+ky=f(x)なら{y*e^(kx)}'を、
y'-ky=f(x)なら{y*e^(-kx)}'を、
y''+my=f(x)なら(y*sinAx)''を、
y''-my=f(x)なら(y*cosBx)''を、
計算してみてください。きっと左辺と似た形状でることと思います・・
(ただしm>0,k>0)
38 ◆FHB7Ku.g :02/02/01 22:11
>36
ばれました。はい。
39:02/02/01 22:17
>>35
おまえよく大学の問題手が出せるなー関心
>>38
がむばれ
41132人目の素数さん:02/02/01 22:23
>>39
◆FHB7Ku.g=三国無双
ってこと?有名なの?解説希望。
優秀なリア厨age
>>30
1) (1,0,-3/5),(0,1,2/5)
2) (2,-1,4)
3) (1,1,0),(0,1,1)
43:02/02/01 22:32
>>41
いや、三国無双はなんのこといってるが知らんが
>>33はどっからみても大学の問題だろ
それをリア厨で解くなんてすごいと感心しただけさ
4430:02/02/01 22:40
>>42
レスありがとうございます。
解き方が解らないので途中の式も教えて下さい。
4541:02/02/01 22:40
>>43
どうも。
まあ、>>33は高校でもやるところもあるけど、凄い事には変わりないね。
大抵、背伸びしてやってみたが、実際には問題は解けないしったか君になる
場合が多かったりする中、こういうリア厨がいるとホッとする。
46132人目の素数さん:02/02/01 22:44
>>33

v'= -(c/m)(v-mg/c) …(1)

u = v-mg/c …(2) とおけば

u' = v' より (1) は

u' = -(c/m)u …(3)

∴u(t) = K1・exp((-c/m)t) …(4)

v(0) = 0 より(2)から

u(0) = -mg/c …(5)

(4),(5)より K1 = -(mg/c) 

よって(4)より

∴ u(t) = -(mg/c)・exp((-c/m)t)

(2)より

∴ v(t) = -(mg/c)・(exp((-c/m)t)-1) … [答]

********************************

(3)より

du/dt = -(c/m)u …(6)

変形して

du/u = -(c/m)dt …(7)

両辺積分して
log|u| = -(c/m)t + K2

∴ u = exp(-(c/m)t + K2
= K3・exp(-(c/m)t …(8)

v(0) = 0 より(2)から

u(0) = -mg/c …(9)

(8),(9)より

 K3 = -(mg/c) …(10)
 
よって(8)より

∴ u(t) = -(mg/c)・exp((-c/m)t)

(2)より

∴ v(t) = -(mg/c)・(exp((-c/m)t)-1) … [答]


47 ◆FHB7Ku.g :02/02/01 22:55
>>聞き流してください・・。
くだらないことだけど、僕は数学の才能はないことだけは分かった。
その証拠にパターン類題からずれると解けないし…。
「ひらめかない」暗記系の脳だと思う。数学に関しては数3までで頭パンパン。
僕の場合,解き方まで覚える系の勉強方法なので,「情報処理」みたいな感じ。
大学に受かったら数学はやめると思う・・ついて行けなく落第すると思う。
4833:02/02/01 22:57
>>35>>46
どうもありがとうございます!
リア厨って…。
弟もリア厨だけど爪の垢のませてやりたいよ。。。
高校飛び級しちゃって下さい。
>>46
ご丁寧にどうもありがとうございますです。

いまから読解モードに入ります.
49:02/02/01 23:02
>>47
待てお前リア厨なのになぜ数3やってる?
趣味?
50 ◆FHB7Ku.g :02/02/01 23:07
>>49兄のススメ・・。趣味っていうと,そうかもしれないけど,根暗な子の印象が強く打ち出されて
しまい,イジメられそうだから黙ってようと思います..。英語、社会、体育、柔道が得意だと人気アップ
確実だけど,数学が趣味ですなんてばれたらホントに何されるか・・。
5142:02/02/01 23:09
1)
z=-(3/5)x+(2/5)yより
x=a,y=bとすると
(x,y,z)
=(a,b,-(3/5)a+(2/5)b)
=(a,0,-(3/5)a)+(0,b,(2/5)b)
=a(1,0,-3/5)+b(0,1,2/5)
2)
(x,y,z)
=(2t,-t,4t)
=t(2,-1,4)
(3)(a,b,c)の形のベクトル全体。ただしb=a+c
(a,b,c)
=(a,a+c,c)
=(a,a,0)+(0,c,c)
=a(1,1,0)+c(0,1,1)
5241:02/02/01 23:13
>>47
んなこたーない。
パターンから外れた特殊な問題を特殊な方法で解くのは確かに見ていて鮮やかだけど
問題の背景にある多いな体系や構造を見抜く(そして既存のものに求めるもの
がなければ自分でつくる)力の方が重要だと思う。
というか、人材が他の分野に流れるのが寂しいので、数学続けて欲しい。
いや、ほんとお前みたいな天才未満みたいな感じなのが意外に伸びたりするんだって!
5341:02/02/01 23:17
>>50
いや、そういうDQNは放っておけ。
そういう奴らに凄いと思われるよりも、本当に凄い奴らに認められる方が
何倍もいいから。
スマソ、ちょっと熱くなってしまった。
54:02/02/01 23:22
>>50
いい趣味だ、がんばれや
俺もパターンにはまった問題しか解けん
がお前にはまだ時間がある
将来が楽しみだ。俺もなんかやる気が出てきたわ
まけずとがんばるべ
5530:02/02/01 23:28
>>51さん、ありがとうございました!
56 ◆FHB7Ku.g :02/02/01 23:37
なんとかがんばります。有難うございます。
57 ◆FHB7Ku.g :02/02/02 00:08
>>19問題を下のように書き換えてみると、、

「曲線y=x^3-xと、曲線上の点(2,6)における接線とで囲まれた部分の面積を求めよ。」
y=x^3-x
y'=3x^2-1
接線をy=(3t^2-1)(x-t)+t^3-tとおく。
x=2,y=6を代入し,t=2を解に持つことを考慮して,
(t-2)^2*(t+1)=0
∴t=-1,2
よって,求める面積は
∫[-1,2]-(x+1)^2*(x+2)dx=(1/12){2-(-1)}^4=27/4・・・答
58132人目の素数さん:02/02/02 00:22
位相空間の位相同型写像の問題なんですが、

S^n-{p0}同相E^n同相U^n (n=2)
(同相の記号が出なかったので、漢字で書いておきました)

の証明の仕方を教えてください。よろしくお願いします。
59:02/02/02 00:49
はさみうちの原理がよくわかんないので詳しく教えてください。
宜しくお願いします。
6033:02/02/02 00:58
>>46
1階線形とみて解いた場合と
変数分離形とみて解いた場合の違いがよくわからないんですが
どういう違いなんでしょう?
61ないと:02/02/02 01:50
二進数でみたとき、3の倍数になる値の正則表現てわかります?
11 = 3
110 = 6
1001 = 9
1100 = 12
>>61
しっかり考えたわけではないが・・・
偶数桁にある1の数と奇数桁にある1の数の差が3の倍数なら3の倍数?
63 ◆GaussrLU :02/02/02 02:22
>>61-62
それが正解と思います.
3 で割った余りの世界で考えれば,
2^n は, n が奇数のとき, 1 で, n が偶数のとき 2 (=-1) となります.
従って, k を
k = Σa_n 2^n
と2進表記するとき,
nを奇数, mを偶数とすれば,
k ≡ Σa_n - Σa_m (mod 3).
となりますので, 偶数桁にある数の和と, 奇数桁にある数の和の差が
3 で割れるかどうかで判定できます.
6462:02/02/02 02:25
>>63
トリップカコイイ!
65 :02/02/02 02:29
>>61
正則というのがよくわからないが、もし62が言ってるような内容でよければ。
各位の数を2けたごとにわけていき、その和が11(2)の倍数なら3(10)の倍数。
具体的にいうと、
10001101ならば、
10+00+11+01(2)=110となり3(10)の倍数となる。
もし7の倍数ならば3けたごとにわけて足す。
つかこんなの簡単に証明できるか。
66 ◆pvySbQO2 :02/02/02 02:40
>>61
正則表現をどう表したいのか分からないけど
3の倍数は
0か
1の後に「0の後に1を0個以上の後に0」を0個以上の後に1
を0個以上です。
>>61
{A,B,C}の3状態オートマトン:
A,B,C はそれぞれ mod3 で0,1,2 に対応する。
状態遷移は
  0 1
A  A B
B  C A 
C  B 1

これから連立方程式
A=0+B1 (1)
B=1+C1 (2)
C=C1+B0 (3)
を得る。(2)を(3)に代入して
C=C1+10+C10
これを解いて
C=(10)(1+10)^*
よって
A=0+(1+C1)1=0+((10)(1+10)^*)1


6867訂正:02/02/02 03:06
状態遷移は
   0 1
A  A B
B  C A 
C  B C
6967まちがってた:02/02/02 03:17
これから連立方程式
A=0+A0+B1 (1)
B=1+A1+C0 (2)
C=B0+C1 (3)
を得る。
7069続き:02/02/02 03:48
(3)より
C=B01*
(2)に代入して
B=1+A1+B01*0
これを解いて
B=(1+A1)(01*0)*
(1)に代入して
A=0+A0+(1+A1)(01*0)*1=0+1(01*0)*1+A(0+1(01*0)*1)
これを解いて
A=(0+1(01*0)*1)(0+1(01*0)*1)*
71質問です。:02/02/02 04:18
次の広義積分を求めよ

∫[0,∞]{1/√(1+2x^2)−1/(√2)(1+x)}dx

教えてください。おねがいします。
72132人目の素数さん:02/02/02 04:21
di/dt + αi = Asinωt (α,A,ωは正の定数)

この時iをtの式で表せ。
ただし、t=0のときi=0とする。

積分永久ループ状態に陥って解けないです。
こういうのはどうすれば良いのでしょうか?
73>72:02/02/02 04:29
d/dt(i exp(αt))=(di/dt + αi)exp(αt)= Asinωt exp(αt)
両辺をtで積分してexp(αt)で割るとiが求まる。
74132人目の素数さん:02/02/02 04:34
>>71
ふつうに0からAまでの積分もとめて
A→∞ でいいと思うが?
75132人目の素数さん:02/02/02 04:47
>>73
Asinωt*exp(αt)
の積分ができないのです;;

どうすればできるんでしょう(T-T)
>75
I=∫sinωt*exp(αt) dt
を2回部分積分すると

I=〜I+〜
という形の式になるのでこれをIについて解く
77質問です。:02/02/02 04:52
>>74
・・・その積分ができないんです。
まず√2x=tanθって置換しますよね?
7875:02/02/02 05:04
>>76
うまい事解けそうです!
どうもありがとうございます!!!
>71
最後の(1+x)はどっちにかかってんだろ?
80質問です。:02/02/02 05:17
>>79
分母です。
すいません、わかりにくくて・・・
81132人目の素数さん:02/02/02 05:24
整数論で出てくる類数って何のことか教えてください。
>>80
∫{1/(√((x^2)+A))}dx = log|x+√(x^2+A)|

これ使えば解けると思うけど?
83132人目の素数さん:02/02/02 06:52
>>58
E^n, U^nって何?

>>81
Qの有限次拡大をK, K内でのZの整閉包をO, Kの乗法群K^* = K-{0},
Oの素イデアルで生成される自由アーベル群をDiv(O)とする。
f∈K^*にfO∈Div(O)を対応させることにより
準同型φ:K^*→Div(O)が得られる。
φのcokernel coker(φ)は有限群で、この位数をKの類数という。
8483:02/02/02 06:55
>Oの素イデアルで生成される自由アーベル群をDiv(O)とする。

×素イデアル → ○ 0でない素イデアル
85質問です:02/02/02 07:47
1  ∫(1/1-x^4)dx
2  ↑のxを3乗にしたもの
3  ∫[0,1](x/(√2+x))dx

の3つが解けません・・・。どなたか回答をお願いいたします。
あとこちらで幾何学の質問をしてもよろしいのでしょうか?
>>85
カッコの付け方が無意味っつーか変

1∫dx/(1-x^4)  部分分数にわける
2  同上
3∫[0,1](x/√(2+x))dx  √(2+x)=tと置換
87132人目の素数さん:02/02/02 09:05
>>85
1 ∫(1/1-x^4)dx=-1/2∫1/(x^2+1)dx -1/4∫1/(x+1)dx +1/4∫1/(x-1)dx
=-1/2tan^-1(x) -1/4log|x+1| +1/4log|x-1|
=-1/2tan^-1(x) +1/4log|(x-1)/(x+1)|

2 1/(1-x^3)=1/3(x-1) -1/3*(x+2)/(x^2+x+1)
=1/3(x-1) -1/3*(x+2)/{(x+1/2)^2+3/4}
=1/3(x-1) -1/6*2(x+1/2)/{(x+1/2)^2+3/4}-1/2{(x+1/2)^2+3/4}より,

 ∫(1/1-x^3)dx=1/3log|x-1| -1/6log|(x+1/2)^2+3/4| -1/√3tan^1{2/√3(x+1/2)}
=1/6log|(x-1)^2/(x^2+x+1)| -1/√3tan^1{4/√3(2x+1)}

3 √(x+2)=uとおくと、
∫[0,1](x/(√2+x))dx=∫[√2,√3](u^2-2)/u*(2udu)
=∫[√2,√3](2u^2-4)/u du
=〔2/3u^3-4u〕[√2,√3]
=2√3 -8/3√2
解法は正しいと思いますがミスがあるかもしれません。
8885:02/02/02 09:47
>>86 >>87さんご丁寧に有難うございました。
どうも表記がしにくく、見にくい数式になってしまい申し訳ありません。
89質問です。80:02/02/02 11:09
>>82
それは公式的なものなのでしょうか?
90ネット屋 ◆.t4dJfuU :02/02/02 11:24
>>14 遅くてすまんす。
252^(4/3)=252 6^(2/3) 7^(1/3)
これを単に15進数にしただけです。
91質問です。:02/02/02 11:31
僕にとっては凄く難しい問題なので解答をお願いします。

1 ある歪んだコインを1000回投げたところ、表が600回出た。

 (1) このデータを用いて表の出る確率が0.5である、という仮説を有意水準5%で
    カイ2乗検定しなさい。なお、帰無仮説と対立仮説を明示すること。
    またP(χ^2≧4)=0.05としなさい。
 (2) 表の出る(真の)確率をPoとすると、n回中X回表が出る確率(確率関数)はどのように
    表されるか?また、Poの最尤推定値およびそれに対応する最大対数尤度はn、Xを用いて
    どのように表されるか?(答えのみでなくその導出過程も明記すること)
 (3) AICを用いると、以下の4つの仮説ではどれが一番適切と考えられるか?
     仮説1:Po=0.5
     仮説2:Po=0.57
     仮説3:Po=0.62
     仮説4:Po=(2)を用いた最尤推定値

2 次の二つの分布A,Bのどちらが分布Cに近いか、カルバック・ライブラーの情報量を
  用いて比較せよ。
    @分布A:N(1,3)、分布B:N(2,1)、分布C:N(0,1)
    A分布A:λ=0.5の指数分布、分布B:λ=1.5の指数分布、分布C:λ=1の指数分布
    ただし、パラメーターλの指数分布の密度関数は
           λe^-λχ  χ≧0  
    f(χ)={   0      χ<0
    である。
    
3 カルバック=ライブラーの情報量とAICについて、それぞれの意味及び二つの関係を述べよ。
  その他、それらに関して理解したこと書きなさい。
92132人目の素数さん:02/02/02 11:48
>>91
3くらい自分でやれや。何でも聞くな。
93132人目の素数さん:02/02/02 12:33
>>83
E^nの図形
U^n={(x1,x2,...,xn)∈E^n:x1^2+x2^2+...+xn^2<1}
をn次元開球体といいます。
特にn=1のときU^1=(-1,1)です。
一般に任意の次元nと点p0∈S^nに対して<<58の式が成り立つそうです。
分かりにくい説明で本当にすいません。
94ぴーこ:02/02/02 15:20
誰か牽引曲線の弧長によるパラメーター表示の出し方を教えて下さい
95質問です:02/02/02 16:16
1次元球面S1から1点p0∈S1を取り除いた図形S1−{p0}はE(1)と位相同型
である。中心が(0,1)で半径が1のS1を考えp0=(0,2)とする。このときE(1)
をx軸と考えると、

 f:S1−{p0} → E(1);(x,y) → 2x/2-y

は位相同型である。実際fは全単射で、fとその逆写像

 f(-1):E(1) → S1−{p0};x → (4x/x^2+4 , 2x^2/x^2+4)

とは連続である。

という問題で2次元球面の場合の時でもfが全単射であり、f(-1)がfの逆
写像であることを証明せよ。分かる方いましたら証明をお願いしします。

なおf(-1)はfのインバース、E(n)はn次元ユークリッド空間のことです。
大変分かりにくい文になってしまい申し訳ありませんが宜しくお願いい
たします。
>>90
n進法を勘違いしてない?
2/3、1/3は15進法でも2/3、1/3でっせ。
おもろそうな奴がいる。
>95
2次元球面のときのfを構成すれば一目瞭然
あと、分数は分母分子がわかるように括弧でくくってくれ‥
99質問した95:02/02/02 17:41
うっ・・ 大変申し訳ありませんが詳しく書いていただけないでしょうか?
本当にわけがわからないのです・・・。
100132人目の素数さん:02/02/02 20:25
質問です。
球状の惑星の海全部の水の体積を求めたい。
データとしては海抜0を表す半径=r、
経度=φ緯度=θでの深さd(φ,θ)が2変数の関数として
記録されている。
2重か3重積分になるだろうけど、微分要素と式が分からないんだ。
101132人目の素数さん:02/02/02 21:10
>95
まず、問題の意味はわかってる? 図に描いて考えたりしてる?
S1−{p0}からE(1)への写像は、図で言えば、p0を通りx軸と交わる直線を考えればよい。
この直線は、S1−{p0}上の一点とx軸(つまりE(1))上の一点を通っているから、
その2点を対応させれば全単射になっている。
具体的に2点の座標を求めれば質問の式が出てくる。

これを3次元空間内の2次元球面で考えてみる。
xy-平面に、中心(0, 0, 1)、半径1の球面が接していて(つまり南極が原点)、
p0=(0, 0, 2) (つまり北極)を通りxy-平面と交わる直線を考える。
この直線によって、S2-{p0} と xy-平面上の点の全単射が得られる。
具体的には自分で計算してみれ。
102132人目の素数さん:02/02/02 22:56
>>100
勝手に記号を変えて申し訳ないが、微積分でdはややこしいのでh(φ,θ)にします。
∫dvを座標変換によるヤコビアンで計算すれば言い。ヤコビアンは行列式なんだけど、なじみがなければ体積素を見た感じで直感的に計算すれば言い。
厳密な理論じゃなければそれでいい。

ある点の地球の中心からの距離をR、経度=φ、緯度=θとする。R,φ,θをわずかに動かした時に出来る直方体もどきの体積を求める
緯度方向の長さ=Rdθ 経度方向の長さ=Rcosθ・dφ (緯度って赤道が0度、だよね?) 高さ=dR
これらが直角を成しているから
dV=Rdθ・Rcosθ・dφ・dR=R^2*cosθ・dθdφdR

積分範囲は θ:―π/2〜π/2   φ:0〜2π  R:r-h(φ,θ)〜r
積分の順番はRからやる。

ひょっとしたらr≫h(φ,θ)としてφ,θの2重積分で良いのかもしれない。
その場合は、φ,θをわずかに動かした時の海面上に出来る面積に深さをかければ四角柱の体積が出るのでそれの総和を取る
つまり、r^2*h(φ,θ)*cosθ・dθdφ
積分範囲は  θ:―π/2〜π/2   φ:0〜2π

103ご冗談でしょう?名無しさん:02/02/03 02:23
マイナスとマイナスを掛けると何故プラスになるの?
>103
氏ね
105132人目の素数さん:02/02/03 02:30
>>104
Rを1を含む果敢なRingとします。
1の加法逆元の自乗は1に常になるでしょうか?
106132人目の素数さん :02/02/03 02:34
>>104>>103の問題が解けません
>>104ウジね
107132人目の素数さん:02/02/03 02:38
0=(1-1)^2=1^2+2*1*(-1)+(-1)^2=1-2+(-1)^2
だから当たり前.
ちょっとは考えろ>>105
>106
何言ってるのか若らんけど
さんざん語り尽くされてまだやるのか?そのネタ
109  :02/02/03 03:16
変数変換がx=3u+v,y=2u+3vで与えられた時、ヤコビアンは7になりましたが、
変数変換におけるヤコビアンで出た数字というのは、倍率みたいなものでしょうか?
例えば、1辺が1の正方形だったら、ヤコビアン倍の面積に移るってことですか?
110132人目の素数さん:02/02/03 03:31
ヤコビアンの絶対値倍の面積をもつ平行四辺形に移る
111  :02/02/03 03:33
>>110
なるほど。分かりました。どうもありがとうございました。

「位相群Gの閉部分群Hの正規化群はまた閉部分群になる」
どうやって証明したらよいのでしょうか?
>112
閉部分群の定義そのまま確認
114103:02/02/03 06:27
>>107
わかりましたー!
0=
{(-1)+1}X{(-1)+1}
=(-1)x(-1)+(-1)x1+1x(-1)+1x1
=(-1)x(-1)+(-1)+(-1)+1
=(-1)x(-1)+(-1)

ゆえに(-1)x(-1)=1

そういうことだったんですね。
11595:02/02/03 06:38
遅くなりましたが有難うございました。もう少し自分で考えてみます。
116132人目の素数さん:02/02/03 09:00
>>91
悪い、俺がわかっている(と思われる)とこまで。

(1)
つまりは、1000回の現象を「表」「裏」と分類するとき、次のように分類されたことになる。
 表  | 裏 
−−−+−−−
600  |400 

表の出る確率を P_1、裏の出る確率を P_2 とすると、
帰無仮説: P_1 = 0.5、P_2 = 0.5
対立仮説: P_2 ≠ 0.5、P_2 ≠ 0.5
となる。カイ2乗検定における検定統計量は

Q = Σ_[i=1 ,k]{(X_i - n*P^_i )^2/n*P^_i} = Σ_[i=1, k]{(X_i)^2/n*P^_i} - n

と表される、ここで X_i は 各分類の個数、 P^_i は各 P_i の最尤推定量、n は全標本数、k は分類数。
これは近似的に自由度 k-1 のカイ2乗分布に従う(但し、各 P_i が指定されているとき)。
であるので、棄却域 R を R={Q > χ^2_(k-1)(α)} と設定すると近似的に有意水準αの
カイ2乗検定となる。

今の場合、k=2、n=1000、α=0.05、X_1=600 、X_2=400 で
帰無仮説の元では、P^_1=0.5、P^_2=0.5 であるから
Q = 40、また P(χ^2≧4)=0.05 より χ^2(0.05)=4。
よって、Q は棄却域に入るので、対立仮説は有意、つまり、表の出る確率を 0.5 とはできない。
117132人目の素数さん:02/02/03 09:37
>>91
(2)
n回中 X回表が出る確率は、独立試行の公式から、
F(Po)=C[n, X] Po^X * (1-Po)^(n-X)

Poの最尤推定量とは、上のF(Po)を最大にするときの値Po^のこと。
これは、logF(Po)を最大にするのと同じことなので、(Po の変わりにθと書いて)

logF(θ) = log(C[n, X]) + X*log(θ)+(n-X)*log(1-θ)

を考える。logF(θ) を θ で微分した尤度方程式
{logF(θ)}' = X/θ − (n-X)/(1-θ) = 0
が求める解なので、θ = X/n である。

以後は、わからない単語が・・・
lim_[x→+∞](1+(3/x^2+x))^x^2

この問題ですが、対数微分法を使うところまでは分かったのですが、
その後の計算がなぜか上手くいきません。
こんな低レベルの問題で申し訳ありませんが教えてください・・・。
119132人目の素数さん:02/02/03 11:37
>>­118  対数微分法とやらはつかわない.lim[x->∞](1+1/x)^x=eの公式のみ.
lim_[x→+∞](1+(3/(x^2+x)))^x^2
=lim_[x→+∞][(1+(3/(x^2+x)))^{(x^2+x)/3}]^{(3x^2/(x^2+x))}
=lim_[x→+∞][(1+(3/(x^2+x)))^{(x^2+x)/3}]^{(3/(1+1/x)}
=e^3
>>119 ありがとうございます。
対数微分法は別な問題でした・・・。
今ちょっと頭危ないようです・・・。
121132人目の素数さん:02/02/03 18:16
リアル工房です。一つ疑問に思ったことがあるのでどうか教えてください。

a,b,c,kを定数としたときに、
∫[k,x]f(t)dt=ax^2+bx+cならば
x=kと置いてax^2+bx+c=ak^2+bk+c=0からcの値を求めると言う解法は説明が十分なんでしょうか?
これでは厳密性に欠けているならばなぜそうなのかを教えて下さい。これで大丈夫ならばいいのですが・・・
122100(海の体積):02/02/03 18:23
>>102
詳しい説明ありがとう。
微分要素の作り方が見えてきました。
123132人目の素数さん:02/02/03 19:20
下は平成7年の開成高の問題ですが,(1)がどういう誘導なのかわかりません。
問題集の解答では,(1)の結果を無視して(2)を方程式で解いていました。
ちなみに答は (1)80分 (2)A54km/時 B36km/時 です。

A,B2台の車が,60km離れた2地点P,Q間を,AはP地点を,BはQ地点を
同時にそれぞれQ,Pに向かって出発し,40分後に出会った。
AはQに到着後,20分間停車してからPに引き返し,BはPに到着後,
直ちにQに引き返したところ,帰路でA,Bは,初めてであった地点から
16.8kmP寄りの地点で再び出会った。ただし,A,Bはそれぞれ一定の
速さで走るものとする。

(1)仮に,AもQより直ちに引き返すとしたら,初めて出会ってから何分後に
2台の車は再び出会うか。
(2)A,Bの速さは,それぞれ時速何kmか。
124 ◆FHB7Ku.g :02/02/03 19:47
>>121
∫[k,x]f(t)dt=ax^2+bx+c・・・ア
∫f(t)d=F(t)とおくとアは
F(x)-F(k)=ax^2+bx+c・・・イ
イの両辺をxで微分すると
f(x)=2ax+b
またイにx=kを代入すると,ak^2+bk+c=0

よって得られる関係式は
f(x)=2ax+b,ak^2+bk+c=0・・・答
125132人目の素数さん:02/02/03 20:26
問題では無いのですが

(1/y)*{(d^2y)/(dx)^2}

などで
↑式はxの関数のみで成り立っているので定数とみなす。

というのをよく見るのですが何故そうなるのかが
わかりません。
例えばy=x^2だったとしたらxの値によって↑式は
変化するので定数ではないのでは?と思ってしまいます。
何故定数とみなせるのか教えて下さい。
>125
その文章が出てきた個所に拠るのでなんとも言えません。
127132人目の素数さん:02/02/03 20:41
>>126
そうなのですか?
私の見た問題では
{(d^2u)/(dt^2)}=(T/M)*{(d^2u)/(dx^2)} T,Mは定数
という問題の解法で
u(x,t)=X(x)*Y(t)として代入すると
(Y''/Y)=(T/M)*(X''/X)
両辺はおのおのt,xのみの関数なので定数とおける。

となっているのですが。。。
128 :02/02/03 20:54
1から10までの数字から順に3つを選んで左から順に並べる。2番目が2でなく、かつ3番目が3でない確率を求めよ。
まず2番目から考えて、2でないから9通り。3番目は3でないから8通り。1番目は残りの8通り。
よって確率は9×8×8/10P3=4/5
としたんですが違ってました。
この考え方はどこが間違ってるんですか?
>>128
>まず2番目から考えて、2でないから9通り。3番目は3でないから8通り。1番目は残りの8通り。

君のように2番目→3番目→1番目の順に決めたとする。
確かに2番目は2でないから9通りだ。しかし、この「2番目」を
 3以外にするか、3にするか
で、3番目の決め方が変わってくるのだ!

つまり、
 ○「2番目」に3以外を並べるとき
 ○「2番目」に3を並べるとき
で分けなくてはダメ。
130129:02/02/03 21:08
>>128
その前に問題の「1から10までの数字から」ってのは
「0から9までの数字から」
じゃないのか?
それとも「1から9までの数字から」
じゃないの?

131 ◆FHB7Ku.g :02/02/03 21:11
>>123
確かに1と2の関係がわからない・・。独立題??
(2)
Aの速さをVA,Bの速さをVB,2回目に出会うときの時刻をtとおくと,
VA*(40/60)=60-VB*(40/60)
VB*(t-60/VB)=60-VA*{t-(60/VA+20/60)}=VA*(40/60)-16.8
∴VA^2+630VA-270*136.8=0
よってVA=54,VB=90-54=36・・・答
132 ◆FHB7Ku.g :02/02/03 21:31
>>123
(1)求める時刻をtとおいて,
VA*(40/60)=60-VB*(40/60)
VB*(t-60/VB)=60-VA*(t-60/VA)
∴t=2
∴120分・・・答
あれ,答と違った・・鬱
133 ◆FHB7Ku.g :02/02/03 21:36
>>132自己レス
そうか,始めて出会ってから何分か、だから
120-40=80分・・・答
だ。
でも(1)と(2)の関係はわからないです。
>127
その場合は、u(x,t)の変数x、tは独立に取れる
xを止めてtを動かすこともできるし
tを止めてxを動かしてもなんの問題も無く
u(x,t)の値が決まる

(Y''/Y)=(T/M)*(X''/X)
の左辺はtだけの関数で
右辺はxだけの関数

tを止めると左辺は定数
xだけ動かしてもこの式は成り立っているはずだから
右辺も定数だからこの式の値はxに拠らない

同様にxを止めてtだけ動かすとtに拠らないことが
わかるので定数
135132人目の素数さん:02/02/03 21:49
数論の内容なのですがQ(√5)、Q(√6)、Q(√7)、の単数群を求めていただけないでしょうか?

自分で少しいじってみたのですが、答えに自信が無くて・・・
よろしくお願いします。
136関大の試験問題:02/02/03 21:49
さいころを3回投げて、出る目の数を順にX1,X2,X3とする。
この時(  )をうめよ。

(1)X3=(X1+X2)/2となる確立は( )/216である。

(2)X3<(X1+X2)/2となる確立は( )/216である。

(3)X3>(X1+X2)/2となる確立は( )/216である。

(4)X3=(X1*X2)^1/2となる確立は( )/216である。

(5)X3がX1,X2の少なくとも1つと一致する確立は( )/216である。

(補足)(4)はルートのつもりです。
    もしかしたら分数の書き方が逆になってるかもしれないです。。。
137 ◆FHB7Ku.g :02/02/03 21:50
>>128,129
問題文を「0から9までの数字」と直すと・・
ベン図をかいて計算・・。
U:全体
A:2番目が2である
B:3番目が3である

とすると,
Uの個数=10P3=720個
Aの個数=9P2=9*8=72個
Bの個数=9P2=9*8=72個
AかつBの個数=8個
よって「AバーかつBバー」の個数は,全体からベン図の2つの○部分を除いた
ものなので,
720-(72+72-8)=584個・・・答
138見にくかったので修正:02/02/03 21:50
さいころを3回投げて、出る目の数を順にX1,X2,X3とする。
この時(  )をうめよ。

(1) X3=(X1+X2)/2となる確立は( )/216である。

(2) X3<(X1+X2)/2となる確立は( )/216である。

(3) X3>(X1+X2)/2となる確立は( )/216である。

(4) X3=(X1*X2)^1/2となる確立は( )/216である。

(5) X3がX1,X2の少なくとも1つと一致する確立は( )/216である。

(補足)(4)はルートのつもりです。
    もしかしたら分数の書き方が逆になってるかもしれないです。。。
139137:02/02/03 21:51
今日の関大の問題です。
回答どこにもないのでよろしくお願いします。
140  :02/02/03 21:54
Cは原点を中心とする半径1の円で向きは半時計回り(反時計回り?問題文の間違いか?)
とする。
∫_C (2x^2+y)+(x-2y^2)dyの積分を計算しろ

って、問題がわかりません。
グリーンの定理より0になるのかな?とも、思いましたが「向き」ってのがひっかかります。
どうか教えてください。
141137:02/02/03 21:55
(1)って24出会ってる?
142132人目の素数さん:02/02/03 21:56
>>135
自分でどういじってどういう答えが出たのか書くように。
143 :02/02/03 22:01
>>141
18?
144137:02/02/03 22:05
>>143
ほんとに?
ハァ・・・。終わった。。。
145 :02/02/03 22:05
>>144
なだれしきに間違えか・・・
146132人目の素数さん:02/02/03 22:08
>>140
∫_C {(2x^2+y)+(x-2y^2)}dy ?
147  :02/02/03 22:09
>>146
あ!間違えてました。

∫_C (2x^2+y)dx+(x-2y^2)dy

です。
148 ◆FHB7Ku.g :02/02/03 22:13
>>141自信ないけど・・
x1とx2がともに偶数か共に奇数でなくてはならないことより
(x1,x2)=(1,3,5のうち2つ)OR(2,4,6のうち2つ)
x1,x2が定まれば自動的にx3が定まるから,,
3P2+3P2=12
149137:02/02/03 22:14
(1,1)とかは2倍しなくていいのか。。。
やっぱり間違えた。
(2)(3)は、216-(1)/2でだして、(4)は数えたけど(1)と同じ間違いしてるとおもう。
(2)(3)の考え方は間違ってますか?
150 :02/02/03 22:14
>>147
なら0だよ
151 :02/02/03 22:16
>>149
上から
18
99
99
8
66
たぶん
152137:02/02/03 22:16

{216-(1の答え)}/2です
154 :02/02/03 22:19
>>152
考え方はあってるよ。
155 :02/02/03 22:20
(5)は
すべてが異なる確率を
1*6/5*6/4=120/216をだして
3でわって2倍して6引きました。
156132人目の素数さん:02/02/03 22:21
>>147
とにかく閉曲線を1周するならグリーンの定理より0でしょ。
(2x^2+y)dx+(x-2y^2)dy = d(xy-(2/3)y^3+(2/3)x^3)
だから0、と言ってもいい。
157137:02/02/03 22:22
(5)は
すべてが異なる確率を
1*6/5*6/4=120/216をだして
3でわって2倍して6引きました。
これは考え方はあってますか?
>138
(1)X1とX2がともに偶数かともに奇数の時式を満たすX3が唯一存在
X3はこの数字しか駄目
X1とX2がともに偶数なのは3*3=9通り
ともに奇数なのも9通りなので18通り

(2)
X1=6の時、X2=6、5、4、3、2、1に対して
X3は5以下、5以下、4以下、4以下、3以下、3以下
となるので24通り

X1=5の時、これが一つずれて
X3は5以下、4以下、4以下、3以下、3以下、2以下
となるので21通り

以下同様にX1=4で18通り、X1=3で15通り、X1=2で12通り
X1=1で9通り(3つずつ減っていく)

全部足すと99通り

(3)
全体から(1)と(2)を引いて216−99−18=99通り

(4)(1)と同様X1とX2だけでX3は一通りに決まる
X1=X2の6通りと
(1、4)、(4、1)の8通り

(5)
X1=X2の時、X3も同じ値でなければならない
これは6通り

X1≠X2の時、これは全部で6*5通りあるが
それぞれの場合においてX3は2通りあるので60通り

あわせて66通り
159  :02/02/03 22:22
>>150
そうですか、ありがとうございます!

グリーンの定理で解いてOKですか?
あと、半時計方向ってのが気になるんですが、関係ないんでしょうか?
気になるので教えてください。
160 ◆FHB7Ku.g :02/02/03 22:22
すいません
>>148はまちがい。重複してもいいから,(1)は
3*3+3*3=18でした。
161137:02/02/03 22:24
間違えた。整理してから書き直します。
162  :02/02/03 22:24
>>156
なるほど。親切にありがとうございました。
163 :02/02/03 22:25
>>159
関係ないでしょう。

>>160
さいころ2個振ったら目の和は偶数と奇数で同確率ですよね。
164132人目の素数さん:02/02/03 22:27
>>123
>>138
関大と開成高校では後者の方が難しい出題をすることがわかった(w
165137:02/02/03 22:27
>>158さん
ありがとうございます。
なんか全滅っぽい。。。
166 :02/02/03 22:28
>>165
終わったことを考えてもしょうがないからな。
他のとこは頑張れよ
受験生だったら

(2) X3<(X1+X2)/2となる確立は( )/216である。

(3) X3>(X1+X2)/2となる確立は( )/216である。
これは
(6-X3)<((6-X1)+(6-X2))/2

で(2)と同じ確率になる筈なので

全体の216から(1)の18を引いて2で割って
(2)と(3)は共に99というのも当然アリ
(1)で間違えると痛いけどね
168137:02/02/03 22:29
>>164さん
総合情報は数学UBまでなのでちょっと文型よりの学部なんですよ。
169132人目の素数さん:02/02/03 22:33
開成高(受験者は中3)>>>関大総合情報(受験者は高3、浪人)
170 :02/02/03 22:34
>>169
学歴板にでもいってください。
171132人目の素数さん:02/02/03 22:37
>>134
どうもありがとうございます。
ちょっと考えてみますです。
172 ◆FHB7Ku.g :02/02/03 22:40
質問なんですが、
さっきのサイコロの問題で、N回サイコロをふった場合、
Xn>(Σ[k=1,n]Xk)/n
を満たす(X1,x2,…,Xn)の個数と,
Xn<(Σ[k=1,n]Xk)/n
を満たす(X1,x2,…,Xn)の個数は等しいですか。
173 :02/02/03 22:42
>>172
等しいよ。
>172
>167と同様の変形により等しい。

サイコロの目をそのまま読むか
6から引いて読むかの違いしかないから
175 ◆FHB7Ku.g :02/02/03 22:44
>>172訂正・・
X(n)>(Σ[k=1,n]X(k-1))/(n-1)の個数とX(n)<(Σ[k=1,n]X(k-1))/(n-1)
の個数の誤りでした。

176 :02/02/03 22:45
>>175
X(0)は何?
>175
X(0)てのは何だ?
178 ◆FHB7Ku.g :02/02/03 22:46
>>173,174
6からひけばいいから同じですよね・・。
ありがとうございます。
179 :02/02/03 22:46
>175
訂正しなくて良かったのに
>167=>174
冷静に考えてみると6から引くのではなく
7から引くのでした。
181 ◆FHB7Ku.g :02/02/03 22:48
X(n)>(Σ[k=1,n-1]Xk)/(n-1) (n≧2)
というべきか・・。
182132人目の素数さん:02/02/03 22:51
2ゲット!!!!!!

やった、ついに俺はやったぞ!!!!!!!
感動で涙が止まらない。

>>3へ    おせ〜んだよ(ププ
>>4へ    2ちゃん辞めろ(ププ
>>5へ    人間辞めろ(ププ
183 :02/02/03 22:52
>>181
>>172 demo onaji dayo
三次元のガウス・グリーンの証明見てて思ったんですけど
xyz空間で、球面がSが(x、y、ψ(x、y))(=Xとする)で与えられていたら
その球面上の任意の点Pでベクトル(∂X/∂x)と(∂X/∂y)の外積のz成分が1になる
ように読み取れる個所があったんですけど、これって正しいんでしょうか?
馬鹿な質問かもしれませんけど、よろしくお願いします。
185184:02/02/04 00:40
日本語おかしいですね、すいません。
186質問です:02/02/04 01:05
整数の定義で
Zは加法減法に関して閉じたNを含むRの最小の部分である。
 
となっていますが、この「閉じた」とはどういう意味なのか
教えてください。
>184
外積計算すれば自明。

>186
ある集合Sがある演算・について閉じているとは
Sから元を持ってきて演算をした時に、得られる値が
Sの元であるということです。

その場合ならZの元を持ってきて、加法なり減法なりした
結果は、Zの元ってことです。
>>187
ありがとうございます。
あと、本当に馬鹿な質問で申し訳ないですけど
でもz軸に平行な曲面上では外積のz成分0になりますよね?
感覚としては分かるような気がするんですけど論理的に分からないんです。

あと、もう一つ追加質問です。
微分可能な関数の導関数は必ずしも連続じゃない、と微積の本に書いてあったんですけど
複素関数の本にはある関数が複素微分可能ならそれの導関数も連続である…(1)事が導かれると書いてありました。
この点について複素微分と微分ってどう違うんでしょう?
(1)の証明も兼ねて教えて頂けないでしょうか?
>この点について複素微分と微分ってどう違うんでしょう?

微分可能の定義を見れば違いがわかるでしょ。
lim(f(z+Δz)-f(z))/Δzの分母Δzが複素数か実数かの違い。
>>189
いや、でもそれだけだと、なんで導関数が連続になるのか分からないんですけど・・・
>188
>でもz軸に平行な曲面上では外積のz成分0になりますよね?

座標を取り替えればいいだけ。


複素関数を微分する時は、実関数の時と違い
lim{f(z+h)-f(z)}/hという式でhを
どの方向から0に近づけても同じ導関数を定義できなければ
ならないから、複素平面というのは単に軸が2本というわけではない

複素関数の場合は1回微分可能なら、無限回微分可能でもあることの証明は
打つのが面倒なので解析概論でも読んで下さい。正則関数とか
コーシーの積分公式とか書いてあるあたりにあると思います。
>>191
どうもありがとうございます。
でも最後にこれだけ。
>座標を取り替えればいいだけ。
具体的にどうやるのでしょうか?
193死にかけ坊主:02/02/04 02:35
関数、f:R→R,f(x)=3x^2+xに対し、2におけるfの変化量を
最も良く近似する一次関数と、それらの誤差を求めよ。

説いて下さい。お願いします
194132人目の素数さん:02/02/04 02:41
cos(x+y)=cosx*cosy-sinx*siny
sin(x+y)=sinx*cosy+cosx*siny
加法定理を図形で証明するのはどうやるのですか。
教えてください。
>192
証明の流れに拠る。
196  :02/02/04 03:31
y''+2y'+3y=0を解いて、一般解を求めよ。

答えは、y=e^(-x)(C_1cos√2x+C_2sin√2x)ってなったんですが、(C_1、C_2は任意定数)
問題文に、実数値関数で求めよってあるんです。
どういうことでしょうか?教えてください。
197132人目の素数さん:02/02/04 03:40
複素数値関数の解もあるけど
実数値関数の解をこたえよってこと
198  :02/02/04 03:52
>>197
え?どういうことでしょうか?
特性方程式より、λ=-1±i√2になったんで、
y=e^(-x)(C_1cos√2x+C_2sin√2x) ってしたんですが、間違ってますか?
199 ◆pvySbQO2 :02/02/04 04:00
200132人目の素数さん:02/02/04 04:03
>>198
間違ってません。
ちなみに y=exp(λx)(λは定数)という関数が y"+2y'+3y=0
の解だとするとλ=-1±i√2である(このとき確かに解になる)。
だから
C_1*exp((-1+√(-2))x) + C_2exp((-1-√(-2))x)
(C_1、C_2は任意定数)
も一般解。だけどこれだと複素数値関数が出てきてしまう。
こういう表示はヤメてくれというのが「実数値関数で求めよ」
という要求の意味。変な要求だとおもうが出題者が言うんだから
仕方ない。
201132人目の素数さん:02/02/04 04:08
ちなみに
exp((-1+√(-2))x) = exp(-x){cos((√2)x)+i*sin((√2)x)}
exp((-1-√(-2))x) = exp(-x){cos((√2)x)-i*sin((√2)x)}
だから200の表示を198の表示に変えるのは簡単。
202  :02/02/04 04:09
>>200
i抜けてませんか?

分かりました。なるほど。では、ハジメのであってるということですね。
指数でも表せるが、複素数がからんで嫌と・・・
203  :02/02/04 04:10
あ、ルートの中がマイナスか。
204132人目の素数さん:02/02/04 04:10
 >194
数学Uの教科書に思いっきりのってると思うのですが。
一応書いておくと、
@単位円書きます。横にx軸、縦にy軸をとります。
Ax軸から角度αだけ回転したところに線を入れ、
そこから更にβ回転したところに線を入れます。
Bx軸から角度−αだけ回転したところに線を入れ、
x軸から角度βだけ回転したところに線を入れます。
AとBによってできる三角形は合同なので、
原点の対辺の長さが等しいことを云って終了。
205200:02/02/04 04:12
漏れの場合 λ=-1±i√2 からすぐ出てくる解は
exp((-1±√(-2))x) なんだけど、、、人それぞれってことか。
206  :02/02/04 04:18
>>205
いや、僕もそっちでも出せますが・・・
なんとなく、
異なる実解(x=α,β)の時 C_1e^(αx)+C_2e^(βx)
重解のとき (C_1+C_2x)e^(αx)
異なる複素解(x-α±iβ)の時 =e^(αx)(C_1cosβx+C_2sinβx)

としてます。複素解でも実解の時のようにやればいいんですが・・・
207132人目の素数さん:02/02/04 07:34
漸化式
a[n+1]=a[n]/(2+n*2^(n-1)*a[n]) a[1]=1のとき、
a[n]を求めて頂けないでしょうか。おねがいします。
208132人目の素数さん:02/02/04 08:09
最初の5個ぐらい出してみれば…
>207
2^(n-2)*a[n]*b[n]=1とすれば
漸化式 ⇔ b[n+1]=b[n]+n
b[n]は簡単
210132人目の素数さん:02/02/04 12:28
重積分  ∬[D]log(x^2+y^2)dxdy [D]=[x^2+y^2≦1]
の解き方を教えていただけないでしょうか?
211132人目の素数さん:02/02/04 12:54
>>210
極座標に移行すれば?
212132人目の素数さん:02/02/04 12:58
∬[半径1の円](rlog(r))drdθ
あとは簡単、変数変換して。
213212:02/02/04 13:07
違う、置換積分だった。
214132人目の素数さん:02/02/04 13:45
(x,y,z)-空間内の円柱面:x^2+y^2=2ax(a>0)、
放物面:z=x^2+y^2、および平面:z=0によって囲まれる部分の体積を求めよ。
という問題が分かりません。教えてください。
215( ´D`):02/02/04 14:50
>214しゃん
(5/8)πa^3 れす.やり方を今から打ち込むれす.
216( ´D`):02/02/04 15:30
>214しゃん
平面z=k^2で立体を切るのれす.切り口の面積をS(k)とすると 求める体積Vは V=∫[k=0→2a]S(k)dk.
S(k)=πa^2- [ α*a^2-(a-k^2/2a)*√{k^2-(k^2/2a)^2} ] - [ β*a^2-(k^2/2a)*√{k^2-(k^2/2a)^2}]
     ( ただし cosα=1-k^2/(2a^2),cosβ=k/(2a) よって cosα=1-2(cosβ)^2 ∴α+2β=π )
   =(π-α)a^2-βk^2+a√{k^2-(k^2/2a)^2}
   =2βa^2-βk^2+ak√{1-(k/2a)^2}
   =2βa^2-β(2acosβ)^2+a(2acosβ)sinβ  (∵k=2a*cosβ)
   =2βa^2-2βa^2(cos2β+1)+a^2sin2β
   =a^2(sin2β-2βcos2β)
k=2a*cosβかつk:0→2a より β:π/2→0 そして dk/dβ=-2asinβ 以後β=b
V=∫[k=0→2a]S(k)dk.
 =∫[b=π/2→0]a^2(sin2b-2bcos2b)*(-2asinb)db
 =∫[b=0→π/2]a^3(2(sin2b)^2-2bsin2b*cos2b)db
 =∫[b=0→π/2]a^3((1-cos4b)-bsin4b)db
V/a^2=[b-(1/4)sin4b-(-(1/4)bcos4b)][0, π/2]-∫[b=0→π/2](cos4b)db
    =π/2+π/8-[(1/4)sin4b][0, π/2]
    =(5/8)π
∴V=(5/8)πa^3・・・ans.res

圭さんが半端じゃ無いのれす.ヽ( ´;D;`)ノ ウワーン
多分これを見たって分からないと思うのれす.でも説明するのは大変なのれす.
( ´D`)< ふひゅ〜〜〜,れす.
>>216たん
お疲れさん。解析系の授業での定番問題だよね。
マグロウヒル大学演習に、例題として載ってなかった?
218まーこ:02/02/04 15:50
このなかに楕円曲線上に有理点が存在しない条件を求められる人いますか?
現在このねたでレポート作成中です。
もしアドバイスしてもいいという方がいらっしゃいましたら、レスお願いします
有志募るゴルァ
219(; ´D`)ノ:02/02/04 16:34
>217しゃん
マグワロウヨルとか言われてもノノは全然分からないのれす.
この問題自体は初めて見たのれす.ののみはまだかよわい乙女なのれす.
いつもお菓子を食べて頑張っているのれす.これからも応援ヨロシクなのれす.
220( ´D`):02/02/04 16:44
http://akimimi.ath.cx/gbbs/img-box/img20020204164152.jpg
ののはこういう女の子なのれす.頭良いのれす.
221210:02/02/04 17:45
>>212
レスありがとうございます。返事が遅れてすみません。
でも、解けないです。
Yをθに変換するとき、定義域はどうなるのかいまいち分かりません。
222132人目の素数さん:02/02/04 17:53
あの、最近証明された、古くから有る問題で、
MのN乗プラスMプラス1のN乗=LのN乗
を満たす組は
2乗以上ではないとかいうやつ(たぶんおぼえまちがってます)
の、名前ってなんでした?
ティ子部ラーえとか関係ないのばっか思い出せる
223質問です。:02/02/04 18:14
問題!
y=e^xの関数の 0≦x≦logA (A>1)間の曲線の長さを求めよ。

ルートのはずし方とかがわかりません。
誰か教えてください!
224頭いい人おしえてください:02/02/04 18:19
曲線y=|x(x−1)|、・・・@と直線y=ax・・・Aで囲まれた図形の
面積の面積をS(a)とする(ただし、0≦a≦1)

(1)S(a)をもとめよ
(2)S(a)を最小にするaの値をもとめよ
225224:02/02/04 18:22
ちなみに高校の積分のところです
>>223
e^x=y と置換、さらに y=sinh(θ)と置換すると
原始関数は log(tanh(θ/2))+cosh(θ)
となります。
227質問です。:02/02/04 18:28
>>226
あ、そっちにもってくんですか。
でも僕、sinh(θ)とかtanh(θ)てよくわからないんです・・・

e^x=tanθとおくやり方ではできませんか?

それと、もし答えが分かったら教えてもらえるとありがたいんですが・・・
228( ´D`):02/02/04 18:32
>227
t=√(1+e^(2x)) とおくれす.そしたら t:√2→√(1+A^2)  dt/dx=e^(2x)/√(1+e^(2x))=(t^2-1)/t
だから
長さ=∫[x=0, logA]√(1+e^(2x))dx=∫[t=√2→√(1+A^2)]{t^2/(t^2-1)}dt
=∫[t=√2→√(1+A^2)]{1+1/(t^2-1)}dt
=√(1+A^2)-1+∫[t=√2→√(1+A^2)](1/2){1/(t-1)-1/(t+1)}dt
==√(1+A^2)-1+ 〜・・・ とやっていくれす
229質問です。:02/02/04 18:35
>>228
答えはわかります?
>>227
答えは √(1+A^2)-√(2)+log(A(1+√2)/(1+√(1+A^2)))
になると思います。

tanθの置換でもできるんじゃないでしょうか.
231224:02/02/04 19:02
224もおねがいします
232132人目の素数さん:02/02/04 19:47
解き方が分かりません。教えてださい

∫[0,1](1/2x)*(e^x^2)dx

233質問です:02/02/04 20:00
友達に質問を出されたのですが、わかりません。
1回バウンドするごとに半分の高さバウンドするボールがあったとします。
バウンドの軌跡=バウンドの所要時間とします。
縦軸にバウンドの回数、横軸に時間、をとってグラフにしたところを
思い浮かべてください。漸近線ができます。その漸近線の時間には
ボールはいったいどんな運動をしているのでしょうか?


234質問です:02/02/04 20:01
1次写像f:R^3→R^3はR^3の基底
u=[1,1,0]
v=[0,1,1]
w=[1,0,1]
基底{u,v,w}に関するfの表現行列を求めよ

途中の過程が特に知りたいです。
235132人目の素数さん:02/02/04 20:19
曲線log x の接線が原点を通るとき,その接線の式を求めよ

わかりません
答えはe/xだったと思います。過程を教えてください
>233
>漸近線ができます。

その漸近線というのはどうやって定義されてるの?
なんとなくこんな感じで線が見えるとかではなしに
一意に決められてる?
237質問です:02/02/04 20:21
n
Σ√k = f(n)
k=1

この式の n が実数のときにも定義できるf(n)を教えて下さい。
>234
>1次写像f:R^3→R^3はR^3の基底

何が言いたいのかわかりません
239224:02/02/04 20:24
224もあねがいします(・∀・)
240132人目の素数さん:02/02/04 20:39
>>238
ヽ(`Д´)ノ ウワァァン!!
問題にそんな風に書いていて自分も意味わからないのよー
>240
問題文が忠実に再現されているのであれば
「日本語を書いてくれないと何が言いたいのかわかりません」
とでも書いて置いてください。
>>234
元の(ないし線形変換後)の基底は何でしょう?
それがないと求まりません
243132人目の素数さん:02/02/04 20:57
>235
x座標 x=t での接線は、傾き 1/t で、接点(t, logt)を通るから、
y=(x-t)/t + logt
これが原点を通るとき、x=y=0 を代入して、t=eになる。
よって答えは y=x/e
>>242
標準基底じゃない?
245132人目の素数さん:02/02/04 20:59
>>240>>242
すみません書き損じがありました。
u=[1,1,0]
v=[0,1,1]
w=[1,0,1]
をそれぞれ
p=[0,1,-1]
q=[-1,1,0]
r=[1,-2,1]
に写す。

ホントすいません・・・
>244
計算するまでもないと思われ(w
>>239
あまりに簡単だからみんなレスしてないのだと思われる。
どこが分からないの?
248 ◆FHB7Ku.g :02/02/04 21:04
>>224
(1)
直線y=axと曲線y=|x-x^2|の交点のx座標は0<a<1を考えて,
x=1-a,1+a
よって
S(a)=∫[0,1-a](x-x^2-ax)dx+∫[1-a,1](ax-x+x^2)dx+∫[1,1+a](ax-x^2+x)dx
∴S(a)=(-1/6)*(a^3-9a^2+3a-1)・・・答

(2)
S'(a)=(-1/2)*(a^2-6a+1)
よって0<a<3-2√2でS'(a)>0,3-2√2<a<1でS'(a)<0
∴a=3-2√2・・・答
249237:02/02/04 21:05
書き直しました。

f(n)=Σ_[k=1,n]k^(1/2)

nを実数まで拡張したf(n)の定義を教えてください。
250 ◆FHB7Ku.g :02/02/04 21:06
>224みたいな問題って計算しなくても解ける方法ってあるのかな?
はみだしなんとか法とかって聞いたことあるけど・・
251132人目の素数さん:02/02/04 21:09
>245
変換行列をAとして
uA=p
vA=q
wA=r

なる連立方程式を解く

すなわち
[1,1,0]   [0,1,-1]
[0,1,1]A=[-1,1,0]
[1,0,1]   [1,-2,1]

なる行列Aを求める
252234:02/02/04 21:23
>>251
ありがとうございます
できました
253:02/02/04 21:32
異常者
254 ◆FHB7Ku.g :02/02/04 21:50
>>214の問題ですが,,,y=kで切ると切り口のS(k)は
S(k)=2∫[0,a+√(a^2-k^2)](x^2+k^2)dk=(2/3)*{a+√(a^2-k^2)}^3+2k^2*{a+√(a^2-k^2)}
∴V=(4/3)*∫[0,a]{a+√(a^2-k^2)}^3dk+4*∫[0,a]k^2*{a+√(a^2-k^2)}dk
となる?けど,最初の部分の積分計算ができないですね・・・
やっぱりz=k(k^2)で切るほうが計算楽ですか??
あと,もしこの問題が模試で出た場合(記述式),何分で解かなければいけないでしょう?
立式までで,部分点くれるのかなとか・・最近そういうのも気になるので…
詳しい方,お願いします。。。
>>221
その認識はまずい。
重積分って事は今大学生ですよね。
極座標と直交座標について学んだこと無いですか…?

(基本)
r^2=x^2+y^2
x=rcosθ
y=rsinθ
(解析学で習う)
直交座標->極座標(円柱座標)へのヤコビアン = r
領域D: 半径1の円 ∴0<=r<=1

さぁ、θの範囲は?
円をぐるっと積分するんだから…?
256132人目の素数さん:02/02/04 21:57
>254
>∫{a+√(a^2-k^2)}^3dk
>∫k^2*{a+√(a^2-k^2)}dk
どちらの場合も

k=a cos θ

で計算できるはずですが?
257 ◆FHB7Ku.g :02/02/04 22:02
>>256
すいません,書き間違いです・・
「最初の部分の積分計算がめんどう・・」
にして読んでください・・。
258132人目の素数さん:02/02/04 22:06
>>257
どっちの解法にせよ今やっておけば、
似た問題ならすぐに解けるようになるのでは…
お願いします。
R^4のベクトルの
[1 ] [1 ]
a=[0 ] b=[-1]
[2 ] [1 ]
[-1] [-3]

の内積、長さ、なす角θなんですが。
260132人目の素数さん:02/02/04 22:46
>>259
ネタですか?スレ違いですよ。
261ごくう:02/02/04 22:48
>>259
そんなこといってると、かめはめ波撃つぞ。
か〜め〜・・・
>>259
内積
1*1+0*(-1)+2*1+(-1)*(-3)=6
長さ
a=(1^2+2^2+(-1)^2)^0.5=6^0.5
b=(1^2+(-1)^2+1^2+(-3)^2)^0.5=2*3^0.5
cosθ=6/((6^0.5)*(2*3^0.5))
=1/(2^0.5)
θ=π/4
>>262
どうもありがとうございました。
どうやらスレ違いだったみたいですいません。
わざわざご返答ありがとうございました。
俺はまだ高1なんですけど、大学の兄がわからないからネットで調べてくれって言うんで、質問したんですが、
住人の皆さんご迷惑おかけしました。
264質問です:02/02/04 23:32
V={(x,y,z) ; x^2+y^2≦a^2 , y^2+z^2≦a^2} (a>0)
この立体の体積を3重積分をつかって解きたいです。
>>264
(16a^3)/3
266132人目の素数さん:02/02/04 23:41
>>265
ありがとうございます
267132人目の素数さん:02/02/04 23:51
>>259
4次元のなす角の意味も分かってないのに計算してそうなアホだな。
268  :02/02/05 01:16

x = 2cosθ + sinθ
y = cosθ + asinθ
(0≦θ≦π/2)

(1)xのとりうる値の範囲を求めよ
(2)yのとりうる値の範囲を求めよ
269 :02/02/05 01:16
↑よくわからないのでおねがいします
今日の理科大の問題です
x = 2cosθ + sinθ
 =(2,1)・(cosθ,sinθ)
だから(図をかいて)
1≦x≦√5
y=cosθ + asinθ
 =(1,a)・(cosθ,sinθ)
なので(図を書いて)
1≦y≦√(a^2+1)   (1≦aのとき)
a≦y≦√(a^2+1)   (a<1のとき)


自信ないな。間違ってたらスマソ
>268
x = 2cosθ + sinθ
=√5 sin(θ+α)

ただしαはxy座標平面で原点と(1,2)を結ぶ線分と x軸の成す角(0<α<π/2)
(0≦θ≦π/2) で1≦x≦√5

y = cosθ + a sinθ
=√(1+a^2) sin(θ+β)

ただしβはxy座標平面で原点と(a,1)を結ぶ線分と、x軸の成す角(0<β<π/2)
(0≦θ≦π/2) でmin{1,a}≦y≦√(1+a^2)
272 ◆FHB7Ku.g :02/02/05 01:57
>>268
(1)x=√5sin(θ+α) (sinα=2/√5,cosα=1/√5)
sinα=2/√5,sin(α+π/2)=cosα=1/√5
だから1/√5≦sin(θ+α)≦1
∴1≦x≦√5・・・答

(2)
y=√(a^2+1)*sin(θ+α) (sinα=1/√(a^2+1),cosα=a/√(a^2+1))
0≦α≦π/4のときsinα≦sin(θ+α)≦1
π/4≦α≦π/2のときcosα≦sin(θ+α)≦1
π/2≦α≦πのときcosα≦sin(θ+α)≦sinα
よって,
a≦0のとき,a≦y≦1
0≦a≦1のとき,a≦y≦√(a^2+1)
1≦aのとき1≦y≦√(a^2+1)
【入試】東京理科大学入試反省会【入試】
http://school.2ch.net/test/read.cgi/kouri/1012712276/

74 名前:神楽坂 投稿日:02/02/04 (月) 23:28 ID:syvoVYa3

【2】より一部抜粋

x = 2cosθ + sinθ
y = cosθ + asinθ
(0≦θ≦π/2)

(1)xのとりうる値の範囲を求めよ
(2)yのとりうる値の範囲を求めよ


77 名前:神楽坂 投稿日:02/02/04 (月) 23:31 ID:syvoVYa3

>>74
付け足し。
aは1より大きい定数
274tr:02/02/05 01:58
# ありゃ?答えがまちまち (汗)

>>268 さん
(1) ↑s = (2,1), ↑t = (cosθ,sinθ),
また ↑s, ↑t のなす角φとして
  x = (2,1)・(cosθ, sinθ) ← 内積
   = √5*1*cosφ
0≦θ≦π/2 のとき, 1≧φ≧1/√5 だから
  1≦x≦√5

(2) ↑u = (1,a), ↑v = (cosθ,sinθ),
また ↑u,↑v のなす角をφとして
  y = √(1+a^2)*1*cosφ

i) 0≦a≦1 の場合
  a/√(1+a^2)≦cosφ≦1 ⇒ a≦y≦√(1+a^2)
ii) 1<a の場合
  1/√(1+a^2)≦cosφ≦1 ⇒ 1≦y≦√(1+a^2)
iii) a<0 の場合
  1/√(1+a^2)≧cosφ≧a/√(1+a^2) ⇒ 1≧y≧a
275tr:02/02/05 02:02
かぶりまくりやがり。(>_<) ごめんにゃ
276270:02/02/05 02:09
0>a  のときを全然考えてなかった。反省
277理科大受験者:02/02/05 02:14
受験番号  氏名
270 132人目の素数さん
271 132人目の素数さん
272 ◆FHB7Ku.g
274 tr

どれが正解?
278/名無しさん[1-30].jpg:02/02/05 02:21
正確な問題はどれよ
279/名無しさん[1-30].jpg:02/02/05 02:23
付け足し。
aは1より大きい定数

ってのはどうすんだ

>>277
272
が正解。
281/名無しさん[1-30].jpg:02/02/05 02:27
>>280
他の答えはどうして不正解なんですか?
>>281
場合わけ不完全な気がするから
283/名無しさん[1-30].jpg:02/02/05 02:31
>>282
解法自体に問題はありませんか?
受験番号  氏名         解法      解答
270 132人目の素数さん  ベクトルの内積  △
271 132人目の素数さん  三角関数の合成  △
272 ◆FHB7Ku.g       三角関数の合成  ○
274 tr           ベクトルの内積? ?(わけわかんない答の出し方)
285132人目の素数さん:02/02/05 02:39
横レススマソ。
そもそも本を読んでて何いってるのか意味がさっぱりわからないときって、
みなさんどうしてますか?
286132人目の素数さん:02/02/05 02:52
>>285
窓開けて大声を出してみる
287132人目の素数さん:02/02/05 02:52
ちなみに私は寝るのが一番いいように思います。
思いつきでいろいろやってみてあせって放棄してしまうこともよくわりますが、
わかるときはふとした瞬間に、そうだったのか!ってわかるもの。
何回寝たか、っていうのが結構重要だと思うのです。
288132人目の素数さん:02/02/05 03:00
自分に激しく自己嫌悪し、頭にきてみたりもしますが、
やはり気分を落ち着け、寝ます。さて寝るか。
>>286
寝る前に2chに叫びます。

あああああああああああああああああああああああああああああああああああああああああああああああああああああああああああああ
あああああああああああああああああああああああああああああああああああああああああああああああああああああああああああああ
あああああああああああああああああああああああああああああああああああああああああああああああああああああああああああああ
ああああああああ・・・・、あ、、、。。。。

やはり基地外だ・・・鬱打氏脳

290132人目の素数さん:02/02/05 03:25
袋の中に赤玉がA個、青玉がB個、白玉がC個入っている。(A,B,C≧1)
袋から玉を1個取り出す試行をXとする。なお、1度引いた玉は袋に戻さない。

(1)
1回目の試行Xで赤を引き,かつ2回目の試行Xでは青を引かなかったとき,
3回目の試行Xで白を引き,かつ4回目の試行Xで青を引く条件付き確率P(a,b,c)を求めよ。

(2)
試行Xをk回連続で行い,赤玉を引いた回数が青玉を引いた回数よりも多かったとき、
取り出した白玉の個数が,袋の中に残っている白玉の個数より多い条件付き確率Q(a,b,c)
を求めよ。

291290:02/02/05 03:38
問題まちがえてました。訂正します。

問題
袋の中に赤玉がA個、青玉がB個、白玉がC個入っている。(A,B,C≧1)
袋から玉を無作為に1個取り出して,取り出した玉の色を確認し、
再び玉を袋に戻す試行をXとする。

(1)
1回目の試行Xで赤を引き,かつ2回目の試行Xでは青を引かなかったとき,
3回目の試行Xで白を引き,かつ4回目の試行Xで青を引く条件付き確率P(a,b,c)を求めよ。

(2)
試行Xをk回連続で行い,赤玉を引いた回数が青玉を引いた回数よりも多かったとき、
白玉を引いた回数が,赤玉を引いた回数より少ない条件付き確率Q(a,b,c)
を求めよ。 (k≧2)

292132人目の素数さん:02/02/05 07:01
>>291
こういう非本質的なことは周囲の方にご相談下さい。
293 :02/02/05 11:22
Aの箱には赤玉2個、白玉3個が、Bの箱には赤玉3個、白玉3個が、Cの箱には赤玉4個、白玉3個が入っている。箱を1つ適当に選んで赤玉を1個取りだす確率を求めよ。
2+3+4/5+6+7=1/2
この考えはなぜ違うんでしょうか?
解答では
Aの箱を選ぶ時
1/3×2/5
というように場合わけしてました。
294ふえ〜〜:02/02/05 11:50
最近この問題出されて解けません。
もう問い一から難しいです。
http://www1.odn.ne.jp/drinkcat/topic/column/z_figpaz.html
どなたかお願いします。
295 :02/02/05 12:14
>>293
Aの箱には赤玉100個が、Bの箱には白玉1個が入っている。
箱を1つ適当に選んで赤玉を1個取りだす確率を求めよ。
296 :02/02/05 12:25
>>294
ブラクラ?
297132人目の素数さん:02/02/05 12:30
294 ブラクラじゃ無かったよ。
図形の問題。
298132人目の素数さん:02/02/05 12:32
>>294
私の脳みそでは無理です
(高3受験生)
299 :02/02/05 12:37
>>297
ほんとに?294=297だったりしない?
最近人間不信で・・・
300132人目の素数さん:02/02/05 12:42
疑わしいなら無視すれば?
質問に答える義務は誰にも無い。
301132人目の素数さん:02/02/05 12:45
そうそうそういう事
ちなみに分からんかった。
>298
受験はあきらめてください。
303132人目の素数さん:02/02/05 12:54
1+π/3-√3
(55-30√3)π/26
305293:02/02/05 14:01
でも箱に入ってる数が同数だったら僕のやりかたでも答えはあいますよね?
306132人目の素数さん:02/02/05 14:27
>>305
何の数?

玉全体?赤球?
307 ◆FHB7Ku.g :02/02/05 14:45
>>294=297?
最後の問いについて。求める式までは出せたけれど・・。
半径をr,4つの直角三角形の一番小さい角度をθとおく。
直角三角形の3辺は1,sinθ,cosθで、面積=(1/2)*1*cosθ*sinθ・・・ア
またこの直角三角形の面積は「(全体の正方形-真中の正方形)/4」」でも
あらわせる。
全体の正方形の面積=1,真中の正方形の面積=2r*2r=4r^2だから
直角三角形の面積=(1-4r^2)/4・・・イ
ア=イより,sinθ*cosθ=(1-4r^2)/2・・・・・・・ウ

また直角三角形の内接円の半径はr。内接円の公式(r=2s/(a+b+c))を使うと,
r=2*{(1-4r^2)/4}/(1+sinθ+cosθ)
よって
sinθ+cosθ=(1-4r^2-2r)/(2r)・・・・エ

エを二乗した式にウを代入してθを消去すると,
32r^4+16r^3-12r^2-4r+1=0

この4次方程式が解けない・・・。
308 ◆FHB7Ku.g :02/02/05 14:47
コミパルで見てるせいかもしれないけど、294のページ、異常に表示が遅い
気がします。なんでだろー??
309132人目の素数さん:02/02/05 14:53
(xy)^3=x+y で、陰関数yの極値を求めたいんですがやり方が分かりません。
どなたか教えていただけませんか?
>>307
32r^4+16r^3-12r^2-4r+1=(2r+1)(2r-1)(8r^2+4r-1)
311 ◆FHB7Ku.g :02/02/05 15:27
>>310
あ、それじゃr=1/2じゃない(図から判断)から
r=(-1+√3)/4
だと求められた。円の直径=(-1+√3)/2・・・答
立式でめんどくさくなってしまって計算放棄した自分はやばいかも・・。
計算ありがとうございました
>>311
実はθ=π/6というオチ
313二人の父親:02/02/05 16:01
>>294
問1から誰か説明してほしいな。
何回か考えたけど正三角形が何なんだ??
どうなるの?
314132人目の素数さん:02/02/05 17:28
ある問題の解答で、

4r^2(−1/2){cos120°−cos(2θ−120°)}=
2r^2(√3/2*sin2θ−1/2*cos2θ+1/2)

となる過程がわかりません。わかる方、ご教授お願いします・・。
>314
4r^2(−1/2){cos120°−cos(2θ−120°)}
=2r^2{cos(2θ-120°)-cos120°}
=2r^2{cos2θcos(-120°)-sin2θsin(-120°)-cos120°}
=2r^2(-1/2*cos2θ+√3/2*sin2θ+1/2)
=2r^2(√3/2*sin2θ−1/2*cos2θ+1/2)
316 ◆FHB7Ku.g :02/02/05 18:08
>>313
2人の父親なのになぜこんな問題を解く必要があるの?
円周率=πで解答します。

練習問題
求める面積をSとおくと
S=四分円の面積+四分円の面積-正方形
∴S=π/2−1・・・答

問1
正方形の中の領域は、9つに分けられるが、これは3つの図形の組み合わせ。
よってこの3つの面積をa,b,Sとする。(Sは求める面積部分)

練習問題で求めた部分は2b+xなので,
2b+x=π/2-1・・・ア
また2a+b=正方形-四分円=1-π/4・・・イ
図のa+2b+xの部分(四分円と四分円の重なっている部分)は、
(六分円)+(六分円)-(1辺が1の正三角形)に等しいから
a+2b+x=π/6+π/6-√3/4=π/3-√3/4・・・ウ
ア,イ,ウよりS=(π+3-3√3)/3・・・答

問2
一番左上の円の中心から正方形にそれぞれ垂線をおろすと一辺rの正方形ができる。
よって斜めの線=対角線。
したがって
r+(4r)/√2+(2r)/√2+r=1
∴r=(3√2-2)/8
∴S=5πr^2=(20-15√2)π/16・・・答
317ももんが:02/02/05 18:25
√2は、なんで1.414〜なの?
また、それはどうやって答えをだすの?
>>316
>問2
>r+(4r)/√2+(2r)/√2+r=1
はいいとして、
r=(3√2-2)/14
では?
319132人目の素数さん:02/02/05 18:46
>>317
もっとも基本的な考え方としては以下の通り。

x=√2 とすると x^2=2 となるはずだな。
x=1 としてみると x^2=1 で小さすぎる。
x=2 としてみると x^2=4 で大きすぎる。
x=1.5 としてみると x^2=2.25 でちょっと大きい。
x=1.4 としてみると x^2=1.96 でおしい!!
x=1.41 としてみると x^2=1.9881 でまだ足らない。
x=1.42 としてみると x^2=2.0164 でおおきすぎ。
x=1.415 としてみると x^2=2.002225 ですごくおしい。
x=1.414 としてみると x^2=1.999396 で...
以下略。
320a:02/02/05 18:48
集合X={1,2,3,4,5} XからXへ写像fで、a≠bに対しf(a)≠f(b)なる性質をもつような全体

1)f(1)=1 をみたすfは何とおり?
2)f(2)=2 をみたすfは何とおり?
3)すべてのaに対し、f(a)≠aをみたすfは何とおり?
>>317
平方根を開く(開平)筆算のやり方は
ttp://www5b.biglobe.ne.jp/~feathers/math_kaihei.htm
322a:02/02/05 18:56
↑訂正
集合X={1,2,3,4,5} XからXへ写像fで、a≠bに対しf(a)≠f(b)なる性質をもつような全体

1)f(1)=1 をみたすfは何とおり?
2)f(1)=1かつf(2)=2 をみたすfは何とおり?
3)すべてのaに対し、f(a)≠aをみたすfは何とおり?
323質問です:02/02/05 18:59
(1) lim_[x→+0]xlog(2x)
(2) lim_[x→+∞](logx)^4/x^2
(3) ∫(x^2)(e^3x)dx

一応(1)(2)はロピタルの定理、(3)は部分積分を使って答えを出したのですが、
どうも正しい気がしないので・・・。明日午後まで確認することができませんが
宜しくお願いいたします。初歩的な質問ですみません。
324 ◆FHB7Ku.g :02/02/05 19:02
>>393
問3
証明は出来ないけど,カンで,(笑)
正方形の対角線の長さは4r+r+r=6rになると思う。。いろいろ補助線ひくと・・。
だからr=(√2)/6,S=2*πr^2=π/9・・・答

オマケ問題
一番左上の円の中心をP、下の円の中心をQ、右の円の中心をRとすると△PQRは一辺2rの
正三角形。
またPから正方形の2辺にそれぞれ垂線をおろすことにより、PとQRの中点を結ぶ線は
対角線とわかる。
よって正方形の一辺の長さ=r+r+PQ*sin75=2r(1+sin75)
∴2r(1+sin75)=1⇔r=1/{2*(sin75+1)}
sin75=sin(30+45)=sin30cos45+cos30sin45=(√2+√6)/4より、
r=2/(√2+√6+4)=(-5√2+4√3-3√6+8)/2
円の直径=2r=-5√2+4√3-3√6+8・・・答

オマケ問題(改)は
>>307
>>311
で解きました。
>>323
1) 0, 2) 0 3) (x^2/3-2x/9+2/27)e^(3x)
>>294の問3
小さい円と大きい1/4円を一つづつ選び、その2円の接点をPとする。
Pでの大きい円の接線とPでの小さい円の接線は共通。この接線をlとする。
Pから大きい円の中心Oへ延ばした線mとlは直角をなす。
Pから小さい円の中心oへ延ばした線nとlは直角をなす。
よってoはm上にある。
Oから小さい円へ接線を引き、接点をQとすればOoQは直角三角形。
三平方の定理より
(1-r)^2=r^2+(√2/2)^2
これを解いて
r=1/4
327294のまとめ ◆FHB7Ku.g :02/02/05 19:23
>>294計算ミスしてたので,いちおうそれも直してまとめときます。
<練習問題 >
求める面積をSとおくと
S=四分円の面積+四分円の面積-正方形
∴S=π/2−1・・・答
<問1>
正方形の中の領域は、9つに分けられるが、これは3つの図形の組み合わせ。
よってこの3つの面積をa,b,Sとする。(Sは求める面積部分)
練習問題で求めた部分は2b+xなので,
2b+x=π/2-1・・・ア
また2a+b=正方形-四分円=1-π/4・・・イ
図のa+2b+xの部分(四分円と四分円の重なっている部分)は、
(六分円)+(六分円)-(1辺が1の正三角形)に等しいから
a+2b+x=π/6+π/6-√3/4=π/3-√3/4・・・ウ
ア,イ,ウよりS=(π+3-3√3)/3・・・答
<問2>
一番左上の円の中心から正方形にそれぞれ垂線をおろすと一辺rの正方形ができる。
よって斜めの線=対角線。
したがって
r+(4r)/√2+(2r)/√2+r=1
∴r=(3√2-2)/14
∴S=5πr^2=(55-30√2)π/98・・・答
<問3>
証明は出来ないけど,カンで,(笑) 正方形の対角線の長さは4r+r+r=6rになると思う。。いろいろ補助線ひくと・・。
だからr=(√2)/6,S=2*πr^2=π/9・・・答
<オマケ問題>
一番左上の円の中心をP、下の円の中心をQ、右の円の中心をRとすると△PQRは一辺2rの
正三角形。
またPから正方形の2辺にそれぞれ垂線をおろすことにより、PとQRの中点を結ぶ線は
対角線とわかる。
よって正方形の一辺の長さ=r+r+PQ*sin75=2r(1+sin75)
∴2r(1+sin75)=1⇔r=1/{2*(sin75+1)}
sin75=sin(30+45)=sin30cos45+cos30sin45=(√2+√6)/4より、
r=2/(√2+√6+4)=(-5√2+4√3-3√6+8)/2
円の直径=2r=-5√2+4√3-3√6+8・・・答
<オマケ問題(改)>
半径をr,4つの直角三角形の一番小さい角度をθとおく。
直角三角形の3辺は1,sinθ,cosθで、面積=(1/2)*1*cosθ*sinθ・・・ア
またこの直角三角形の面積は「(全体の正方形-真中の正方形)/4」」でも
あらわせる。
全体の正方形の面積=1,真中の正方形の面積=2r*2r=4r^2だから
直角三角形の面積=(1-4r^2)/4・・・イ
ア=イより,sinθ*cosθ=(1-4r^2)/2・・・・・・・ウ
また直角三角形の内接円の半径はr。内接円の公式(r=2s/(a+b+c))を使うと,
r=2*{(1-4r^2)/4}/(1+sinθ+cosθ)
よって
sinθ+cosθ=(1-4r^2-2r)/(2r)・・・・エ
エを二乗した式にウを代入してθを消去すると,
32r^4+16r^3-12r^2-4r+1=0⇔=(2r+1)(2r-1)(8r^2+4r-1)=0
図からr≠1/2でないと判断。
∴r=(-1+√3)/4 よって円の直径=(-1+√3)/2・・・答
>>324 問3について
小さい円の中心を O, 小さい円同志の接点を P
大きい4分円の中心(=四角形の隅)A, 4分円と小さい円の接点を B
とすると AOB は同一直線上にあるので OB=1-r
これに注意して 直角三角形 AOP について3平方の定理を使うと
r=1/4 となると思うんですが どうでしょう。
329326:02/02/05 19:25
>>326に書き忘れ

>Oから小さい円へ接線を引き、接点をQとすればOoQは直角三角形。



>三平方の定理より

の間に次の1行を入れます

2つの小さい円の接点Rは対称性から正方形の中心であり、QとRは一致。
330294のまとめ ◆FHB7Ku.g :02/02/05 19:28
問3は>>326の人ので、正解だと思うのでそっち参考にしてください・・。
にしても、、この板(スレ)って高校生でなく、大学生か大人の板(スレ)
の気がしてきた・・。
331328:02/02/05 19:29
訂正
× OB=1-r
◯ OA=1-r

かぶってすまそ
>>322
Xは1,2,3,4,5の順列全体と同一視できる。
1) 2,3,4,5の順列と同一視できる
2) 3,4,5の順列と同一視できる
3) 4*3*2*2!
<3の理由(間違ってるかもしれない。。)>
f(1)=a aの選び方は1以外の4とおり
1でもaでもない数字を一つ選びbとする
f(b)=c cの選び方はaとb以外の3とおり
1でもaでもbでもcでもない数字をdとする
f(d)=e eの選び方はaとcとd以外の2とおり
残りの2つについては、完全に自由に選べるので2!とおり
333332:02/02/05 19:46
あ、違う。
Xが1,2,3,4,5の順列全体と同一視できるのではなく、
「XからXへ写像fで、a≠bに対しf(a)≠f(b)なる性質をもつような全体」が
1,2,3,4,5の順列全体と同一視できる。
334323:02/02/05 19:52
ちょっと時間ができたので来ました。
2は合っていたのですが1と3は違う答えになってました。
(1は無限大、3は243分の〜というものすごい値に・・・)
本当に申し訳ありませんがどなたか過程を書いていただけないでしょうか?
今度こそくるのが明日になってしまいますがお願いいたします。
335332:02/02/05 19:52
発想的にはこんな感じ。
定義域 値域
1  →  a
a
b  →  c
c
d  →  e
>>334
1) lim_[x->+0] x log(2x)=-1/2 lim[y->∞]y/e^y
=-1/2 lim[y->∞]1/e^y=0
(ただしy=-log(2x), ロピタル1回使用)
3) 部分積分2回でもいいですが、原始関数を
(a x^2 +b x +c)e^(3x) と仮定して
この導関数が x^2 e^(3x) に等しくなるように
a,b,c を決めるという方法が早いです。
∫(x^2)e^(3x)dx= (∂/∂a)^2 ∫e^(a x)dx |_[a->3]
=(∂/∂a)^2(e^(a x)/a) |_[a->3]= …
以下略

なんてやり方もあるよん
338 ◆FHB7Ku.g :02/02/05 20:29
>>323
(1) lim_[x→+0]xlog(2x)
log(2x)=-tとおくとx→+0のときt→∞
よってlim_[x→+0]xlog(2x)=lim[t→∞]{(-1/2)*(t/e^t)}
ロピタルの定理よりt/e^t→0(t→∞)
∴lim_[x→+0]xlog(2x)=0・・・答

(2) lim_[x→+∞](logx)^4/x^2
ロピタルの定理を4回使うと,3/(2x^2)となるので
lim_[x→+∞](logx)^4/x^2=lim_[x→+∞]{3/(2x^2)}=0・・・答

(3)∫(x^2)(e^3x)dx=(x^2)*{e^(3x)}/3-(2/3)∫x*(e^3x)dx
=(x^2)*{e^(3x)}/3-(2/3)〔{x*e^(3x)}/3-{e^(3x)}/9〕+C
==(x^2)*{e^(3x)}/3-2*{x*e^(3x)}/9+2*{e^(3x)}/27+C・・・答

339322:02/02/05 20:34
>332
答えはどうなるのでしょうか?
340132人目の素数さん:02/02/05 20:55
入試直前でわからない問題があります。
ぜひ数学板の皆さんの協力が必要なのです。お願いします。
問題
複素数 z=cos72°+isin72°に対して、t=Z+1/Zとおくとき、次の問い
に答えよ。
tの値は テcos72°である。
ての部分を教えてください。お願いします!!
341 :02/02/05 20:56
>>340
わかったんじゃなかったのか?
342    :02/02/05 20:56
(問1)nが自然数の時、次の等式を数学的帰納法で証明せよ。
(n+1)(n+2)(n+3)・・・・・・(2n)=2のn乗・1・3・5・・・・・(2n−1)

(問2)(1+x)のn乗の展開式を利用して次の等式が成り立つことを証明せよ。
nC0−nC1+nC2−・・・・・・+(−1)のn乗・nCn=0

 ↑ ↑  ↑
これらは小さい0,1,2です。
教えて下さい。お願いします


2 だ
344 :02/02/05 20:58
>>340
つーか、普通に計算しても答え出るだろ?
345>>>340:02/02/05 21:06
馬鹿でごめんなさい 普通に計算してもわからないです
323の(1)って
x→0の時
x→0,log(2x)→-∞だから
2つをかけて
答えが0ってのはだめ?
>>346
たとえば x→+0の時
x→0,-1/x^2→-∞だからといって
2つをかけて
答えが0 ってだめですよね
>>347
あーそうですよね
ありがとうございます
>>340
z=cos72°+i*sin72°のとき
1/z=cos72°−i*sin72°だから
z+(1/z)=2*cos72°
350>:02/02/05 21:59
>>349
ありがとうございました!!!
351バカ文系:02/02/05 22:13
@217名の男子学生のBMIから計算した平均値はX=23であった。
母集団の分散は既知の値σ^2(二乗です)=6であるとして、
母集団の期待値μの信頼係数95%での信頼区間を求めよ。

AA:9,10,14,19,14 B:19,25,22,17,15
(1)AとBの2つのデータを合わせた分散を計算せよ
(2)2つの正規母集団平均の差の検定を有意水準1%で行なえ。
352質問です:02/02/05 22:42
http://jove.prohosting.com/~yosoike/cgi-bin/imgboard2/img-box/img20020205063841.jpg
上の画像は、頂点Aが20度の二等辺三角形である。
図中のXの角度は何度でしょう? 

こんなものは分度器で図れば答えは出ますが、
解法が知りたいので、誰か解いて下さい。
353132人目の素数さん:02/02/05 22:51
Turing機械についてわかる人いますか?
1変数入力について1を加える関数型とはどのようなものなのでせう?
なんともわかりません
354314です:02/02/05 23:06
>>315さん

どうもありがとうございました!加法定理ですね・・公式忘れてた・・
355質問です:02/02/05 23:20
1) f:A→B,g:A→B,h:B→Cに対してh・f=h・gが成立しているとする。
  このとき、 hが単射なら、f=g となる。
  は成立するか?成立すれば証明し、そうでなければ反例をあげよ。

2) f:A→BとしX,YをBの部分集合とする。
  f^(-1)(X−Y)=f^(-1)(X)−f^(‐1)(Y)
  は成立するか?成立するなら証明し、そうでなければ反例をあげよ。
356質問@文系数学:02/02/05 23:45
a<-1とする。
<1>x^2+ax≧-xをみたすxの範囲を求めよ
<2>xが<1>の範囲にある時、x^2+axの最小値mをaで表せ。
<3>m=-3/4となるaの値を求めよ。
スイマセンスイマセン。御教授下さいませ。今日の同志社大学神学部の問題です。
357質問@書き方:02/02/05 23:47
a∈A,b∈B,c∈C,f:A×B→C は c = f(a, b)
f^{-1}:C→A×B は (a, b) = f^{-1}(c) と書くのは正しい(一般的)ですか?
358質問です:02/02/05 23:50
関数が連続であるってことをいうには、何がどうなってればいいの?
教科書見たけど、難しすぎてわかんなかった
誰か簡単に教えてください
359 ◆GaussrLU :02/02/05 23:55
>>355
写像 f と g の合成を, f * g で表すことにする.

1) 成立する.
h * f = h * g より,
任意の a ∈ A に対して,
h * f (a) = h * g (a) がなりたつ.
よって, h ( f(a) ) = h ( g(a) ) であるが,
h : 単射より, f(a) = g(a) //

2) 成立する.
f^{-1} ( X-Y ) = { a ∈ A | f(a) ∈ X-Y }
であるから,
b ∈ f^{-1} ( X-Y )
=> f(b) ∈ X かつ f(b) は Y の元ではない.
=> b ∈ f^{-1} (X) かつ b は f^{-1} (Y) の元ではない.
=> b ∈ f^{-1} (X) - f^{-1} (Y).
よって, f^{-1}(X-Y) ⊂ f^{-1} (X) - f^{-1} (Y)
逆も同様にたどれる. //
36089454:02/02/05 23:55
    これマジ……!? とりあえず読んどけ……
                        ∧_∧
                  ∧_∧ (´∀` )
            ∧_∧ (´∀` )(    )
      ∧_∧ (´∀` )(    )| | |
 ∧_∧(´∀` )(    )| | | .(_(_)
 ̄ ̄ ̄ ̄ ̄ ̄ ̄ ̄ ̄ ̄ ̄ ̄ ̄ ̄ ̄ ̄ ̄ ̄ ̄
http://www.puchiwara.com/hacking/
361132人目の素数さん:02/02/06 00:01
>>355の解答を書こうと思って気になったんだけど、
「x は A の元でない」の記号ってどうやって出すの?
362 ◆GaussrLU :02/02/06 00:05
>>357
逆写像と逆像の違いはわかってますか?

>>358
y=f(x)
で考えると, x がちょっとだけ変わったときに,
y の値もちょっとしか変わらないよ,
ということを厳密に表現しているのです.
別の言い方をすれば, 関数のグラフが途切れ途切れになっていない,
ということを表したとも言える.
そう思って連続の定義をよく読んでみるとよい.
>>357
f:X→Yという写像があったとき、
A⊆Yに対してAの逆像f^(-1)(A)とは
f^(-1)(A)={ x ∈ X | f(x) ∈ A }
のことです。
364132人目の素数さん:02/02/06 00:26
>>362
勉強になります。
A×BとCが全単射の場合でもダメですか?
>364
問題ない。
366132人目の素数さん:02/02/06 00:54
>>365
すみません。問題ないというのは書き方((a, b)=f^{-1}(c))もですか?
>366
もちろん。

全単射でない場合は、A×Bの部分集合の集合でも持ってこればよい。
368132人目の素数さん:02/02/06 01:07
>>367
わかりました。ありがとうございます。
369355:02/02/06 01:25
>>359
有難う御座いました!
370132人目の素数さん:02/02/06 02:14
>>352
指針のみ示そう。
△BCD, △BCE の角度はすべてわかるから、
BC=1 とすれば、正弦定理を用いて、
CD,CEの長さが求まる。
次に、△CDEに対して余弦定理を用いれば
DEの長さが求まる。
ここで、△CDEに対して正弦定理を用いれば
DE/sin∠DCE = CE/sin(x) より
sin(x) が求まり、x=20°がわかる。
>>370
20°になるか?
372371:02/02/06 02:21
>>370
ごめん、20°になりそう。
373352:02/02/06 02:38
>370
ありがとうございます! なんかこれ、「ラングレーの問題」
とか言うらしいですね。そんな有名なものとは知りませんでした。

正弦・余弦定理なんて言葉は高校2年以来5年ぶりに聞きました!
文型に進んだので数学とは無縁の生活だったので・・・。

>>373=>>352
「ラングレーの問題」で検索したら初等幾何的解答が見つかった。
ttp://www.mitene.or.jp/~tomo-s/ruidai/r03.html
の解答2
375132人目の素数さん:02/02/06 04:07
>>356

(1)
x^2+ax≧-x より x{x+(a+1)}≧0、 -a-1>0 より x≦0, -a-1≦x

(2)
f(x) = x^2+ax = (x+a/2)^2-a^2/4、軸は x=-a/2
-a/2≧-a-1 を解くと a≧-2 だから
(i) -2≦a<-1 のとき、最小値 f(-a/2) = -a^2/4
(ii) x<-2 のとき、 f(-a-1) = a+1<0 = f(0) より
   最小値 f(-a-1) = a+1
(i)(ii)より
-2≦a<-1 のとき、m = -a^2/4
a<-2 のとき、m= a+1

(3)
-2≦a<-1 のとき、-a^2/4 = -3/4 より a = -√3
a<-2 のとき、a+1 = -3/4 より 解なし
よって、a = -√3
376 ◆FHB7Ku.g :02/02/06 04:22
>>356なんとなく375さんと結果が違くなったけど一応書いときます。
このごろ夜寝れなく,学校でねるというとんでもないリズム…。自律神経失調症気味…
<1>
x(x+a+1)≧0
-a-1>0(∵a<-1)であることからxの範囲は
x≦0,-a-1≦x・・・答

<2>
f(x)=x^2+ax=(x+a/2)^2-(a^2)/4とおく。
i)0<-a/2≦-a-1,すなわちa≦-2のとき
m=f(-a/2)=-(a^2)/4
A)-a-1≦-a/2,すなわち-2≦a<-1のとき
m=f(-a-1)=a+1
まとめて
a≦-2のときm=-(a^2)/4,-2≦a<-1のときm=a+1  ・・・答

<3>
a≦-2かつ-3/4=-(a^2)/4を満たす実数aは存在しない。
-2≦a<-1かつ-3/4=a+1を満たす実数aはa=-7/4
∴a=-7/4・・・答
377 ◆FHB7Ku.g :02/02/06 04:24
>>351
1.
分散=6だから標準偏差σ=√6
よって,信頼区間の公式から危険率5%で,
23-1.96*√6/√217<μ<23+1.96*√6/√217・・・答
(関数電卓で計算すると、22.674…<μ<23.325…)

2.
(1)A:9,10,14,19,14 B:19,25,22,17,15
平均値=(9+10+14+19+14+19+25+22+17+15)/10=16.4
よって分散をσ^2とおくと
σ^2={1/(10-1)}Σ[k=1,10](X(k)-16.4)^2
=(1/9)*{(9-16.4)^2+(10-16.4)^2+(14-16.4)^2+(19-16.4)^2+(14-16.4)^2
+(19-16.4)^2+(25-16.4)^2+(22-16.4)^2+(17-16.4)^2+(15-16.4)^2}

=(1/9)*(54.76+40.96+5.76+6.76+5.76+6.76+73.96+31.36+0.36+1.96)
=(1/9)*228.4
=1142/45
∴σ^2=1142/45・・・答

(2)チャート式発展の項目に2つの平均値の差の検定方法がのっているので、それを利用します。
(x1,x2がそれぞれ独立に平均μ1,μ2、標準偏差σ1,σ2の正規分布に従うとき,
変数(xバー1)-(xバー2)は平均μ1-μ2,標準偏差√{(σ1)^2/n1+(σ2)^2/n2}の正規分布に従う)
Aの平均μA=13.2 σA=(1/4)*(17.64+10.24+0.64+33.64+0.64)=15.7
Bの平均μB=19.6 σB=(1/4)*(0.36+29.16+5.76+6.76+21.16)=15.8
よって(xバーA)-(xバーB)は平均=μA-μB=-6.4,標準偏差=√99.226の正規曲線に従う。
よって危険率1%のとき信頼区間は
-2.35<{(xバーA)-(xバーB)-(-0.64)}/√99.226<2.35
∴-0.64-2.35*√99.226<(xバーA)-(xバーB)<-0.64+2.35*√99.226
関数電卓で計算すると、‐24.05<(xバーA)-(xバーB)<22.77・・・答
378どっちにかきこ?:02/02/06 04:36
波動関数Ψ(x)=(2πξ^2)^(-1/4)*exp[iax/h-x^2/4ξ^2]
(ξ,a,hは定数です。iは虚数単位です)
をフーリエ変換して Φ(p)と運動量空間に変換したいのです。
途中の関数がガウス関数とコサイン関数の積の積分になってしまって、挫折してしまいました。
どうかおねがいします。
379377の訂正 ◆FHB7Ku.g :02/02/06 04:37
>>351
2の(2)間違いました!!分散の√とるの忘れた・・。どうりで答が変だったわけだ・。
下のように訂正;;

(2)チャート式発展の項目に2つの平均値の差の検定方法がのっているので、それを利用します。
(x1,x2がそれぞれ独立に平均μ1,μ2、標準偏差σ1,σ2の正規分布に従うとき,
変数(xバー1)-(xバー2)は平均μ1-μ2,標準偏差√{(σ1)^2/n1+(σ2)^2/n2}の正規分布に従う)
Aの平均μA=13.2 σA^2=(1/4)*(17.64+10.24+0.64+33.64+0.64)=15.7
Bの平均μB=19.6 σB^2=(1/4)*(0.36+29.16+5.76+6.76+21.16)=15.8
よって(xバーA)-(xバーB)は平均=μA-μB=-6.4,標準偏差=√6.3の正規曲線に従う。
よって危険率1%のとき信頼区間は
-2.35<{(xバーA)-(xバーB)-(-0.64)}/√6.3<2.35
∴-0.64-2.35*√6.3<(xバーA)-(xバーB)<-0.64+2.35*√6.3
関数電卓で計算すると、-6.538<(xバーA)-(xバーB)<5.258・・・答
380351さんへ ◆FHB7Ku.g :02/02/06 04:51
1.標準偏差をS、分散をVとするとV=s^2、V={1/(n-1)}Σ[k=1,n](Xk-Xバー)^2

2.平均μ、標準偏差sの正規分布であるときZ=(x-μ)/s(正規分布曲線)

3.危険率5%ときたら0.95/2=0.475のところすなわちZ=1.96
  危険率1%ときたら0.99/2=0.495のところこの場合Z=2.35

統計の分野はこのくらいでいい気もします・・。(受験生さんなら)
捨ててる人も多いみたいだし、、。
>>376
-a/2<-a-1 のときは、x≦0, -a-1≦x に軸はないよ。
>>381
そーでしたね・・どーも最近ミスが。みなさんすいません。
神学部?は無理のようでした・・。最近長文読解のせいで数学が・。
英語ばっかりやると数学が落ちる・・。数学やり直すと、理社を忘れる・・
の繰り返し・・。数学を維持すんの難しい…。
383132人目の素数さん:02/02/06 05:17
>309
教科書の陰関数の存在定理を参考にしてもらえれば、問題ないと思うが。
∂f/∂x=0、f=0
の連立をとくことにより、陰関数の極値を与えるx、yの組の候補が得られる。
それが極大なのか極小なのかを判定するには、陰関数の2回微分を考えるのがベストだが面倒なので
教科書見てくれ
384おろかな高1@数学板のみなさま:02/02/06 05:22
a,bを有理数の定数とする.
次の2つの方程式が無理数の共通解を持つとき,a,bの値と共通解を求めよ.
x^3-3x^2+ax+5=0
x^3-4x^2+bx+10=0

三国、生きてたらこの問題おしえて。三国の解答みたけど理解できず!!!
2ch模試の問題ね。んじゃよろしく〜!
382は三国っしょ?英語まで極めないでね。頼むから。あといつものスレ来てね。
さびしいから。
385おろかな高1@数学板のみなさま:02/02/06 05:27
>384
三国の答はa=−3、b=−1、共通解1±√6
になっていた。よろしく〜
386tr@横レス:02/02/06 05:47
# 参考までに

>>384 さん
  { f(x) = x^3 - 3x^2 + ax + 5
  { g(x) = x^3 - 4x^2 + bx + 10
とする。

有理係数方程式 f(x)=0, g(x)=0 が
無理数の共通解 αをもつとき,
もうひとつ, 無理数の共通解 βをもつ。

ここで
  h(x) = f(x) - g(x) = x^2 + (a-b)x - 5
だから α, βは h(x) = 0 の 2解であるが
定数項に注目して
  { f(x) = h(x)(x - 1)
  { g(x) = h(x)(x - 2)
と因数分解できるから
  { 0 = f(1) = a + 3
  { 0 = g(2) = 2b + 2
  ⇒ a= -3, b= -1
このとき
  h(x) = x^2 - 2x - 5
  ⇒ 共通解 x = 1 ± √6
387おろかな高1@数学板のみなさま:02/02/06 06:12
>386
伝説のtrさん?
h(x)を求めるまではいたけど、定数項で因数分解できるのか。
さすがー。。分かりました。
解と係数の関係でやっていったけど解けなかったんですよ。。
すごい。
マジ、感謝です。
388おろかな高1@数学板のみなさま:02/02/06 06:19
伝説のtrさんのようになりたい!!でも数Iは赤点とった・・。
期末で、
x^2+a+b^2=0
x^2+bx+a^2=0
の共通解を求めよ。(a、bは定数)
というのがあったんですが自分は、
2つの式を引いて
(a-b)X=(a-b)(a+b)
よってX=a+b
と答でたのに、担任に×されました。なぜ??
>388
多分上の式はxが抜けてるのだと思うけど、
a=bの時
x^2+ax+a^2=0の2解が共通解・・・
390132人目の素数さん:02/02/06 06:31
∫[π,0]xsinx/(3+sin^2x)dx
が分かりません。助けて。
391tr:02/02/06 06:33
>>388 = 高1さん
  (a - b)x = (a - b)(a + b)
  ⇔ (a - b){x - (a + b)} = 0
  ⇔ a - b = 0 or x - (a + b) = 0
うっかり割るとイテテ。(>_<)

# それじゃ寝ます (_ _).zZ
392132人目の素数さん:02/02/06 06:35
>391
>うっかり割るとイテテ。(>_<)

おっさん この行が 激しく痛いよ・・・(;´д`)
>>392
氏ね
∬_[D]1/(a^2+x^2+y^2)^3/2dxdy D={(x,y);0≦x≦a,0≦y≦a}
∬_[D]√4x^2-y^2dxdy D={(x,y);0≦y≦x≦1}
∫[1,2]dx∫[x/1,2]ye^(xy)dy
∬_[D]lnx/y^2dxdy D={(x,y);1≦y≦x≦2}
1は極座標に変換していくようですがどれも挫折してしまいました。
395323:02/02/06 06:50
>>325 >>336-338 皆様ご協力有難うございました!
私が考えていなかった解法が多々あり自分のレベルの低さを痛感しました・・・。
今後また質問することがあると思いますのでその時はまたよろしくお願いいたします。
そして遅くなりまして本当に申し訳ありませんでした。
396線形が得意な方…お願いします:02/02/06 07:13
一次変換T:V→V
エルミート変換H:V→V (T*=T を満たす一次変換)
に対し、
Im(H)の直行補空間=Ker(T) である証明。



397132人目の素数さん:02/02/06 07:18
378助けてください (((( ;゚Д゚)))ガクガクブルブル
>>397
板違い
399378:02/02/06 07:29
フーリエ変換するだけです。
途中の積分で詰まってます
400132人目の素数さん:02/02/06 09:02
>>378
こんな計算もできない奴は今すぐ首つって死ぬか出家して僧になれ。
401132人目の素数さん:02/02/06 09:21

そう初歩的な計算です。普通の大学生ならできます。
400番さんは、どこがポイントだと思われます?どうやりました?
やり方だけ教えてください。
402132人目の素数さん:02/02/06 09:23
どうして板違いなのかわからんw
数学的な部分でつまずいてるのに、、ちなみに量子ね
403400:02/02/06 09:27
>>401
フーリエ変換の定義どおりに積分を書いてexpの中身をxについて平方完成しろ。
それでわからなければ今すぐ首(以下略)
404132人目の素数さん:02/02/06 09:28
兵法完成で 置換でいいのかな?
405396:02/02/06 09:29
僕も出家したほうがいいのかな…
方針がさっぱりわからない・・・
406132人目の素数さん:02/02/06 09:30
内せき 関係ないの?
カーネルやから なんか関係ありそう。
大体直交ホ空間に内席入ってるし。
407396:02/02/06 09:38
エルミート変換の定義と内積をどう使うが見えてこないのです。。。

固有空間の直和に分解して考えても導けるのだそうですが、
それは余計にわからない・・・・
408132人目の素数さん:02/02/06 09:50
いややっぱ 素直に直和分解で無理?
409132人目の素数さん:02/02/06 09:51
x^3-1
の因数分解ってどうやるんでしたっけ?
410396:02/02/06 10:06
>>408
直和分解を素直に使う…というの分かります?

dimV=dim Ker(T)+dim Im(T)
dimV=dim W+dim (Wの直行補空間)

でWを(Im(H))とおいて、
dim (Hの直行補空間)=Ker(H)

ちがうなぁ。。。わけわからんです。。
411396:02/02/06 10:09
>>109
x^3-1=(x-1)(x^2+x+1)
412411:02/02/06 10:10
×109
○409
>>390ってできますか?
414 :02/02/06 12:23
f(n)=(n-1)(n-2)(19-n)(3≦n≦18)
f(n)が最大のときnの値を求めよ。
微分以外のやりかたってないですか?
415age:02/02/06 12:42
すみませんDQSなのですが
11x^2−12xy+4y^2−4x+72=0を満たす 実数x,yは どうやれば・・・
おねがいします
>>414
x軸と1,2,19で交わる
右下がりのグラフなので、1〜2に極小値、2〜19に極大値があるのが分かる
あとは根性?
417バカ文系:02/02/06 13:36
>>377
どうもありがとうございます!!
これで統計学の単位が取れそうです!!
>>390 y=π-x と置換すると
∫[0,π]xsinx/(3+sin^2x)dx
=∫[0,π](π-y)siny/(3+sin^2y)dy となるので
=π/2∫[0,π]sinx/(3+sin^2x)dx
=π/2∫[0,π]sinx/(4-cos^2x)dx
=π/8∫[0,π]sinx{1/(2-cosx)+1/(2+cosx)}
=π/8 [log{(2-cosx)/(2+cosx)}]_[0,π]
=π log(3)/4
419413.390:02/02/06 14:26
>>418
凄い・・・
マジ感謝
420132人目の素数さん:02/02/06 14:59
>>409
x^3-1
=(x-1)(x^2+x+1)
421132人目の素数さん:02/02/06 15:10
友人に頼まれました。

関数f(x,y)=x^3+y^3-9xy+1

の極値を求めろ、という問題です。
友人の話では大して難しくはないが、なぜか解けない・・・とのことです。
私は数学がさっぱりなので問題の意味すら分かりませんがお願いします。
できれば過程も書いてほしいとのことです。
422ぼんもーる大国:02/02/06 15:16
>>415
1つの式で2変数の解は求まらない。
423 ◆FHB7Ku.g :02/02/06 15:36
>>415
11x^2−12xy+4y^2−4x+72=0⇔4(y-3x/2)^2+2(x-1)^2+70=0
よってこれを満たす実数x,yは存在しない。・・・答
424なっちゃん:02/02/06 15:41
(1,2,2)のまわりのπ/3の回転の行列はどうやってもとめるのですか?
>>424
回転軸を倒してz軸とかにあわせる→回転→軸をもどす。
てな感じでやってみれば?
426132人目の素数さん:02/02/06 16:25
396よ
次元定理は無理なの?そのVって有限空間?
そういう、基本的というか完全にそれぞれを理解してないと出来ない問題はむずい。
ましてや工学部の俺には無理だw 
427ほにゃ:02/02/06 16:38
角度(の平均など)を計算できる体(?)を教えて下さい。
例えば
(0°+ 90°)/2=45°
(0°+270°)/2=315°
です。
私の頭では2個の平均は計算できますが、3個以上になると???です。
というか、そもそも
(0°+180°)/2=
が90°にも270°にもなります。
根本的に考え方が間違っているのでしょうか?
>>427
角度は原則、反時計周りで数えるので、それで一度やってみて下さい。
429132人目の素数さん:02/02/06 16:55
大学に居た頃
「集合の理論で、すべての数学を説明できる」
と聞いたのですが、本当ですか?
430 ◆FHB7Ku.g :02/02/06 16:56
>>394
一部かっこがどこまでかかるかわからないけど下のように解釈してやってみました・・。
(2)
∬_[D]√(4x^2-y^2)dxdy D={(x,y);0≦y≦x≦1}
=∬_[D]√(4x^2-y^2)dxdy=∫[0,1]{∫[0,x]√(4x^2-y^2)dy}dx

∫[0,x]√(4x^2-y^2)dyについてy=2xsinθとおくと、
∫[0,x]√(4x^2-y^2)dy=∫[0,π/6]4x^2*cos^2θdθ=2x^2*∫[0,π/6](1+cos2θ)dθ={π/3+(√3)/2}*x^2

∴与式=∫[0,1]{π/3+(√3)/2}x^2dx=(2π+3√3)/18・・・答

(3)
∫[1,2]dx∫[x/1,2]ye^(xy)dy

∫[x/1,2]ye^(xy)dy=∫[x,2]ye^(xy)dy=e^x*∫[x,2]ye^ydy=e^x*{e^2+(1-x)e^x}
=e^2*e^x+(1-x)*e^(2x)

∴与式=∫[1,2]{e^2*e^x+(1-x)*e^(2x)}dx=(7e^4-e^2-4e^3)/4・・・答

(4)
∬_[D]ln(x/y^2)dxdy D={(x,y);1≦y≦x≦2}
=∫[1,2]{∫[y,2]ln(x/y^2)dx}dy

∫[y,2]ln(x/y^2)dx=∫[y,2]logxdx+{log(1/y^2)}(2-y)=(y-4)*logy+(2log2-2)

∴与式=∫[1,2]{(y-4)*logy+(2log2-2)}dy=∫[1,2]ylogydy+(-6log2+7/2)
次に∫[1,2]ylogydyを計算。
∫[1,2]ylogydy=∫[1,2]y'*(ylogy)dy=4log2-3/2-∫[1,2]ylogydy
よって∫[1,2]ylogydy=(1/2)*(4log2-3/2)

したがって,与式=(1/2)*(4log2-3/2)+(-6log2+7/2)=-4log2+11/4・・・答
>>429
「今時点では、集合の論理で数学の大部分が基礎付けられている」というだけです。
>>294
オマケ問題(改) やっぱり大学受験レベルは・・・???

直角三角形の直角をa、他をb,c (辺bcが正方形の1辺)の△abcとし、内接する円の中心をoとする。
更に、oと各辺(接線)を結び、その交点を辺ab上をe、辺bc上をf、辺ca上をgとする。
それぞれの角a,b,c、点e,f,gをoと結び、△abcを6つの三角形に分ける。

内接円の性質から、辺ao,bo,co のそれぞれ向かい合う三角形は合同・・・(1)
また、辺eo,fo,go(円の半径=r )は△abcそれぞれの辺に対し直角。

「△bfo+△cfoの面積」は、辺bcが長さ1だから (1*r)/2 、・・・(2)
また「△beo+△cgoの面積」も、(1)から (1*r)/2 。・・・(2’)
「△aeo+△agoの面積」は、中央にある正方形の1辺が2rであり、
高さ=半径r、更に直角であることから1辺rの正方形なので、面積は r^2 。・・・(3)

以上のことから、△abcの面積は、{(2)*2}+(3)= r^2+r 、
また中央の正方形は、「1辺= 2r 」だから面積は 4r^2 。

したがって、面積から関係式を作れば・・・・

4*(r^2+r)+4r^2=1*1

あとは解の公式を使えば r がでます。
(これ、中学レベルの問題ですね)
433421:02/02/06 17:29
すみませんが>>421お願いいたします・・・。
434132人目の素数さん:02/02/06 17:38
>432
もうがいしゅつ。>>327で解いてある。初等幾何のこんな問題蒸し返すな。

>>421
1階微分して極値侯補は (0,0)と(3,3)
2階微分を調べて (0,0)は鞍点, (3,3)が極小点になる。

うそかも
436132人目の素数さん:02/02/06 18:15
x^n−1を有理数多項式の範囲で因数分解すると、各既約多項式の係数は-1、0、1しか出てこないのかな?

例)x^12−1=(x^6―1)(x^6+1)
       =(x^2―1)(x^4+x^2+1)(x^2+1)(x^4―x^2+1)
       =(x+1)(x―1)(x^2−x+1)(x^2+x+1)(x^2+1)(x^4―x^2+1)

途中に出てくる多項式にも±1と0以外、出てこないな気がするのですが


全然関係無いけど、「多項式」って単項式や定数も含めるよね?
437429:02/02/06 18:18
>>431
ありがとう
438421:02/02/06 18:26
>>435
私も時間なので友人に伝えておきます。有難うございました。
>>436
n=3*5*7=105で初めて2が出てくるそうだ。
440436:02/02/06 18:31
>>439
ありがとうございます。反例があるのですね。証明しようとして時間を無駄にしなくてよかった。
しかし、105までは反例が無いとなると、一般的に成り立つと思っちゃうよね
441436:02/02/06 18:40
n<105では成り立つ、ていう中途半端な定理の証明なんてあるのかな?
もちろん、有限個だから全部虱潰しに確かめれ売いんだけど、そう言うんじゃないやつで。
>>440
前スレの831以降でガイシュツだが今見れないようだ。
↓n<105では係数に0と±1しか出ないことの証明。
ttp://www4.justnet.ne.jp/~mi_kana/story/cycrotomic.pdf
443132人目の素数さん:02/02/06 18:43
友人から頼まれた問題なのですが全くわからないので、お願いします。

f(x)は(2n+1)次の多項式で、しかも最高次の項の係数は1である。
このとき、「-1<k<1」なる任意の実数kに対して、
f(x)=ksinxを満たす実数xの個数は、高々(2n+1)個であることを証明せよ。
ただし、nは負でない任意の整数である。

よろしくです〜
444132人目の素数さん:02/02/06 19:00
In[1]:=
x^105-1//Factor
Out[1]=
(-1 + x)
(1 + x + x^2)
(1 + x + x^2 + x^3 + x^4)
(1 + x + x^2 + x^3 + x^4 + x^5 + x^6)
(1 - x + x^3 - x^4 + x^5 - x^7 + x^8)
(1 - x + x^3 - x^4 + x^6 - x^8 + x^9 - x^11 + x^12)
(1 - x + x^5 - x^6 + x^7 - x^8 + x^10 - x^11 + x^12
- x^13 + x^14 - x^16 + x^17 - x^18 + x^19 - x^23 + x^24)
(1 + x + x^2 - x^5 - x^6 - 2 x^7 - x^8 - x^9 + x^12 + x^13
+ x^14 + x^15 + x^16 + x^17 - x^20 - x^22 - x^24 - x^26
- x^28 + x^31 + x^32 + x^33 + x^34 + x^35 + x^36 - x^39
- x^40 - 2 x^41 - x^42 - x^43 + x^46 + x^47 + x^48))
445436:02/02/06 20:34
>>442
ありがとうございます。pdfって見えないのですが、どうすればいのでしょうか?
パソコン素人なんですみません。

>>444
ありがとうございます。―2となるのが一箇所だけなんですね。

例えばnをどんどん大きくした時に、0、±1以外の係数の個数や頻度の大体の値はnで表せるのだろうか?
素数の大体の個数を連続関数で表すように。
447132人目の素数さん :02/02/06 21:27
正規確率紙の書き方がわかりません。どなたか教えてくださいな
448宮廷娯楽:02/02/06 21:33
(a,b,c)a=(4,4,7),b=(-8,1,4),c=(1,-8,4)
この行列が回転を表すことを示し、その回転軸と回転角を求めよ。

この問題の意味すら分かりません。誰か分かりやすくお願いします。
449質問:02/02/06 21:41
t=tanαと置くとき次の式をtで表してください

@sin2α
Acos2α
450 ◆FHB7Ku.g :02/02/06 21:57
>>449
t=tanαのとき
(cosα,sinα)=(±1/√(a^2+1),±a/√(a^2+1)) (複号同順)

1.
sin2α=2sinαcosα=(2a)/(a^2+1)・・・答

2.
cos2α=cos^2α-sin^2α=(1-a^2)/(a^2+1)・・・答
451132人目の素数さん:02/02/06 22:08
>>414
f(n+1)/f(n)≧1 とか調べたら上手くいくんじゃないの?
ひとつ因子が消えるっしょ.
>>448
固有値が 9、 ±9I なんで、回転と相似のあわさったものになるんだけど。

3次元の1次変換が回転を表すとは、その変換の不動点からなる直線があって(これが回転軸)、
他の点はその点を通って回転軸と垂直な平面上を回転してる(?)ってな感じ。
行列を変形して
{{1, 0, 0}, {0, cosθ, -sinθ}, {0, sinθ, cosθ}}
ってなようになればいい。
このとき 1に対する固有空間(直線だけど)が軸、θが回転角。
453451:02/02/06 22:23
f(n+1)-f(n)≧0 の方がいいかも.スマソ.
454436:02/02/06 22:50
>>446
ありがとうございます。がんばってやってみます
455質問です:02/02/06 23:40
[2,1,-1]
[1,2,1]
[-1,1,2]

この3*3対称行列を対角化する直交行列を求めたいです。
途中の過程を書いてくれると助かります。お願いします。
456132人目の素数さん:02/02/06 23:50
>>455
今は大学の定期試験の時期らしいな。
教科書を見ろ、といいたいがたぶん教科書読めないだろう。
この問題については、まず、固有値を求める。
このとき重解をもつが、そっちのほうの固有ベクトルは直交し、長さ1にとる。
ここの操作で、固有空間を求めたうえで、そこでグラム・シュミットの直交化の方法を使うとよいだろう。
他方の固有値の固有空間の固有べクトルも長さ1にとっておく。
このベクトルたちを基底行列として選ぶと直交行列の対角化ができているはずだ。
確率の問題でわからないことがあります
1-8まで番号がついた8個の異なるボールが袋の中に入っています
1)1個ずつボールを取り出し8回取り出したとき
2)8個まとめてボールを取り出したとき
取り出したボールが1,2,3,4,5,6,7,8の番号すべてそろっている確率を求めよ
袋の中のボールは大量あるものとする

1)と2)で確率は違うのでしょうか?
計算式はどのようになるのでしょうか?
458132人目の素数さん:02/02/07 00:19
T=σ sqrt(ln N) で決まる値Tはどんな意味があるのでしょうか?
σ:分散値
N:情報の数
459132人目の素数さん:02/02/07 00:32
>>456
なんとか、やり遂げることができました。
ありがとうございます。

教科書読めない・・・仰るとおりでございます

460132人目の素数さん:02/02/07 00:37
>>457
同じだろ。
どの番号の玉も取り出す確率は1/8で (1/8)^8
461132人目の素数さん:02/02/07 00:39
>>457
袋の中には8個の大量のボールがあるんですね。

「豪華粗品贈呈」みたいなもんだな
462132人目の素数さん:02/02/07 00:42
dx/dt=εx+y
dy/dt=-y
で、ε=0のとき、危点は安定ってのはわかったけど
「●●安定」って答えないと点数をくれない・・・
漸近安定じゃないって言われたのだけど、それも何故だか
わからないです。
だれか助けてください。
463409:02/02/07 00:54
>>411
>>420
どもども。
他の公式から導き出そうと頑張ってみたのですが、
どうやらこれ自体を公式として覚えた方が良さそうで…
464457:02/02/07 00:55
8個→8種の間違いです
2)は重複組み合わせにならないのですか?
465132人目の素数さん:02/02/07 01:12
>>464
>2)は重複組み合わせにならないのですか?

確率と組み合わせの違いは分かってますか?
コインを2枚投げた時の出方は、重複組み合わせで3通り
A)両方表、B)両方裏、C)片方表片方裏
それぞれ1/3の確率。C)となったら私に2万円ください。A)、B)だったら1万5千円差し上げます
あなたの期待値=1万5千円*2/3―2万円*1/3=3333円、私の期待値=―3333円、あなたの方が得でしょ
466132人目の素数さん:02/02/07 01:18
>>460

8種の玉はどの順番でもいいので、
7!/8^7
ではないですか?
467457:02/02/07 01:36
>>465
理解しました。ありがとうございます
その勝負、一回きりなら受けて立ちます
468一日一禅。:02/02/07 01:57
確立の記号についてお伺いします。

! は、「階乗」と読むんですよね。
C は、「コンビネーション」と読むんですよね。

では、
P は、何と読むのでしょうか?

469tr:02/02/07 01:56
>>443 さん
-1 < k < 1, n : 0 以上の整数
f[n](x) : (2n+1)次の多項式で, 最高次の係数は 1
g[n](x) = f[n](x) - k*sin(x) (注 : [n] は添え字のつもり)
-----
g[n](x) = 0 の実解が, 高々(2n+1)個であることを示せばよい。

i) n = 0 のとき
  f[0](x) = x + a
と表せるから
  g[0](x) = (x + a) - k*sin(x)
  ⇒ g'[0](x) = 1 - k*cos(x) > 0 (∵ -1 < k < 1)
となり g[0](x) は単調増加。
  ∴ g[0](x) = 0 の実解は, 高々 1個

ii) n = j に対し
  g[j](x) = 0 の実解は, 高々(2j+1)個
と仮定する。 このとき
  g[j+1](x) = f[j+1](x) - k*sin(x)
  ⇒ g''[j+1](x) = f''[j+1](x) + k*sin(x)
であり, g''[j+1](x) について
  次数 : 2j+1, 最高次の係数 : (2j+3)(2j+2) > 1
だから, 仮定が適用できて
  g''[j+1](x)/{(2j+3)(2j+2)} = 0 の実解は, 高々(2j+1)個
  ⇔ g''[j+1](x) = 0 の実解は, 高々(2j+1)個
  ⇔ g[j+1](x) の変曲点は, 高々(2j+1)個
  ∴ g[j+1](x) = 0 の実解は, 高々(2j+3)個

以上, i) ii) より題意は示された。
470132人目の素数さん:02/02/07 02:00
>>468
Permutation
471132人目の素数さん:02/02/07 02:04
>>467
1回と言わず大数の法則が実感できるまで何回もやりましょう。

これって結構だまされる人いないかな?ちょっと見分からなくて、わずかにこちらの期待値が大きくて長くやってると、得するってやつ。
472一日一禅。:02/02/07 02:10
 470さん、ありがとうございます。

 「順列、置換」の意ですね。
473132人目の素数さん:02/02/07 02:20
ううう
>>448ではないんですが
>>448の問題>>452の解説をみてもわかりません、、、
474473:02/02/07 02:52
>>448
なんとか自分なりにわかりました。
スレ汚しスマソ
475457:02/02/07 03:08
最後にもう一つ助けてください
>>457で、n個を取り出したときに8種類そろっている確率が求められません
よろしくお願いします

>>471
いいえ、遠慮しておきます
100円くらいならだまされてのってくる人いるかもしれませんね
476132人目の素数さん:02/02/07 03:11
1の分割とか何ですか?
よろしくお願いします。
477132人目の素数さん:02/02/07 03:48
円の面積の公式 \pi r^2 を三角関数の積分を本質的に使わないで
導き出す方法って無い?無いならそれを示して。
478132人目の素数さん:02/02/07 04:59
xy+xz+xu+yz+yu+zuの2次形式を変数の直交変換により標準化してください。
ついでに直交変換ってなんすか??
479高校生です:02/02/07 10:09
くだらない問題はここに書けって叱られたので泣く泣くここにきました。でも、答えて
くれるのでしょうか。
雪印マークの問題です。雪印のマークは正六角形に正五角形(星型)が内側から、
ぴったり接触しているように見えるのですが、厳密にそのようなことは可能なので
しょうか。
つまり、正六角形の辺上に5つの点があります。そのうちの1つは正六角形の角に
あります。あとの4点を動かして5点が正五角形の配置になるようにできるのでし
ょうか。
>>499

正六角形に内接する円も、正五角形に外接する円も、
円なので一通りしか考えられず、頂点の場所を考えるとぴったりということはないと思われます。
481132人目の素数さん:02/02/07 12:28
なぜ正六角形の内接円が関係するのかわからんのう
ひとつは正六角形の頂点なんでしょ?
482132人目の素数さん:02/02/07 12:44
a*(b+c*x)dy/dt=d*(e-y)-(f-c*y)*z
dx/dt=-1/a*z
z=g*(w(y)-i)

定数(a,b,c,d,e,f,g,i)
未知数(x,y,z)
w(y)はyによって決まる定数.

この連立微分方程式はプログラムか何かで
数値的に計算するしかないのですか?
>>481
かんちがいしていました。
484高校生です:02/02/07 13:52
>>483
勘違いですね。
485132人目の素数さん:02/02/07 14:11
>>478
問題あってる?
486TASHIROFAN:02/02/07 14:16
http://cheese.2ch.net/test/read.cgi/math/1012983458/
から飛んできました。
14>>
>頂点の一つが重なって、正五角形の残りの頂点が
>正六角形の辺のどこかに接する場合ならありそうです。

これには気づきませんでした。勉強になりました!!
ありがとうございます。
>>486
実際には、正六角形の中に三点で接することが出来る正五角形に外接する円の
最大の場合を検討して、作図などをしてみるとよいようですので、頑張ってみてください。
488 :02/02/07 14:38
複素数z(1)=r(cosθ+i sinθ)をとる。ただし、r>0、0<=θ<360、iは虚数単位とする。また、複素数z(2)は│z(2)│=2│z(1)│を満たすもののうちで、z(1)との複素数平面上の距離が最大になるようにとる。
θが0、180でないとする。実数a,b,cに対してz(1),z(2)がxに関する4次方程式の解である時a,b,cをcosθを用いて表せ

手も足も出ません。お願いします。
489 :02/02/07 14:40
>>488
a,b,c?
490288:02/02/07 14:50
実数a,b,cに対してz(1),z(2)がxに関する4次方程式x^4+ax^3+bx^2+cx+4の解である時a,b,cをcosθを用いて表せ

スマソ
491288:02/02/07 14:51
x^4+ax^3+bx^2+cx+4=0です。

スマソスマソ

お願いします。
492132人目の素数さん:02/02/07 15:19
>488
x^4+ax^3+bx^2+cx+4=(x-z1)(x-(z1)~)(x-z2)(x-(z2)~) 右辺を展開
(z1)~=r(cosθ-isinθ)
(z2)~=-2r(cosθ-isinθ)
493132人目の素数さん:02/02/07 15:39
素因数分解したい数nについて、次のような数列を考える。
x0=2
xi+1≡xi2+1(mod n) (i≧0)
ある数をnで割った余りは、
0、1、2、……、n−1
のn通りしかあり得ない。
よって、上の計算をずっと続けていくと、いつかは、
xi≡xj(mod n)
となるようなi、jが見つかるはずである。
この時、
g=gcd(n,xj−xi)
を計算してやると、1<g<nとなり、nの素因数が見つかる可能性がある。
494493:02/02/07 15:42
>>493
x(i+1)≡x(i)^2+1(mod n) (i≧0)
のまちがいです。。

このとき
x(i)-x(j)はnの倍数だから
g=nになりませんか??
495ほにゃ:02/02/07 16:01
申し訳ございませんが>>427の面倒も見てやってほしいのですが。

427を違う例で申し上げますと、
点A、B、C、Dがあるときに相互の距離が
  A B C D
A 0 1 2 1
B 1 0 1 2
C 2 1 0 1
D 1 2 1 0
となる座標系を教えていただけたらありがたいです。
496転載:02/02/07 16:01
166 名前:132人目の素数さん :02/02/07 15:52
ブール代数では1+1=1っていうのはホントですか?

167 名前:132人目の素数さん :02/02/07 15:58
>>166
1+1=1 1+0=1 0+1=1 0+0=0
1−1=1 1−0=1 0−1=0 0−0=1
1*1=1 1*0=1 0*1=1 0*0=0
1÷1=0 1÷0=1 0÷1=1 0÷0=0
だよ。
497中二:02/02/07 16:09
学校の先生に回答なしでこの問題を考えてこいって言われたのですが
分からないので教えてください。

平面上に円Oがあり円Oの外に二点A、Bがある。今円O上に
点Pを取りAP+PBが最小になるようにしたい。点Pはどのように
とればいいか。
>>497
直線ABに垂直な直線と、円との交点じゃない?
499498:02/02/07 16:18
んで、直線ABとの距離が最小になる点。
500 :02/02/07 16:18
>>497
考えるトレーニングとして出したんだろうから
自分で考えたほうが後々自分のためになると思うよ。
ABの中点では?
502132人目の素数さん :02/02/07 16:41
>>501
ABの中点って円外の点だろ
503スイマセン:02/02/07 16:53
>>502
円との交点を通る直線が通る点がです。
504 :02/02/07 16:57
>>498 >>501
そんな気もするが実は違うよ(ABの位置によって)。
  ○
━━━・━━・━━・━━━
     A   N   B

線分ABの垂直ニ等分線が円と共有点を持つとは限らない
506132人目の素数さん:02/02/07 17:00
「有界」を工学部のオレにも分かるようにそれほど厳密でなくていいから
教えて下さい。数学書だとはっきり分からない
A,Bの位置関係による
508お願いしまっす:02/02/07 17:07
10人の選挙人が3人の候補者の1人に投票する。
次ぎの各場合に何通りの票の分かれ方があるか?
(1)記名投票の場合
(2)無記名投票の場合
509質問です:02/02/07 17:10
1.6人を2つの部屋A,Bに入れる方法は何通りあるか?
但し、全部の人が同じ部屋に入ってもよいとする。

2.6人を2つのグループに分ける方法は何通りあるか?
但し各部屋には少なくとも1人は入るものとする。

3.6人を2つのグループに分ける方法は何通りあるか?
510 :02/02/07 17:12
ここは宿題をやってあげるスレですか?
>>508
(1)記名投票の場合 10人一人一人に3通りの選択肢がある。樹形図を書けばすぐわかる

(2)無記名投票の場合  重複組み合わせ

無効票、白紙はなしだよね?
512132人目の素数さん:02/02/07 17:14
>ここは宿題をやってあげるスレですか?

ヒントだけの方が良いかな?
513132人目の素数さん:02/02/07 17:30
>>507>>506への答えですか?
>>512
いい問題であれば、子供なりにいい解答を作り直そうと思うのでは???
>>513
そんな訳はないよ。。。
X<=1というのと、Xの取り得る最小値が1というのとは同じ事だけど、
X<1というのと、Xは下への有界というのと大体同じとか、そういうのでなかったけ。
516Gallop:02/02/07 17:41
質問ッス。
 A君は1000円札をx枚,100円硬貨をy枚,10円硬貨をz枚もってお店に行った。
 ある品物を1個買うと,1000円札がy枚,100円硬貨がz枚,10円硬貨がx枚になった。
 さらに同じ物を買うと,1000円札がz枚,100円硬貨がx枚,10円硬貨がy枚になった。
 ただし,所持金は1000円以上10000円以内で,x,y,zは自然数である。
このとき,品物1個の値段を求めよ。
この問題全然わからんので、誰か解いてください。
>>497 円柱状の鏡に光を反射させることを考えれば
円の中心をO として
(OPとPAのなす角)=(OPとPBのなす角)
となるような点 が求める P ではないかと思うんですがどうでしょう。
518質問です:02/02/07 17:51
TEXで分からないことがあります。
= の下に ← がある記号を出したいのですが、
この記号はTEXで用意されていますか?
探しても見つからないので、今は
\underset{\leftarrow}{=}
で代用しています。かなり不恰好ですが。

TEX関連のスレが見つからなかったので、
スレ違いかもしれませんがここに書きこませていただきました。
分かる方いらっしゃったら教えてください。
n=1000x+100y+10z
n-m=1000y+100z+10x
n-2m=1000z+100x+10y

ただし1000<n<10000だろ?
1110(x+y+z)=3n-3m=3(n-m)だから、

なんだ、x=y=z=3で、品物は0円で成り立っちゃうじゃん。
品物の値段の条件が足りてないんじゃない?
520Gallop:02/02/07 17:57
>>519品物の値段は1000円以上です。これだけしか書いてません・・・。
521132人目の素数さん:02/02/07 18:06
次の数量の関係を、等式または不等式を書いてあらわせ。

A、Bの2人がaメートル競争した。Aが決勝点に着いた時、Bはbメートル後ろにいた。
今、2人が同時に決勝点に着くように、Aは出発点のxメートル後ろからスタートした。

解説キボンヌ。
522 :02/02/07 18:08
>>521
a>b
523 :02/02/07 18:10
a>0
Aの速度:Bの速度=a:a-b

次に走る距離は、
Aの距離:Bの距離=a+x:a

同時に到着だから、時間は同じだよね。

時間=距離/速度

a/(a+x)=(a-b)/b  これを整理せよ。

525132人目の素数さん:02/02/07 18:31
>>469
443のやつ
「仮定が適用できて、g''[j+1](x)/{(2j+3)(2j+2)}=0
の実解は、高々(2j+1)個」
とあるが、ここで仮定とは
「(2j+1の式)-k*sin(x)=0の解が高々(2j+1)個」
であるのに対し、対象は
「(2j+1の式)+k*sin(x)/(2j+3)(2j+2)=0」
であるので、仮定を適用する事は不可。
Y=2sin^2x+sin2x  (0゜≦x≦90゜)
(二倍sin二乗xプラスsin二倍x)

をxについて解けという問題なのですが、
解法を教えてください。
解き方の流れや解だけでも結構です。
よろしくお願いします。
>>526
Y=2sin^2x+sin2x=1-cos2x+sin2x
=1+√2 sin(2x-π/4)
∴ x=π/8 +1/2 Arcsin((Y-1)/√2)
カナーリ強引な解き方

n=1000x+100y+10z
n-m=1000y+100z+10x
n-2m=1000z+100x+10y

これから、nとmを消すと、4x+3z=7y という式が出てくる。
これと、x,y,zは1以上9以下の整数という条件から、解を求める。

x=y=z=1〜9という解は、m=0となってしまふので除外。
また、4,3,7が互いに素である事を考えれば、簡単に解ける。
そうすると、元の代金は、9620円か8510円であった事が知られ、品物代金は3330円である。
なぜ数学板のデフォルトは132人目の素数さんなのですか?
530132人目の素数さん:02/02/07 19:01
>>529

>>1を見ろ
531526:02/02/07 19:05
>>527
早速ありがとうございます。
なんて言います工房なので、
x=π/8 +1/2 Arcsin((Y-1)/√2)
がよくわからないのですが。
特にArcが・・・。
数VCの範囲でお願いしたいのですが。
よろしくお願いします。
もしくは、もうすでに数VCの範囲なのでしたら
もう少し詳しくお願いします。


532521:02/02/07 19:06
>>522-524
違うらしいです。
答えは
a分のa-b=a+x分のaです。

解説キボンヌ。中学1年の問題です。
>>531
Arcsin ってのは sin の逆関数です。
高校の範囲では習わないですよね。
Y の値が具体的に与えられてるんじゃないでしょうか。
そうでないなら 一般のYに対して x を表現するには
arcsin が必要になってきます。
534お願いします。:02/02/07 19:11
θ=π/7として
2002
Σ(-1)^k×coskθの値を求めよ
k=1
やっちまった。
ダメ板のダメさをこれ以上なく解析してしまった。(鬱死。
536526:02/02/07 19:17
>>533、527
そうなんですか・・・。
では、僕の出題に間違いがあるのかもしれません。
入試問題で出て、解けなかったので質問させてもらいました。
問題が手元になく、はっきりとした問題を出せなくてすいませんでした。
それでもわざわざ答えていただき、ありがとうございました。
Σ(-1)^k×coskθ= Re Σ(-1)^k exp(ikθ)
=Re (-exp(iθ))(1-exp(i2002 θ))/(1+exp(iθ))
=0
(∵ 2002 θ= 286 π)
538524:02/02/07 19:26
ごめん間違ってた。
Aの速度:Bの速度=a:a-b
Aの距離:Bの距離=a+x:a

から導かれるのは、a/(a+x)=(a-b)/a だね。
>>477
半径rの円を、相当数のn等分割した扇形に切り、円弧が上下交互になるように並べ変える。
出来た図形は、上下の辺が波打ち、僅かに傾いているが、
1辺が円周、高さがrの長方形に近似する。
しかし、分割数を限りなく増やすことによって、この波打ちと傾きは解消されると考える。
従って、円の面積は、変形した長方形の面積={(円周)*r}として計算可能と考える。

これ有名。
540539:02/02/07 20:16
失礼。(円周)*r →(円周/2)*r
541132人目の素数さん:02/02/07 20:37
付かぬことを伺います。

f(x),g(x)が有理数係数の多項式で
f(x)*g(x)が整数係数多項式ならば、適当な有理数a,bを持ってきて
af(x)*bg(x)=f(x)*g(x)  がつ af(x),bg(x)の係数は整数
[↑これからab=1がわかる]
つまり、

f(x),g(x)∈Q[x] ∧ f(x)*g(x)∈Z[x]
        ⇒
∃a,b∈Q:af(x)*bg(x)=f(x)*g(x)  ∧ af(x),bg(x)∈Z[x]

は成り立つのでしょうか?


542お願いします:02/02/07 21:16
正四面体OABCにおいて、辺OAの中点をP、辺OB、OCを各々2:1に内分する点をそれぞれQ、Rとする。
また、辺AB、BCの中点をそれぞれS、T、辺OCを1:2に内分する点をUとする。正四面体の1辺の長さを1、vector OA=vector a、vector OB=vector b、vector OC=vector cとして、次の問に答えよ。
(1)平面PQR上の点をXとするとき、
vector OX=α/2 vector a+2β/3 vector b+2γ/3 vector c、α+β+γ=1
と表されることを示せ
(2)平面PQRと平面STUの交線をlとする。lの方向ベクトルを求めよ
(3)直線lの、正四面体OABCの内部に含まれる部分の線分の長さを求めよ。
543 :02/02/07 21:24
ほんと宿題ばかりだな
544541:02/02/07 21:25
私のは宿題じゃないので。念のため
545質問です:02/02/07 22:04
>>511
>(1)記名投票の場合 10人一人一人に3通りの選択肢がある。樹形図を書けばすぐわかる

>(2)無記名投票の場合  重複組み合わせ

>無効票、白紙はなしだよね?

っていうことは(1)は3の10乗ですか?
(2)もうちょっと詳しく教えて下さい!!全く分かりません。
2個重複、3個重複とかって全部考えてくの?
546 :02/02/07 22:09
>>545
(2)は全部書き出してみろ。ノート1ページにもならんだろ。
547541:02/02/07 22:28
重複組み合わせの公式は知らないですか?俺も知らない。というかいちいち覚えていない。
ここで考えよう。

組み合わせというのは、
ABCDEFGの7枚のカードがあって、そこから4枚取る取り方、とかでABCD=CADB、つまり順序は度外視する。

7種類のカードが沢山あって、そこから4枚取る取り方は何通りか?AAAAとかAABBとかも入ってくる。
これを場合わけするのは大変。

Aの枚数をa、Bの枚数をb、などと表す。
a+b+c+d+e+f+g=4 を満たす、非負整数a,b,,,の組み合わせの個数を求めればよい

         Q
   ■■■■■■
   ■■■■■■
   ■■■■■■
   ■■■■■■
  P

こういう路地(一番外側も路があると思って)をとおってPからQに行く場合の行き方と同じになる
縦の7本の路地にABCDEFGを対応させる。
で、pからQへの道順を一つ決めれば、a+b+c+d+e+f+g=4 を満たす、非負整数a,b,,,の組み合わせが一つ決まる

なぜかをこれで説明するのは難しい。黒板ならすぐだが。あとは自分で考えて


だれか、>>541をお願いします


548541:02/02/07 22:30
Qの位置がずれちゃった。P,Qは対角線上にあります
549お願いします :02/02/07 22:33
十分に長い長ぁ〜い階段がある。
サイコロを振って、出た目の数だけ登っていく。
このとき、超!!上の方の段に立ち止まる確率を求む。
550132人目の素数さん:02/02/07 22:36
150×log(283/373)
の答えが-41.4らしいんですがどうやって解けばいいんですか?

ちなみに大学生の問題です。高校ではないです
近似値とかやったんですけど全然わかんないです・・・・
551132人目の素数さん:02/02/07 22:41
>>550
それ、底がeだね。logじゃなく普通lnって書く。
552132人目の素数さん:02/02/07 22:44
「博士号」とかけて「足の裏の米粒」と解く。
その心は「とらないと気持ち悪いが、とっても食えない」。
553550:02/02/07 22:45
そうです
底はeです 150×loge (283/373)
です
554 :02/02/07 22:47
高校入試の最難レベルの問題とか解けなくては数学者になれない?
555132人目の素数さん:02/02/07 22:49
なれます。全然関係ありません。
556132人目の素数さん:02/02/07 22:51
あるサイトにこのような文章が書いてありました。

素因数分解したい数nについて、次のような数列x(i)を考える。
x(0)=2
x(i+1)≡x(i)^2+1(mod n) (i≧0)
ある数をnで割った余りは、
0、1、2、……、n−1
のn通りしかあり得ない。
よって、上の計算をずっと続けていくと、いつかは、
x(i)≡x(j)(mod n)
となるようなi、jが見つかるはずである。
この時、
g=gcd(n,x(j)−x(i))
を計算してやると、1<g<nとなり、nの素因数が見つかる可能性がある。

そこで質問です。
x(j)−x(i)はnの倍数だからg=nになると思うのですが。。。
この文章は間違いですか??
>>550
logの数表か電卓を使えばいいのでは。
558132人目の素数さん:02/02/07 23:04
間違いですね。g=nです。ところで
>x(i)≡x(j)(mod n)
>となるようなi、jが見つかるはずである
これは本当ですかね。ちょっと分かりません。
559550:02/02/07 23:05
>>557
普通に計算は無理ですか??
560132人目の素数さん:02/02/07 23:16
∫[0,∞]x^p/(x^2+1)dx (-1<p<1)
がわからないので回答お願いします。
大学生の問題です。
>>560 -∞から∞までの積分にして 留数をもとめれば (・∀・) イイ!
S= ∫[0,∞]x^p/(x^2+1)dx とおくと
∫[-∞,∞]x^p/(x^2+1)dx = ∫[-∞,0]x^p/(x^2+1)dx+∫[0,∞]x^p/(x^2+1)dx = (exp(iπp)+1)S …[i]
(∵1項目で -x→x と置換)
一方 左辺は x=i における留数から求められて 2π i x^(p-1)/2 |_[x=i] = π exp(iπp/2) … [ii]

[i],[ii]を等置して S = π/(2 cos (πp/2))
562541:02/02/08 00:27
>>545

ごめん。3人しか候補者がいないなら重複組み合わせを持ち出さなくてもいいや。
>>546みたく書き出しても手間は掛からない


あ 全員が一人に投票する場合
い 2人に投票する場合
う 3人に投票する場合

で考える。あ)は簡単。い)も少し考えればわかる。

う)も簡単だけどちょっと説明


●●●●●●●●●● これを3人でわけるわけだけど

●●●●● ●●● ●● これでAが5票、Bが3票、Cが2票と考える。
これがヒント。



>>550
log(1-x) = -(x+x^2/2+x^3/3+x^4/4+…)
て展開して x=90/373 てやるのはどうでしょう。 4乗までで3桁の精度でます
564560:02/02/08 00:44
>>561
Thanks!!
留数のあたり勉強し直します。
565 ◆pvySbQO2 :02/02/08 01:00
>>541
成り立ちます。

fが有理数係数多項式なのでf(x)=r・p(x)で
rは有理数,pは整数係数多項式で係数の最大公約数は1と
表せgもg(x)=s・q(x)と表せる。
そうするとp(x)q(x)も整数係数多項式で係数の最大公約数は1と
なってrs・p(x)q(x)が整数係数多項式になることから
rsが整数になることがわかるので
s・f(x)=rs・p(x),(1/s)g(x)=q(x)は整数係数多項式になる。
566550:02/02/08 01:27
>>563
それってマクローリン展開ですか?
x=90/373はどこからでてくるんですか?

567550:02/02/08 01:29
>>563
ああ1-x=283/373
を解いたんですね
すいません
568 ◆FHB7Ku.g :02/02/08 01:47
>>509
1. 部屋はA,Bと名がついていているので部屋は区別があることに気をつける。
2^6=64・・・答

2.
問題文を「6人を2つのグループに分ける方法は何通りあるか?但し各グループには少なくとも1人は入るものとする。」
とします。グループに区別はないので,
6C1+6C2+6C3=41・・・答

3.
6C0+6C1+6C2+6C3=42・・・答
569 ◆FHB7Ku.g :02/02/08 01:57
>>568ちょっと訂正
1.部屋はA,Bと区別できるので,2^6=64・・・答

2.部屋を区別してカウントするなら(6C1+6C2+6C3)*2=82・・・答
  部屋を区別しないのなら(6C1+6C2+6C3)=41・・・答

3.グループに少なくとも1人はいなくてはならない場合は,6C1+6C2+6C3=41・・・答
  0人というグループを許容してもいいのなら6C0+6C1+6C2+6C3=42・・・答
570tr:02/02/08 02:30
>>525 さん
略したのがマズかったようですね。(汗)

@469>  ⇒ g''[j+1](x) = f''[j+1](x) + k*sin(x)
@469>であり, f''[j+1](x) について
@469>  次数 : 2j+1, 最高次の係数 : (2j+3)(2j+2) > 1
@469>『だから, 仮定が適用できて』
@469>  g''[j+1](x)/{(2j+3)(2j+2)} = 0 の実解は, 高々(2j+1)個
の意味を, 詳しく書きなおします。(特に 『 』 の部分)

仮定は正確には
  -1 < k < 1 に対し,
  f[j](x) - k*sin(x) = 0 の実解は, 高々(2j+1)個
です。 また帰納法の途中にあらわれる
  0 = g''[j+1](x)/{(2j+3)(2j+2)}
   = {f''[j+1](x) + k*sin(x)}/{(2j+3)(2j+2)}
において, 改めて
  f''[j+1](x)/{(2j+3)(2j+2)} → f[j](x)
  k/{(2j+3)(2j+2)} → -k
とおくことにより
  -1 < k < 1, f[j](x) - k*sin(x) = 0
となり, 仮定が適用できます。
571 ◆FHB7Ku.g :02/02/08 02:35
>>508自信ないけど一応。
(1)
候補者の名前をA,B,Cとし、選挙人の名前を1,2,3,4,…,10となづける。
投票用紙には「3,B」(選挙人3がBを選んだ)というようになる。
この投票用紙は10枚。
したがって,この投票用紙「☆,○」が何通り生じうるか,を考える。
(☆=選挙人の名前,○=候補者の名前)
☆1つに対し,3通りあるので10枚の投票用紙は最大3^10通り生じうる。(最小の場合は全員が
1人の候補者を選んだ場合で投票用紙は10種類)
∴59049通り・・・答


(2)
投票用紙は「○」しか記載されていない。○は○=A,B,Cのいずれか。
また投票用紙は10枚。
したがってA,B,Cの枚数がそれぞれx,y,z枚とすると
x+y+z=10 (x,y,z≧0)・・・ア
アを満たす(x,y,z)の個数を調べる。下の10個の●と、2つのしきり板「|」の並び方が、アの
方程式の(x,y,z)の個数に対応している。

  ●●●●|●●●|●●● →(x=4,y=3,z=3)
●●●●||●●●●●● → (x=4,y=0,z=6)
したがって,この並び方は12!/(10!*2!)=66

したがってアの(x,y,z)の個数は66個。

したがって投票の分かれ方は66通り・・・答

ねなきゃやばい・・
572132人目の素数さん:02/02/08 04:18
>>539

それって数学?ちょっと自分で扇形の面積の総和を評価してごらん。
三角関数の極限を使わないでできるの?
一流でない大学にはその程度の回答で満足する教師がごろごろいるけど。
>>477
πを円の直径に対する円周の長さと定義すると、
円の面積は
∫[0->r] 2πR dR=πr^2
なんてーのでどう?
574tr:02/02/08 04:59
>>573 さん
なるほど!コロンブスの卵ですね。
>>573
572によると駄目らしい。
πをどう定義するかを決定しておかないと話にならないのでは?
577tr:02/02/08 05:37
>>569 = 三国無双さん
ミスを発見したので, フォローします。

2. 6C3 だけ 3人ずつになるので別扱いで
部屋を区別する :
  6C1 + 6C2 + 6C3 + 6C4 + 6C5
  = (6C1 + 6C2)*2 + 6C3 = 62
部屋を区別しない :
  6C1 + 6C2 + (6C3)/2 = 31

3. も同様に修正して
0人を認めない : 31, 0人を認める : 32

# 不肖 tr も, 三国無双さんを応援しています ^-^)/
578132人目の素数さん:02/02/08 05:38
>>575

そうそう、だめだね。しわ寄せを極座標に持っていっただけでしょ?
曲線の長さや面積の概念を極座標から始めて一般論を構築するわけ?
勘弁してくれ。

まず、定義のすり替えは厳禁としよう。でないと話がおかしくなる。

>>572
一流大学とやらではどう回答するのかね?
580578:02/02/08 05:43
スマンが眠いのでしばらく失礼する。

わからなくて逃げたか藁
>>578が今井っていう人ですか?
583  :02/02/08 08:05
線形代数の固有ベクトルについての質問。
ある行列を対角化するとき、固有値を求めて、
その固有値の片方に関する行列のひとつの固有ベクトルを求める時、
僕は、t(1,-i,1)って答えたんですが、回答にはt(1,i,-1)って書いてありました。

僕は固有ベクトルを求める時しばしば回答と符号が違う場合があるんですが、
これらは同じ物なんでしょうか?
584583:02/02/08 08:07
ちなみに、上の質問は行列

0 i 1
-i 0 i
1 -i 0

の固有値-2に関する時です。
>>583
その行列をP,単位行列をEとしたとき
P+2Eとの積が0にならないので間違い。
586583:02/02/08 08:28
>>585
例えば、固有ベクトルをt(x1,x2,x3)とおいて、
行列*t(x1,x2,x3)=t(x1,x2,x3)
を計算したら、x1+x2+x3=0ってなったら、
t(-1,1,0)ってしたが、答えはt(1,-1,0)とかになって間違うんですが、
関係式からではあってますよね?
その、P+2Eとの積が0にならないので間違いってのは何でしょうか?
よくわかりません。
587132人目の素数さん:02/02/08 08:40
>>584の問題の、
固有値が1(重解)に関する行列の固有ベクトルを求める時も、
-x1+ix2+x3=0って関係になったんで、
a_1=t(i,1,0) a_2=t(1,0,1)ってしたんですが、
解答では、a_1=t(1,0,1) a_=t(1,-i,0)でした。
a_1とa_2の順番も何か関係あるのでしょうか?
588132人目の素数さん:02/02/08 10:02
0.9999999…=1
の証明ってどうやるんだっけ?
589132人目の素数さん:02/02/08 12:45
本を読んでいて不思議に思ったのですが、
”単準同型(=準同型写像が単射)f:R→R'が全射にもなっているとき、
fをRからR'(の上)への同型写像という”
”環R,R'がともにSを部分環として含むとき、RからR'への準同型写像が
Sの元をそれ自身に写す(つまり集合として等しいということでしょうか?)
ならば、この準同型写像をS(の上の)準同型写像という”
この二つってすごく紛らわしいと思うのですが・・・。
一番目の表記と全射準同型というのはどのように区別したらいいのですか?
”中への単準同型”という表記にも出くわしたのですが、これって単準同型と
何が違うのですか?
ひとつご助言をお願いします。
>>588
0≦1 - 0.9999999…
≦1 - 0.99...9   ←9がn個並んでいる
= 1 - (1-10^(-n))
= 10^(-n)
これはnが任意の自然数で成り立つ
よって1-0.9999999…=0
591132人目の素数さん:02/02/08 12:50
589に追記です。
”体F上の同型写像”という表記と特に混同しそうです。
まず>>539は近似の精度の評価をしてないので証明したことにならない。
例えば>>539的論理展開で球の表面積を"近似"すると間違った答えが出るよ。

>>573
∫[0->r] 2πR dRが円の面積である理由から示さないと
問題を解決したことにならない。つまり面積の定義をどうするかが問題。


あとちょっとそれるけど、三角関数の積分から円の面積を出すには、
三角関数の微分、つまり(sinθ)'=cosθの証明が必要。
ここで三角関数の定義を無限級数で与えていたなら項別微分可能性を
示してから項別微分すれば済むんだけど、そういう定義じゃない(少なくとも
高校まではそういう流儀じゃないよね)ならsinθ/θのθ→0での極限値が必要になる。
しかし、例えば高校の教科書などに載ってるsinθ/θのθ→0での極限値の証明には
円の面積を使っている。ここで本当は論理のループがあるんだよね。
593132人目の素数さん:02/02/08 13:18
x は 0 ≦ x ≦ 1 を満たす実数として,
P(x) = (1/2) * { P(x/2) + P(1 - x/2) }
という関係から
P(x) = 定数
を導くことってできます?
もちろん逆は明らかなんですが・・・。
>>593
P(x)が連続じゃなければ、例えば
x=m/(2^n) m、n:自然数 の形のとき、P(x)=1
x=それ以外の形のとき、P(x)=0
とかでも関係を満たすよ。
595132人目の素数さん:02/02/08 13:36
放置されたままの482ですが,自分でやってたのですがうまくいきません.
わかりにくく,説明不足なところもあったと思います.
こんなかんじになりました.

y1'=-1/a*g*(p*y2-s)
y2' =h*(e-y2)-g*(f-c*y2)(p*y2-s)/(a*(b+c*y1))

定数(a,b,c,e,f,g,h,s,p)

境界条件
t=0,x=3
t=0,y=28

です.そんなのは解けないだとか,スレ違いだとか言ってる意味がわからない
とか,なんでもいいので反応ください.こまってます.
ルンゲ・クッタ法などのプログラムで数値計算した結果ができれば
ほしいのですが,プログラム板で聞いたサイトのやつはわかりませんでした.
本を見てルンゲ・クッタ法をしっただけで,さっぱりです.

長々となってしまいましたが,とにかくこの連立常微分方程式は手計算で
できますか?それとも数値計算しかありませんか?
596593:02/02/08 13:38
>>594
レスありがとうございます。
「P(x) は連続」という条件を付け加えるとどうでしょうか?
597578:02/02/08 13:58
>>592

おお、やっと分かってくれる人が出てきた。そうそう、もともとそのループを
初等中等段階で解消する方策がないかと考えていることから始まった。

581や582のようにまともな高等教育を受けたことのない人間はほっておく。
ああ、彼らは高校生だったか、失礼した。



598588:02/02/08 14:23
>>590
ありがと!
>>597
負け犬は黙ってなさい藁
600 ◆FHB7Ku.g :02/02/08 15:44
>>577trさん
そーか!等量に分けるときはn!で割らないとだめなんだよね・・。
うっかりミス・・。ありがとうございます。
601教えてください:02/02/08 16:06
f(x)=(x+1)(x-2) g(x)=5x-1
F[a]を実数a小数第1位を四捨五入した値とするとき
F[f(x)]=g(x)をみたす実数xの値を教えてください。
602601↑ ちょと訂正です:02/02/08 16:08
f(x)=(x+1)(x-2) g(x)=5x-1
F[a]を実数aの小数第1位を四捨五入した値とするとき
F[f(x)]=g(x)をみたす実数xの値を教えてください。
603132人目の素数さん:02/02/08 16:15
なんかイイ謎々ありませんか?
604132人目の素数さん:02/02/08 16:38
すんまそん
n = 1 + 2^1 + 2^3 +…+ 2^(k-1)=2^k-1
になる証明を教えてください

605 ◆FHB7Ku.g :02/02/08 17:16
>>542
(1)
平面PQR上の点Xは実数s,tを用いて
PX↑=sPQ↑+tPR↑
を満たす。
OP↑=a/2,OQ↑=(2/3)b,OR↑=(2/3)c
であるから
OX↑=(1-s-t)(a/2)+s(2/3)b+t(2/3)c
となる。s=β,t=γとし,1-s-t=αとおくと
α+β+γ=1
よってOX↑=(α/2)a+(2β/3)b+(2γ/3)c ,α+β+γ=1・・・答 と表される。

(2)
交線上の1つの点をYとおくと,Yは平面STU上の点であるから
OY↑=sOS↑+tOT↑+uOU↑
s+t+u=1・・・ア
と表せる。
OS↑=(a+b)/2,OT↑=(b+c)/2,OU↑=c/3であるので
OY↑=(s/2)a+{(s+t)/2}b+(t/2+u/3)c
また,a=2OP↑,b=(3/2)OQ↑,c=(3/2)OR↑であるので
OY↑=sOP↑+{(3/4)(s+t)}OQ↑+{(3t+2u)/4}OR↑
Yは平面PQR上の点でもあるので
s+(3/4)(s+t)+(3t+2u)/4=1・・・イ
ア,イより(s,t,u)=(2-4k,5k,3-k) (kは実数)
∴OY↑=(1-2k)a+{(2+k)/2}b+{(13k+6)/6}c=(a+b+c)+k(-2a+b/2+13c/6)
よってLの方向ベクトルはk(-2a+b/2+13c/6) (k:実数)・・・答

(3)
Yが平面OAB上の点になるときは,(13k+6)/6=0⇔k=-6/13
Yが平面OBC上の点になるときは,1-2k=0⇔k=1/2
|(-6/13-1/2)(-2a+b/2+13c/6)|が求める線分の長さ。
これを2乗して√をとって、
線分の長さ=25/12・・・答
>>601,602
F[f(x)] は整数なので、g(x)も整数 すると x=(整数)/5
でなければなりません。
あとは g(x)-1/2 ≦ f(x)< g(x)+1/2 で範囲をしぼるもよし
f(x)=g(x) を解いてそれに近い(整数)/5 を代入してみるもよしです。

答えは x=31/5 , -1/5 でしょうか
607      :02/02/08 17:43
問題
四面体ABCDがある。空間内の点PがV[AP]+2V[BP]+3V[CP]+6V[DP]=V[0]をみたすという。このとき、点Pの位置を求めよ。

答えは線分APを2:1にない分する点をM,線分ZDを2:1にない分する点をNとすると、点Pは線分MNを3:1に内分する点である。
1V[AP]+2V[BP]+3[CP]+6[DP]=[0]が2V[AP]-2V[AB]-3V[AC]-6V[AD]-V[0]になるのはわかったのですが、答えがなぜそうなるのかがわかりません。
解法を詳しく教えて下さい。お願いします。
変数 s に関する以下の恒等式が成立することを示して下さい。

∫[0,1] dt t^(s/2-1)Σ_[n=1,∞]exp(-n^2 π t)
= 1/s(s-1) +
∫[1,∞] dt t^(-s/2-1/2)Σ_[n=1,∞]exp(-n^2 π t)

おながいします。
609607ー訂正ー :02/02/08 17:45
問題
四面体ABCDがある。空間内の点PがV[AP]+2V[BP]+3V[CP]+6V[DP]=V[0]をみたすという。このとき、点Pの位置を求めよ。

答えは線分APを2:1にない分する点をM,線分ZDを2:1にない分する点をNとすると、点Pは線分MNを3:1に内分する点である。
V[AP]+2V[BP]+3[CP]+6[DP]=[0]が12V[AP]-2V[AB]-3V[AC]-6V[AD]-V[0]になるのはわかったのですが、答えがなぜそうなるのかがわかりません。
解法を詳しく教えて下さい。お願いします
610601:02/02/08 17:45
>606
ありがとうございます
やはりわかりませんでした…
>>604
    2n =   2^1 + 2^2 + … + 2^(k-1)+ 2^k
−)  n = 1 + 2^1 + 2^2 + … + 2^(k-1)
 ̄ ̄ ̄ ̄ ̄ ̄ ̄ ̄ ̄ ̄ ̄ ̄ ̄ ̄ ̄ ̄ ̄ ̄ ̄
    n = -1 + 2^k
612595:02/02/08 19:29
すいません( ) がたりてなかったです.

y2' =(h*(e-y2)-g*(f-c*y2)(p*y2-s))/(a*(b+c*y1))

がただしいです.
本当にすいませんがだれかよろしくおねがいします.
613609ー訂正ー  :02/02/08 20:10
問題
四面体ABCDがある。空間内の点PがV[AP]+2V[BP]+3V[CP]+6V[DP]=V[0]をみたすという。このとき、点Pの位置を求めよ。

答えは線分APを2:1にない分する点をM,線分MDを2:1にない分する点をNとすると、点Pは線分MNを3:1に内分する点である。
V[AP]+2V[BP]+3[CP]+6[DP]=[0]が12V[AP]-2V[AB]-3V[AC]-6V[AD]-V[0]になるのはわかったのですが、答えがなぜそうなるのかがわかりません。
解法を詳しく教えて下さい。お願いします

614 ◆FHB7Ku.g :02/02/08 20:15
>>613マルチポストネタですか?
与式より12AP↑=2AB↑+3AC↑+6AD↑
したがって
12AP↑=2AB↑+3AC↑+6AD↑={(2AB↑+3AC↑)/5}*5+6AD↑
ここで線分BCを3:2に内分する点をLとおくとAL↑=(2AB↑+3AC↑)/5
よって
12AP↑=5AL↑+6AD↑={(5AL↑+6AD↑)/11}*11
ここで線分LDを6:5に内分する点をMとおくとAM↑=(5AL↑+6AD↑)/11
よって
12AP↑=11AM↑
∴AP↑=(11/12)AM↑
したがってPは線分AMを11:1に内分する点である。

したがって,
線分BCを3:2に内分する点をLとし,線分LDを6:5に内分する点をMとして,
点Pは線分AMを11:1に内分する点である。・・・・答
(答えの表現方法はベクトルの合成の方法によって何種類もある)
615XTORT:02/02/08 21:24
はじめましてXTORTといいます
某高校に通う高校一年生です
どうしてもわからない問題があり数学の検索をしていたところこのページがhitしたので投稿させてもらいました・・・
とにかくよろしくお願いします
<a href="http://members.tripod.co.jp/XTORT/math.html">画像+問題</a>
期限はとりあえず月曜(2/11)までです・・・
なにとぞよろしく!

P・S
ちなみにこの問題はうちの高校の数学オリンピック企画ででたものです・・・
>615
どう見てもbの方が短く見えるが…
>>572
>>578
>>597
罵倒するぐらいならあなたがただしく答えればいい。
>>617
>>572>>578>>597は泣いて逃げたのだから許してあげなさい
1+x+x^2+x^3+・・・+x^(n-1)=Sとするとき、
1+2x+3x^2+4x^3+・・・+nx^(n-1)=Tの値を、Sとxとnを使って表しなさい。
という問題があるんですが、どうしても解けません(++
どうか解き方を教えてください(^^;
620132人目の素数さん:02/02/08 23:26
>>619
T=1+2x+3x^2+…+(n-1)x^(n-2) +nx^(n-1) (1)
xT= x +2x^2+…+(n-2)x^(n-2)+(n-1)x^(n-1)+nx^n (2)

(1)-(2)より
(1-x)T=1+x+x^2+…x^(n-1)-nx^n
=S-nx^n
よって、x≠1のときT=(S-nx^n)/(1-x)

x=1のときT=1+2x+3x^2+4x^3+・・・+nx^(n-1)=n
621 ◆FHB7Ku.g :02/02/08 23:33
>>615
a≦bじゃなくa≧bと仮定して話を進める。
側面を構成する上底a、下底bの台形の高さは(1-b)/2
立体の上の頂点から底面に垂線を下ろし、その足をPとすると
Pと底面の正方形の一辺との距離は(a-b)/2
立体の高さをhとすると、
h^2 + {(a-b)/2}^2 = {(1-b)/2}^2
ここからhが求まる。
立体の体積は、底面を底とする四角錐Aの体積から上面を底とする四角錐Bの体積を引けばよい。
四角錐Aの高さをHとすると、
b : a = H-h : H
∴H = ah/(a-b)
よって求める立体の体積は
a^2*H/3-b^2*(H-h)/3
=(a^3-b^3)h/3(a-b)
=(a^2+ab+b^2)h/3
あとは既に求めたhを代入してね。
623620:02/02/08 23:34
む・・・

x=1のときT=n(n+1)/2

に訂正
624 ◆FHB7Ku.g :02/02/08 23:34

誤爆してしまた・・。
それに620さんが解いたみたいですね・・。
625622:02/02/08 23:34
間違えたー
側面を構成する上底a、下底bの台形
じゃなくて
側面を構成する上底b、下底aの台形
でした。
626132人目の素数さん:02/02/08 23:44
sinXcosXは一ではない?
>>626
マルチポストはやめよう。
-になることもある。x=120°とか。
628132人目の素数さん:02/02/08 23:49
「博士号」とかけて「足の裏の米粒」と解く。
その心は「とらないと気持ち悪いが、とっても食えない」。
629626:02/02/08 23:53
rejpy>
6d@g@ すいません<(_ _)>
630132人目の素数さん:02/02/08 23:59
>>628
ざぶとんn枚。
631132人目の素数さん:02/02/09 00:21
コピペに座布団か?
632132人目の素数さん:02/02/09 00:25
>>592, >>597
文字を使って>>539を書き直しただけだが。

r = 円の半径、
L = 円周の長さ、
π = L/(2r)、
L_n = 内接正2n角形の周長(n=2^m、m=1,2,3,...)、
h_n = 539のように考えたときに出来る内側の長方形の高さ、
とおくと
外側の長方形の高さ = r + (r - h_n)、
内側の長方形の底辺の長さ = L_n/2、
外側の長方形の底辺の長さ = L_n/2、
内側の長方形の面積 = h_n*L_n/2、
外側の長方形の面積 = {r + (r - h_n)}*L_n/2、
lim_[n→∞] h_n = r、
lim_[n→∞] L_n = L
だから
内側の長方形のの面積 → r*L/2 = πr^2、
外側の長方形のの面積 → r*L/2 = πr^2。

ゲンミツだとおもうぞ。ループもしてないとおもう。
592はズレてる。
633質問です:02/02/09 00:29
x は 0 ≦ x ≦ 1 を満たす実数,P(x) は連続関数として,
P(x) = (1/2) * { P(x/2) + P(1 - x/2) }
という関係から
P(x) = 定数
を導くことってできます?
もちろん逆は明らかなんですが・・・。
>>632
>内側の長方形の底辺の長さ = L_n/2、
>外側の長方形の底辺の長さ = L_n/2、

もし内側と外側からはさみうちしようとしているなら、
底辺の長さが等しいのはおかしい。(はさまれている図形の側面が
垂直で無いといけない)
どんな長方形を考えているの?

>lim_[n→∞] L_n = L

この左辺の極限を厳密に評価してください。
635632:02/02/09 00:42
>>634
>はさまれている図形の側面が垂直で無いといけない

垂直だけど。

>この左辺の極限を厳密に評価してください。

「収束する」だけじゃダメなの?
>>539
>半径rの円を、相当数のn等分割した扇形に切り、円弧が上下交互になるように並べ変える。

平行四辺形で近似するなら底辺は密着できるけど、長方形で密着させるのは無理。
両端に空きができるはず。8等分とか、等分が小さい場合の図を書いてみて。

>>635
増大列であることは明らかですね。
有界性は厳密に言えますか?
(有限の図形でも周長が有限とは限りません。)
収束先がL以下であることは明らかですが、Lそのものであることはどうやって?
637636:02/02/09 00:50
すみません、有界性は明らかでした。収束先の評価をお願いします。
638636:02/02/09 00:52
もう一つ書き間違い

>底辺は密着できるけど

じゃなくて「側面は密着できるけど」です。
639632:02/02/09 01:00
>>636
>長方形で密着させるのは無理。

右端をちょっと削って左端にくっつければいい。

>平行四辺形で近似するなら底辺は密着できるけど、

そう考えてもOK。

>収束先がL以下であることは明らかですが、Lそのものであることはどうやって?

L の定義より自明。
640質問です :02/02/09 01:15
解析学の教科書に、 次の微分方程式を解け(変数分離形およびその応用)
(5) y' = x + 2y - 1
(6) y' = exp (x + y) - 1
という問題が載っています。
変数分離形という指示なので、y' = f(x)*g(y) の形に変形しようとして
考えてみましたが、僕の力ではどうしてもできません。

上のふたつの式を変数分離形に変形する方法を教えて下さい。
おねがいします。
641636:02/02/09 01:24
>>639
>右端をちょっと削って左端にくっつければいい。

あ、その長方形ですか。

>>収束先がL以下であることは明らかですが、Lそのものであることはどうやって?
>
>L の定義より自明。

L_nの収束先をLと定義するということですか?
そうするとなると、L_nの有界性の仮定とループしてしまいませんか。
642132人目の素数さん:02/02/09 01:24
x+2y(x)=z(x)、x+y(x)=z(x)などとやってみる。
643636:02/02/09 01:41
L_nが有界であることの根拠は、内接n角形が円の内側にあるから(ですよね?)
つまりL_nの有界性は円周の長さが確定していることが必要ということです。
しかし、その円の長さの定義をL_nの収束先とするなら、
円の長さの定義にはL_nの収束先が確定することが必要になってしまい、
さらにはL_nの有界性が必要になってしまいます。
ここでループします。

もうひとつ。
Lの定義をL_nの収束先としなかった場合でも、
確かに小さな扇型一つ一つを見れば弧と内接n角形の一辺は
nを増やす毎に「近づき」ます。しかし、nを増やせば扇型の数も
増えますから、「近づき」具合がnの増え方に勝るかどうかの
定量的評価が必要ではないでしょうか?
644632:02/02/09 02:04
「滑らかな曲線の場合、内接折れ線の長さは上に有界である」
というのは(この話とは無関係に)一般的に成り立つことだから
それを認めてしまう。
それが嫌なら L_n を直接評価することになるが
L_n≦8r(=外接正方形の周長)
は L の定義に全く触れずに、初等幾何的にわりと簡単に示せる。

「もうひとつ」のほうは意味がよくわからないので>>632の文中で
ギャップがある…と思われるところを(気がむいたら)指摘して下さい。
645数学7:02/02/09 02:30
分からない問題です。 教えてください。



一辺の長さ2の正方形ABCDがあり、線分AB上(両端ふくむ)に点Pを取る。
三角形PCDの外接円を描き、
その円の直径のうちPを通るものとその円とのもう一つの交点をQとする。
このとき、Qの取り得る軌跡と、線分CDとで囲む部分の面積を求めよ。


積分使うのかな?
Qの軌跡を座標軸上の関数とかで表して。
本当に分かりません。
お願いします
うまい方法あるのかなぁ。
A(2,2) , B(0,2) , C(0,0) , D(2,0) とおき、
P(2t,2) とおく。(0≦t≦1)
△PCD の外心は
 CPの垂直2等分線:y=-tx+t^2+1 と
 CDの垂直2等分線:x=1
の交点だから(1,t^2+t+1)。これがPQ の中点だから
Q(2(1-t),2t(1-t)) となる。

(だからQの描く軌跡は放物線y=(1/2)x(x-2)だね。)
647 ◆FHB7Ku.g :02/02/09 04:19
>>646ほとんど同じ答案・・。
A(2,2),B(0,2),C(0,0),D(2,0),P(2t,2)(0≦t≦1)
とおく。△PCDの外接円の中心をRとするとRはPCの垂直2等文線
とCDの垂直2等分線の交点。
よってy=-tx+t^2+1とx=1の交点を求めるとR(1,t^2-t+1)
RはPQの中点なので,Q(2-2t,2t^2-2t)
x=2-2t,y=2t^2-2tとすると、xは0≦x≦2であり
2式からy=(1/2)(x^2-2x) となる。よってQの軌跡は
CDの中点をMとして,Mを通りCDに垂直な軸をもち,CとDを通る放物線のうち
線分CD側に対し凹なもの。・・・答
求める面積をSとすると
S=∫[0,2]{-(1/2)x(x-2)}dx=(1/2)(1/6)*(2-0)^3=2/3・・・答
http://www.angelfire.com/trek/emh/a.jpg

こいつをなんとかしてください
[1]と[2]だけでいいので おながいします
649132人目の素数さん:02/02/09 04:31
>>648
消えてる
650質問です:02/02/09 04:37
次のラプラス変換を求めよ。但し、y/uのラプラス変換とする。
dx/dt=ax(t)+bu(t)
y(t)=cx(t)
ここで、a,bは定数でxは状態値、uは入力、yは出力で x(0)=0 である。

答えは bc/(s-a) となるようです。
お願いします。
651642:02/02/09 05:06
>>640642で適当な事言ってしまったので責任持って回答。
y'=x+2y-1はx+2y(x)=z(x)で本当に解ける。簡単。
y'=exp(x+y)-1はx+y(x)=z(x)では変換が一回増えるのででz(x)=exp(x+y)とおく。
z'=z^2を出した後にw(x)=1/zとおけば解ける(勘がよければz'=z^2の時点で解ける?

応用とあるのはz(x)=exp(x+y)の所でしょう。
変換に2回かかってますがz(x)=1/{exp(x+y)}とおくと一回でいけます。
652132人目の素数さん:02/02/09 05:18
>>650問題をよく見なおす
653648:02/02/09 05:29
アドレスコピペでお願いします(;´Д`)
654 ◆FHB7Ku.g :02/02/09 05:39
>>640
「変数分離形?」(変数分離という意味もわからず・・)
(5)y'-2y=x-1
{ye^(-2x)}'=(x-1)e^(-2x)
ye^(-2x)=∫(x-1)e^(-2x)dx={(1-2x)e^(-2x)}/4+C
∴y=(1-2x)/4+Ce^(2x)・・・答

(6)これは自信ない・・・。
y'=e^(x+y)-1⇔y'+1=e^(x+y)・・・ア
ここで{e^(x+y)}'を計算してみると,
{e^(x+y)}'={(e^x)(e^y)}'=(e^x)*(e^y)*(y'+1)となる。したがって
y'+1={(e^x)(e^y)}'/{(e^x)*(e^y)}となりこれをアに代入して整理。
{e^(x+y)}'=e^{2(x+y)}・・・イ
となる。x+y=uとおくと
(e^u)'=e^(2u)
両辺をuで積分して
e^u=(1/2)e^(2u)+C
∴e^(x+y)=(1/2)e^(2x+2y)+C
e^y=Yとおくと
{e^(2x)}Y^2-2(e^x)Y+2C=0
Yについての二次方程式とみてY={1±√(1-2C)}e^(-x)=ke^(-x) (k=1±√(1-2C))
よってe^y=ke^(-x)となり
y=-x+Cとおくことができる。
このときy'=-1,e^(x+y)-1=e^C-1となり両者は等しくなければならないことからC=0
よってy=-x・・・答
655583:02/02/09 06:49
>>583-587
についてどなたか教えていただけませんか?
>>585にレスもらったんですが、これを答えのたびにチェックするってことでしょうか?
656132人目の素数さん:02/02/09 06:54
>よってy=-x
y'=e^(x+y)-1の解になってません
657 :02/02/09 06:56
この板の人ってやっぱり中学の数学とかカンペキにできるんですか?
僕は大学受験に向けて、基礎がないことから
最近「中学最高水準問題集」をやりはじめたが、難しくて参ってるよー
こういうのをパッと見て解けるようになるにはどうすればいいんでしょう?
658132人目の素数さん:02/02/09 06:58
>>583-584
>僕は、t(1,-i,1)って答えたんですが、

それって固有ベクトル?
659132人目の素数さん:02/02/09 07:03
>基礎がないことから最近「中学最高水準問題集」をやりはじめた
基礎がないなら難しい問題集はやらずに教科書を完璧に理解するべし。
どんどん先に進んで大学教養レベルの本を読むべし。
660 ◆FHB7Ku.g :02/02/09 07:23
>>656
そうだ・・。間違えた。というよりe^C=0となるCはない・・。そうなると
答はどうなるんだろう・・。
661583:02/02/09 07:25
>>658
ある行列を対角化するとき、固有値を求めて、
その固有値の片方に関する行列のひとつの固有ベクトルを求める時、

ひとつの固有ベクトル
6622ch初デビュー:02/02/09 07:27
わからない問題というわけじゃないんですが、高校生ならぎりぎりわかる
ようなのでもいいので、大学の定理・公式・証明・記号etcを
教えてください(マジ)
663132人目の素数さん:02/02/09 07:50
>(e^u)'=e^(2u)
>両辺をuで積分して
xで微分したんですから…
664132人目の素数さん:02/02/09 07:50
>>592
「微分積分学」(難波 誠 裳華房 ISBN 4-7853-1408-7)に
lim(θ→0)sinθ/θ=1 のトートロジーでない証明が出てるぞ.
書きこむ気力がないので見てくれ.
665 :02/02/09 08:05
>>662
Debutしてしまったか・・・
>662
教科書読め
667卵の名無しさん:02/02/09 08:50
統計のことで教えてください。
たとえばある集団で、胃癌が15人、大腸癌が2人、肝臓癌が1人、
食道癌が1人、だったとして、胃癌が有意に多いですよ、ってことを
検定するのには何検定を使うのですか。
素人で申し訳ありませんが。
669質問です:02/02/09 10:33
高校生です。化学の計算をしていて、小数第二位まで求める必要があり
面倒くさい計算だったので、
  (1 + x)^α ≒ 1 + αx (x ≒ 0)
の近似公式を使ったら切り上げした後の答えは0.01ほど違っていました。
…まじめに計算すれば良かったんですけどね。
この式の誤差っていうのはどれくらいなのでしょうか。
小数第何位までの正確さならxはどの範囲
αはどの範囲というふうなものはあるのでしょうか。
670 :02/02/09 11:17
>>669
3項目を考えてみ
671 :02/02/09 11:57
面積が整数の直角三角形の少なくとも1辺の長さは有理数である。
○か×か?
672132人目の素数さん:02/02/09 12:00
×
√6:√6:2√3
673583:02/02/09 12:33
>>658
え!?聞いといて答えてくんないんすか?
お願いしますよ!マジで
674 :02/02/09 12:41
>>673
t(1,-i,1)は固有ベクトルじゃないだろ。
全体が符号反転しているなら可。
質問があるんです。
順列・組み合わせで、10円硬貨2枚と50円硬貨3枚と
100円硬貨4枚をいろいろ組み合わせておつりをもらわずに
払える金額の式がわかりません。おしえてください!
>>675
地道に数えてもそんなにタイヘンじゃない。

式でアッサリ解きたいなら:
 硬貨を少し入れ替えて
  10円硬貨 2枚
  50円硬貨 1枚
  100円硬貨 5枚
 とすると、「各硬貨の使用枚数の組合せ」と「金額」が
 一対一に対応する。するといま
  10円硬貨の使用枚数は0〜2枚の3通り
  50円硬貨の使用枚数は0〜1枚の2通り
  100円硬貨の使用枚数は0〜5枚の6通り
 あるので、「金額」は全部で 
  3×2×6=36通り。
 (もし「“0円”は金額とは考えない」とする場合は35通り。)

677132人目の素数さん:02/02/09 13:05
金額のほうを聞いてるんぢゃないの?
sin10゜
ってどう求めるのですか?
3倍角の公式を使うのですか?
679132人目の素数さん:02/02/09 13:07
わかってるんなら使えば?
つかってもできません
681132人目の素数さん:02/02/09 13:19
なんでよ
そのあとでてくる三次方程式で行き詰まります。
三倍角→三次方程式→解の公式→汚い解
3次方程式に解の公式なんてあるんですか?
激しく感謝します
687132人目の素数さん:02/02/09 13:44
次のようにn個のデータがあるとき、フーリエ級数の係数を求めるプログラムを作りなさい。
i | 0,1,2,3,4....
xi| -3.08,-2.18....
という問題なのですが、調べても複素数データのものばかりで、理論がわかりません。
どのような式を使えばよろしいのでしょうか?
よろしければご教授ください。
688687:02/02/09 14:34
補足です。
データの数は0から11までの12個で、データは全て実数です。
よろしくお願いします。
689132人目の素数さん:02/02/09 14:36
>633
>x は 0 ≦ x ≦ 1 を満たす実数,P(x) は連続関数として,
>P(x) = (1/2) * { P(x/2) + P(1 - x/2) }
>という関係から
>P(x) = 定数
>を導くことってできます?
>もちろん逆は明らかなんですが・・・。

P(x)は定数でないと仮定。P(0)=P(1)である。
閉区間で連続な関数は最小値最大値をもつから、aで最小値P(a)<P(0)をとるとしておく。
(最小値がP(0)のときは最大値をとる点を選んで符号を逆にする。ようは等号成立しないように。)
P(a)=(1/2)*{P(a/2)+P(1−a/2)}…(1)
より、min{P(a/2),P(1−a/2)}≦P(a)…(2)
P(a)の最小性より(2)は等号成立。(1)よりP(a)=P(a/2)=P(1−a/2)
a/2についてこれを繰り返せばP(a)=P(a/2^n)
連続性から0に収束する数列anに対しP(an)→P(0)
an=a/2^nとすれば、an→0 
ところがP(an)=P(a)≠P(0)より矛盾
よって定数。でどうっすか。
>673
固有ベクトルの定義にもどって計算しろ
行列を作用させて本当にそのベクトルの定数倍になるか?
691583:02/02/09 15:26
>>583の訂正
線形代数の固有ベクトルについての質問。
ある行列を対角化するとき、固有値を求めて、
その固有値の片方に関する行列のひとつの固有ベクトルを求める時、
僕は、【t(-1,-i,1)】(←これ訂正)って答えたんですが、回答にはt(1,i,-1)って書いてありました。

僕は固有ベクトルを求める時しばしば回答と符号が違う場合があるんですが、
これらは同じ物なんでしょうか?

692583:02/02/09 15:27
固有ベクトルの向きが反対になってるだけなのでOKなのかな?
693132人目の素数さん:02/02/09 16:17
三角錐とか円錐の体積をもとめるとき、
底面積×高さ÷3としますが「÷3」の意味を
消防、厨房にどうやって説明したらいいでしょうか?
積分をつかわない説明方法を教えてください
円柱の入れ物に同じ底面積の円錐で
何杯入るかを自分でやってもらうしかないんじゃない?
695669:02/02/09 16:46
>>670
3項目ってなんなんでしょう…。
α, xの値を変化させて調べるってことですか?
とりあえずのんびりと |真の値 - 近似値| でも調べてみるとします。
ありがとうございました
696132人目の素数さん:02/02/09 16:53
>>694
実験しかないですかね・・・?
なにかへ理屈ないでしょうか。
>>693
検索したら
ttp://math.ed.niigata-u.ac.jp/~suzuki/yboard/yredata/ysrebun.html
こんなん出てきました。 参考になるかもしれません。
698132人目の素数さん:02/02/09 17:08
699132人目の素数さん:02/02/09 17:13
>>697-698

ありがとうございます!!こんな
鮮やかな説明があるなんて・・ヽ(´ー`)ノ
700質問です:02/02/09 17:39
AB=AC の二等辺三角形ABCで∠ABC=∠ACB=2∠BAC
BからACへ引いた∠ABCの二等分線を参考にsinCを求めてください


>>700 ∠ABCの二等分線とACの交点をDとすると
BCD はABCと相似な二等辺三角形、ABDもDB=DAなる二等辺三角形
となることを利用すると 辺の比がもとまります。

sin C = √(10+2√5)/4 となります。
702 ◆FHB7Ku.g :02/02/09 18:22
>>700
∠A=θとおくと∠B=∠C=2θ
よって5θ=πよりθ=π/5 sinC=2sinθcosθ

sin2θ=sin3θ⇔sinθ(4cos^2θ-2cosθ-1)=0
cosθ=(1+√5)/4よりsinθ={√(10-2√5)}/4
よってsinC=(1+√5)*{√(10-2√5)}/8・・・答
703132人目の素数さん:02/02/09 18:29

T=1
D=1
F=2
K=43
これはなんの公式でしょう。
ヒントTには 志村けん 東幹久。
Dには 北島三郎 安部なつみ
Fには 岡村隆史 久本雅美
Kには 香取慎吾 末広涼子
だそうです 
全然わからないので誰か教えてください
704 ◆FHB7Ku.g :02/02/09 18:33
>>700
というか図形を利用するんだた・・
△ABC∽△BCD
AB=1とすると求められると思う・・。
705132人目の素数さん:02/02/09 18:33
俺も歳だな.末広涼子なんてタレントがいたんだな.
706 ◆FHB7Ku.g :02/02/09 18:35
>>703
T=東京都(1つ)
D=北海道(1つ)
F=京都府、大阪府(2つ)
K=県(43個)
707703:02/02/09 18:38
>>706 
どうもありがとうございました
708132人目の素数さん:02/02/09 20:00
数理科学ならここでも話題ができますよ
どうか来て下さい
http://jbbs.shitaraba.com/study/18/
709132人目の素数さん:02/02/09 20:37
亜jp:亜w」エ:fらwk亜wk香tlv、b、g、sflflstらえ@z、
これなんだか分かりますか?クイズみたいなんだけど…
710 :02/02/09 20:44
>>709
適当に打っただけだろ
711132人目の素数さん:02/02/09 20:46
やっぱそうかなぁ…
712ひよこ名無しさん:02/02/09 21:07
放物線y=ax2乗 + bx + c は二点(−1、3)(3,3)を通り、その頂点
y=−3x−2上にあるときのa,b,cの値は?
誰か詳しく教えてくれ〜
713大学生:02/02/09 21:30
どなたかわかりませんかm(_ _)m?

ガンマ関数で、    Γ(1/2) = π^(1/2)

の証明なんですけれど・・。
>713
定義に入れて積分計算して終わり。
y=f(x)とおく

(−1、3)(3,3)を通るから
3=a-b+c・・・@
3=9a+3b+c・・・A

A−@より
0=8a+4b
∴b=-2a
これを@に代入すると
3=3a+c
∴c=3-3a

よって
f(x)=ax^2-2ax+3-3a
  =a(x-1)^2+3-4a

よって頂点は(1,3-4a)
これが y=-3x-2 上にあるから

3-4a=-3-2
∴a=2
b=-4,c=-3

なんかとんでもない間違いをしてそうだな
間違ってたらスマソ
>>713
積分変数を t として t^(1/2) = s と置換してガウス積分に帰着させる。
または ガンマ関数とベータ関数の関係式を使って B(1/2,1/2) を 三角関数を使って
置換し評価する方法 などがあります。
717132人目の素数さん:02/02/09 22:22
>>713
∫[0,∞]exp(-x^2)dx=√π/2
は知ってるかな。
ここで y=x^2 と置換積分すればいい


718大学生:02/02/09 22:53
>>716,717
ありがとうございました。
解けました!
719132人目の素数さん:02/02/09 23:17
>>549
「十分に長い長ぁ〜い階段がある。
サイコロを振って、出た目の数だけ登っていく。
このとき、超!!上の方の段に立ち止まる確率を求む。」

誰か解けよ。解けないのかよ?
つまり、これは例えば1億段目で止まる確率も、1億1段目で
止まる確率も同じになるだろうってことだろ?
つまりこの確率の極限値を求めろってことじゃねぇの?
ま、俺は解けねぇけどよ!!!(爆!!
720132人目の素数さん:02/02/09 23:24
関数g(x)をg(0)=1,x→∞に対してg(x)→0
となる、ゆっくりとした減少関数とする。このとき
∫[0,∞]g(x)exp(iωx/c)dx=ic/ω
となるようなのですが、どなたか説明できます?
721質問です:02/02/09 23:33
sin3x+sin(x+π/2)=√3sin(x+π/4)を満たすxの値を求める問題なのですが、
答えが-π/4 π/24 5/24πになるらしいのですが解き方がわかりません。
だれか教えてください
722XTORT:02/02/10 00:09
問題修正しましたのでよろしくお願いします・・・<m(__)m>
http://members.tripod.co.jp/XTORT/math.html
期限はとりあえず月曜(2/11)までです・・・
なにとぞよろしく!

P・S
ちなみにこの問題はうちの高校の数学オリンピック企画ででたものです・・・
>>720
g(x) = exp (-εx) として積分を計算して ε→0 の極限をとれば ic/ω
ってなるけど 「ゆっくりとした減少関数」 ってのが厳密に定義されてるのかな?
>>721
答えを方程式に代入しても成立しないので π/24 5/24π は解になってないようです。
-π/4 と 3π/4 のほかに4つほど解があります。
方程式か答えのどちらかが間違ってるんじゃないでしょうか
725721:02/02/10 00:27
問題はあっていますので恐らく答えが間違っていると思います。
どのように解けばよいのですか?
726724:02/02/10 00:46
>>721 右辺で和を積に直す公式を用いて
sin3x+sin(x+π/2)= 2 sin(2x +π/4) cos(x-π/4) = 2 sin(2x+π/4) sin (x+π/4)
と変形、 これが √3sin(x+π/4) に等しいので

sin(x+π/4) =0 or sin(2x+π/4) =√3/2
以下略 ってのはどうでしょう
727724:02/02/10 00:47
訂正
× 右辺で
○ 左辺で
728724:02/02/10 00:54

724の 「π/24 5/24π は解になってないようです。」は勘違いでした。
何度もごめんなさい。
729720:02/02/10 00:58
>>723
回答ありがとうございます
関数は厳密には定義されていないです
なんか、x(n+1)=x(n)+aとおき
与式=Σ_[n=0,∞]g(x(n))exp(iωx(n)/c)として
複素平面内で図形的に考えるようなことが
ちらっと書いてるんですけど、これで一度考えてみてください。
730721:02/02/10 01:04
>724
わかりやすい解答ありがとうございました
731質問です:02/02/10 01:19
1/2^n<x≦1/2^(n-1)のとき f(x)=1/2^(n-1)-x+1/2^n とする。
fは半開区間(0,1]={x∈R|0<x≦1}から
開区間(0,1={x∈R|0<x<1})への全単射になることを示せ。
>>722
>>622
に答えがあるぞ
733質問です:02/02/10 03:40
3次の線形定係数微分方程式
6(d3x/dt3)+6(d2x/dt2)+12(dx/dt)+12x=u
が与えられているとする。ここで入力をuとし、出力をy=xとする。
次の問いに答えよ。

@同伴形式の状態変数表現を求めよ。
Aステップ応答を求めよ。

留年がかかっていて、非常にやばいです。
工学の分野と思いますが、よろしくお願いします。
734132人目の素数さん:02/02/10 04:14
735tr:02/02/10 04:48
>>648 = 653 さん
[1] (1) (2x-1)^5/10 + C (2) ln|x^3 - 2| + C
  (3) sin^4(x)/4 + C (4) {-2x*cos(2x) + sin(2x)}/4 + C

[2] (1) -1/12 (2) 0 (3) (2√2 - 1)/3 (4) e^4(5e^2 - 3)/4

[3] (1) 1/6 (2) π(e^4 - 1)/2

[4] (1) ∂z/∂x = 3x^2 + 6x^2y^2, ∂z/∂y = 4x^3y - 8y^3
  (2) ∂z/∂x = 3cos(3x-2y), ∂z/∂y = -2cos(3x-2y)
[5]
[A] (1) ∂z/∂u = 2u, ∂z/∂v = 0
  (2) ∂z/∂u = e^x{sin(y) + cos(y)}, ∂z/∂v = e^x{-sin(y) + cos(y)}
[B] (1) 3x + 3y - z - 4 = 0 (2) x + y - z + 1 = 0
[C] (1) a(e - 1/e) (2) 2π√(a^2 + b^2)

# ミスはご容赦ください :P
736tr:02/02/10 04:53
>>731 さん
[全射] ∀y∈(0,1), ∃ n∈N ; 1/2^n≦y<1/2^(n-1)
なので x = 1/2^(n-1) - y + 1/2^n と選べば,
1/2^n<x≦1/2^(n-1) をみたし f(x) = y となる。
# i.e. ∀y∈(0,1), ∃x∈(0,1] ; f(x) = y

[単射] (0,1] の 2つの元
  1/2^m < x≦1/2^(m-1), 1/2^n < x'≦1/2^(n-1)
に対し, x > x' が成り立つとする。(ただし m, n∈N)
このとき 「m < n」 or 「m = n」 であるが
  i) m < n の場合
    f(x) = 1/2^m - x + 1/2^(m-1)
       ≧ 1/2^m
       ≧ 1/2^(n-1)
       > 1/2^(n-1) - x' + 1/2^n = f(x')
  ii) m = n の場合
    f(x) - f(x') = -x + x' ≠ 0
と, いずれにしても f(x)≠f(x') が成り立つ。
# i.e. x≠x' (x, x'∈(0,1]) ⇒ f(x)≠f(x')
737質問です:02/02/10 04:55
>>734さんありがとうございます。
深く感謝します。
738132人目の素数さん:02/02/10 08:29
>>723
g(x) = exp (-εx) といのは「急減少関数」じゃないの?
>>729
「ゆっくりとした減少関数」をちゃんと定義しておかないと話は進まないのでは?
739質問:02/02/10 10:49
3 以上の自然数 n に対し,
1 以上 n 以下の整数のうち n と互いに素であるものをすべて掛け合わせ,
その積を n で割った余りを Qn で表す.
例えば Q10 は, 1×3×7×9=189 を 10 で割った余りであるから,
Q10=9 である.

Qn=1 または Qn=n−1 が成り立つことを示せ.

中学生です。おねがいします
740質問です:02/02/10 17:27
フーリエ変換なんですが、

P(x) = 1 / ( a^2 + x^2 )   (aは定数)
  
の導出がわかりません、どなたかわかりませんか?

741質問です。:02/02/10 17:46
s,tを実数とし、座標平面状の4点
(s+1,t+1)、(s-1,t+1)、(s-1,t-1)、(s+1,t-1)
を頂点とする正方形の周および内部からなる領域をDとする。

t=2s のとき、連立方程式
(x+2)^2+(y-4)^2≦16、(x-4)^2+(y-4)^2≦16
が表す領域と、Dが共通部分をもつためのsの条件は?

よろしくお願いします。
742 ◆FHB7Ku.g :02/02/10 18:29
>>734
734のホームページ,保存してちょっと読んだけど3分でザセツ・・。
でもちょっと理解できるところもあった。。
743高校生:02/02/10 19:09
質問です。
「クロネッカデルタ」って公式を本で見つけたけど意味が分かりません(悲)
分かる人説明してもらえませんか?
744 ◆GaussrLU :02/02/10 19:27
>>739
中学生には難しすぎると思います.
全て証明できなくても, 例をいっぱい計算して
何か規則を見つけるだけでも十分でしょう.
考え方を書いてみますので, 参考にしてください.

まず, n≧3 なる整数に対して,
1 ≦ k ≦ n となる整数 k で, k と n が互いに素になる k
は偶数個であることを確認する.
# gcd(p,n)=1 のとき, gcd(n-p,n)=1 であるから,
# 互いに素になるものはペアで出てくる.
# n が偶数のとき, n/2 と n/2 で同じ物がペアじゃないか
# と思うかもしれないが, 互いに素でないので対象外.

n と互いに素な p に対して, p * q を n で割った余りが
1 になる q ( 1 ≦ q ≦ n ) が必ず存在する.
# これはユークリッド(Euclid)の互除法による.
# これが証明の肝になる.

Qn のうち, p ≠ q となるペアは pq ≡ 1 ( mod n ) だから,
省いてしまえる.
p = q となるものが曲者. ( 例えば, n-1 )
所が, p = q ならば, n-p も (n-p)^2≡1 (mod n) となる.
# 展開して計算すれば明らか.
そこで, p (n-p) を計算すると,
pn - p^2 ≡ -1 (mod n)
より, p = q となるものも, ペアにして -1 にできる.

というのが証明の流れと思います.
745数学特訓者:02/02/10 20:27
線形代数学の問題で固有値ベクトルの問題がわかりません。
どなたか教えていただけたらありがたいです!!
よろしくお願いします。

(1実対称行列の異なる固有値λ、μに対応する固有値ベクトルu、vは直交することを証明せよ。

|3 1 -1 |
(2 |1 3 1 | を対角化する直交行列Pを求めよ。
|-1 1 3 |
放物線Y=X二乗と(1、2)を通る直線Y=αX+B+Cにかこまれた部分の面積が最小になるには
αが何の時でしょう??
どなたか教えていただけませんでしょうか?
747731:02/02/10 21:10
>>736さんありがとうございました。
# i.e. ってどういう意味なんですか?
749132人目の素数さん:02/02/10 21:30
素因数分解を用いて10!, 20!!を求めよ、って問題で、
10! = 2^8*3^4*5^2*7 = 3628800,
というのはわかるんですが、回答みると20!!が
2^10*10!=3715891200,
となっているんです. このうち2のべき乗の部分は
わかるのですが,残りの6*10*12*14*18*20が何故10!
と等しいのかわかりません(等しい理由).
馬鹿にしてもいいので、わかりやすく教えてください
お願いします.
>>749
20=10*2
18=9*2
16=8*2
 ・・・
6=3*2
4=2*2
2=1*2

左辺の積=20!!
右辺の積=10!*2^10
20!!=2*4*6*8*….*20
=2*1*2*2*2*3*2*4*・・・*2*10
=2^10*10!
だと思う
2のべき乗部分を抜いたわけじゃないよ→2^10
>>719=549
まず問題をきちんと書くところから始めてください。
2n!! = 2*4*6*...*(2n-4)*(2n-2)*2n
なんだね。>>749-751を読んで始めて知った(恥
>>746 「(1、2)を通る直線Y=αX+B+C 」 のB+C とおく意味が分かりませんが、
とりあえず 書いてみます。
(1,2) をとおる傾きα の直線 y=α(x-1) +2 と y=x^2 を
連立させて得られる x^2 -α x +α-2 =0 … [i]
が直線と放物線の交点のx座標 (以下 a,b, a<bとします)を決めます。

かこまれた部分の面積は (b-a)^3/6 となるので b-a が最小になればいいのですが
これは[i] より b-a = √(α^2-4α+8) =√{(α-2)^2+4}
となるので 最小になるのは α=2 のとき
755731:02/02/10 21:51
>>748さん、ありがとです。
>>743
クロネッカ デルタは 通常δ_mn と表記され
m=nのとき 値 1 をとり、それ以外つまり m≠nのとき 0 となる
関数のことをいいます。
757132人目の素数さん:02/02/10 22:02
「物理が進化するには数学が進化しなければならない」
同様に
「数学が進化するには哲学が進化しなければならない」

ほんと?
758749:02/02/10 22:11
>>750,751
ありがとうございます。
頭固かったです。
759質問です:02/02/10 22:26
フーリエ変換なんですが、

P(x) = 1 / ( a^2 + x^2 )   (aは定数)
  
の導出がわかりません、どなたかわかりませんか?

留年がかかってマジでやばいですm(_ _)m
760132人目の素数さん :02/02/10 22:35
常微分方程式
du/dt+au=H(t-b) (t>0), u(0)=c , a,b>0 ,cは定数 H(t)はヘビサイド関数

の解u(t)をラプラス変換を使って求める問題です。
どなたかわかりますか?
>>759
P(x)が何なのか書いてくれないとどうしようもないです。
>>759
∫[-∞, ∞] exp(i p x) P(x) dx =1/|a| exp(-|a| p sign(p))
( ただし sign(p)= p/|p|)
ってなったよ。 役に立つかな
763 ◆FHB7Ku.g :02/02/10 22:42
>>741図形の考察が難しく,間違えてるかもしれないです。。

正方形の中心の座標は(s,2s)と表される。これは直線y=2x上にあることを示す。
したがって,Dとは一辺2の正方形の中心がy=2x上を動くときの正方形の通過領域に等しい。

(x+2)^2+(y-4)^2≦16、(x-4)^2+(y-4)^2≦16で表される領域をEとおく。
Eの先端は(1,4±√7)
よって
2s+1=4-√7⇔s=(3-√7)/2が最小値の候補。
s=(3-√7)/2のとき、正方形の横はs-1≦x≦s+1であり、(1-√7)/2≦1≦(5-√7)/2となり十分。

2s-1=4+√7⇔s=(5+√7)/2が最大値の候補。
s=(5+√7)/2のとき、正方形の横はs-1≦x≦s+1であり、(3+√7)/2≦1≦(7+√7)/2となり十分。

よって(3-√7)/2≦s≦(5+√7)/2・・・答
764 ◆FHB7Ku.g :02/02/10 22:47
>>763は違うこと判明。s=0でも成り立っているから・・。
となると、s+1=0のときも、s-1=2のときも調べないとだめか。・・

765 ◆FHB7Ku.g :02/02/10 22:56
763の途中までで,どれだけ部分点を得られるか,などという下卑た考えが
浮かんだりする・・。正方形の中心が直線状を動く通過領域と2円の
共通領域との,共通領域問題・・これは,僕の頭には存在しないパターンだった・・。
だれか解き方教えてください・・。
766761:02/02/10 22:57
P(x)をフーリエ変換しろってことだったのかyo。。
>>762
∫[-∞, ∞] exp(-i p x) P(x) dxでは?
767132人目の素数さん:02/02/10 23:25
>>765
正方形はy=2x+3とy=2x-3で囲まれた領域の内部をを平行移動しますよね。
図を描くとわかりやすいのですがy軸方向の上部では正方形の
端点が(s+1,2s-1),(s+1,2s+1)の辺が円 (x-4)^2+(y-4)^2=16
と接するときが上限。このときs=9/2。
また、点(s-1,2s-1)が円(x+2)^2+(y-4)^2=16と交わるときが上限。
このときs=-√(11/5)
よって、-√(11/5)≦s≦9/2
急いでやったからミスあるかも。
768761:02/02/10 23:29
∫[-∞, ∞] exp(-ipx) P(x) dx
= -sgn(p)*π/|a| exp(-|ap|)

じゃないのかなぁ。。
769761:02/02/10 23:38
スマソ、>>768は間違いだ…

>>759
P(x)のフーリエ変換F[P](p)は
F[P](p)
= ∫[-∞, ∞] exp(-ipx) P(x) dx
= π/|a| exp(-|ap|)
770XTORT:02/02/10 23:55
>>622
>>732にさんありがとうございます
できれば二問目のほうもお願いします

→もちろん誰でもOKです!
できれば二問目のほうだとうれしいです<m(__)m>
http://members.tripod.co.jp/XTORT/math.html
なにとぞよろしく!

771質問です:02/02/10 23:58
>>769さんへ

具体的な積分方法をおしえてくれませんか?

導出がわからないのです・・、お願いします。

772132人目の素数さん:02/02/11 00:05
>>771
極 ia での留数
773質問です:02/02/11 00:23
>>772さんへ

留数定理ですね!?

試してみます、ありがとうございます。
774質問:02/02/11 00:29
xy'+(1-2x)y=e^x の解のうち
x=1,y=0 の初期条件を満たす特殊解を求める

ってどうやればいいんでしょうか?
775741:02/02/11 00:50
>>763
真剣に答えてくださってありがとうございます。
4-√7>1なのでs=0は成り立たないのでは?
下限(って言うのかな・・・)は自分と同じ答えなのであっているんじゃないかと思います。
問題は上限なんですよ。
図形を書いて見た感じでは
(x+2)^2+(y-4)^2≦16 の周上の点と正方形の左側の辺のどこかが重なるところが
sの上限じゃないかと思うんですけどそれをどうやって求めればいいかわからない・・・。
それと、実はこれ、某大学の入試問題で、テストがマークシートなんです。
それでテスト用紙には
(シ−√ス)/2≦s≦(セ+√ソタ)/5
って書いてあるんです。
だから上限は間違ってると思います。また考えていただけませんか?

>>767
2つの円の共通領域です。それを踏まえたうえでもう一度力になってくれませんか?
776質問です:02/02/11 00:58
>>772さんへ
極 -ia のときは (-π / a) * e^(-ka)
極 ia のときは (π / a) * e^(ka) となり、
最終的な答えは  (π / a) * {e^(ka) - e^(-ka)} となりました。

ほんとにスミマセン。
もうすこし詳しい導出をお願いします。
横レス悪いんだけどさ
>>741の上限って
(s-1,2s+1)   ←正方形の左上の点
が円(x+2)^2+(y-4)^2=16
に接するときじゃだめなのかな
これを解くと
s=(3±√39)/5
だから上限は
s=(3+√39)/5
ちなみに下限は◆FHB7Ku.gさんと同じ
779132人目の素数さん:02/02/11 01:33
すみません 教えてください。
円の方程式を直感的にわかりやすくする式変形に
平方完成というのがあると思うのですが,
x^2+4x+y^2+4y+4=0 ⇒ (x+2)^2+(y+2)^2=2^2
これのうまいやり方を知っている方がいましたらぜひ教えてください。
お願いします。 (>_<)
>>779
1次の係数/2 を考える。定数は後で引く

x^2+4x=(x+2)^2-4
y^2+4y=(y+2)^2-4

よって
x^2+4x+y^2+4y+4=0

(x+2)^2-4+(y+2)^2-4+4=0
(x+2)^2+(y+2)^2=4=2^2

これでわかるかな?
781779:02/02/11 01:40
780さん、ありがとうございました〜!
感謝・感謝 (^。^)

782741:02/02/11 01:40
>>777
なんかあってそうな感じですね。
でも図形書いて見るとちがってるように思う・・・。
それでいいっていう理由はありませんかね?

ちなみに今気づいたが741の時のカキコで、間違いあった。
×連立方程式
〇連立不等式
スマソ
っていうか円の方程式の前に
二次関数をやったほうがいいよ
784783:02/02/11 01:50
は779へのレス
785783:02/02/11 02:05
>>782
ちがってるように思うって言われても・・・
実はあんまり自信がありません(w

誰か答えてください


sが増えると正方形はグラフの右上に行くから
それとDの位置をかんがえると
sが最大になるのは
この辺 →□
がDと接するとき。
で正方形の左上の点を代入したら実数解がでたから
これが上限かと
俺は考えました
786785:02/02/11 02:07
名前間違えた
777です

自信ないなぁ〜
787 ◆FHB7Ku.g :02/02/11 02:25
解き方わかりました。
正方形の通過領域は2x-3≦y≦2x+3・・・ア
これと2つの円の共通領域を書きます。
そうすると、最小となるのは2s-1=4-√7となるときで、
最大となるときは、y=2x+3と(x+2)^2+(y-4)^2=16の交点(√(11/5),2{√(11/5)}+3)
を線分y=2s-1が通過するときで
2s-1=2{√(11/5)}+3
s=(10+√55)/5
だから(3-√7)/2≦s≦=(10+√55)/5・・・答
788 ◆FHB7Ku.g :02/02/11 02:38
787も違うみたいですね・・マークシートの形みると・・。
うーん、混乱しています・。
789761:02/02/11 03:01
>>776
(-r,r)の線分と上半平面の半径rの半円を積分経路にとる。
半円部分の積分はr→∞のとき0に収束することを示せば、
求める積分値と特異点i|a|での留数*2πiが等しいことが分かる。
-i|a|は経路の中にないので計算には関係ありません。
790761:02/02/11 03:06
特異点i|a|
↓訂正
極i|a|
791質問なんですが・・:02/02/11 04:58
lim_[w→∞]∫[0,a] sin(wx)/x dx
lim_[w→∞]∫[-∞,∞] cos(wx)/(x^2+a^2)^2 dx
lim_[w→∞]∫[-∞,∞] sin(wx)/x(x^2+a^2)^2 dx
lim_[w→∞]∫[-∞,∞] exp(-x^4)cos(wx) dx

の求め方を教えて下さい。フーリエ解析
792132人目の素数さん:02/02/11 05:39
exp[iθ]=cos(θ)+isin(θ) の公式を使えばexp[iπ]=-1っていうのは
すぐわかるでしょ。でもこれをマクローリン展開した形から上手に
計算して求めるの。まーお暇でしたらちょっと考えて味噌。

この問題はハーディーがコーヒーを作っている数分の間に
ラマヌジャンが近似ですがやってしまった問題らしいです。
0+2+6+12+20+30+42........+9900までの和をだしてくだいさい

どなたか↑問題をといていただけませんでしょうか?
794 ◆FHB7Ku.g :02/02/11 09:10
0,2,6,12,20,30・・・
の一般項を(n)woまず初めに求める。階差数列をとると
2,4,6,8,10,・・・
よって階差数列はb(n)=2n
したがってn≧2のときa(n)=0+Σ[k=1,n-1]2k=n(n-1)
これはn=1のときも満たすのでa(n)=n(n-1)
9900=99*100より,9900は数列a(n)の第100項目。
したがって求める和をSとおくと
S=Σ[k=1,100](n^2+n)
=(1/6)*100*(100+1)*(200+1)+(1/2)*100*101=343400・・・答
795 ◆FHB7Ku.g :02/02/11 09:13
>>794 2行目を下のように訂正してください・・。
一般項をa(n)とし,まず初めに求める。
796 ◆FHB7Ku.g :02/02/11 09:16
あさから計算ミスした・・!n^2-nの和でした。
>>793
S=S=Σ[k=1,100](n^2-n)
=(1/6)*100*(100+1)*(200+1)-(1/2)*100*101=335800・・・答
>>794
>S=Σ[k=1,100](n^2+n)

誤 n^2+n
正 k^2-k
798132人目のそうっすさん:02/02/11 10:20
◆FHB7Ku.g って間違い多いな。ちゃんと見なおしてから投稿しれ。
>>798
俺的にはチャレンジ精神を買うガナー
もったいないお化け程間違いは多くないし許してやれ
801tr:02/02/11 10:44
>>741 さん
連立不等式
  (x+2)^2 + 8y-4)^2 ≦ 16, (x-4)^2 + (y-4)^2 ≦ 16
の表す領域を E と書く。 また
  P(s,2s), Q(s+1,2s+1), R(s-1,2s+1), S(s-1,2s-1),
  { C1 : (x+2)^2 + 8y-4)^2 = 16,
  { C2 : (x-4)^2 + (y-4)^2 = 16
  A(1,4+√7), B(1,4-√7) (←C1・C2 の交点)
と名付ける。

[下限] P を y=2x 上で, 左下から右上へと動かしたとき
領域 D の上辺 RQ が, 一番に領域 E と共通部分をもつ。
(∵ 点B は, Q・R の軌跡が囲む領域 y≧2x-1, y≦2x+3 に属する)
  ∴ 2s + 1 = 4 - √7 ⇒ s = (3 - √7)/2

[上限] 逆に P を, 右上から左下へと動かしたとき
領域 D の点S が, 一番に領域 E と共通点をもち,  \■ 領域 D
(∵ 点A は, S の軌跡の上方 y≧2x+1 に属する)    \
境界線 C1 に重なる。                  E の境界線 C1
  ∴ {(s-1) + 2}^2 + {(2s-1) - 4}^2 =16
  ⇔ 5s^2 - 18s + 10 = 0 ⇒ s = (9 + √31)/5
(∵ S の軌跡と C1 の 2交点のうち, x座標が大きい方)

以上まとめて (3-√7)/2≦s≦(9+√31)/5
>>741
>>765
>>801
記述形式の場合もう少し。
DとEがカド以外で1点だけを共有する(正方形の辺で接する)ことがあるかどうかを調べる。
直線x=kやy=kがEに接するとき、接点の候補は6つ。
Dは領域F:2x-3≦y≦2x+3の内部に収まるが、
6つの接点の候補はいずれもFの内部に無い。
よってカドの4点を動かして調べればよい。

>領域 D の上辺 RQ が, 一番に領域 E と共通部分をもつ。

近づき方次第で右の辺や左の辺が最初に接する場合もあるのでは。
(t=2s+18の場合など)
803802:02/02/11 11:28
>6つの接点の候補はいずれもFの内部に無い。

嘘。接点の候補(4,8)は力いっぱいF内でした。
804802:02/02/11 11:42
どうも噛みあわないと思ったら>>767の人と同じ過ちを…。
Eは2円の共通部分だったのか…鬱氏。
玉砕しないのtrタンだけだな
806732:02/02/11 12:57
>>770
>>622でaとbを用いた体積が求まる(これを式Vとする)
そしたらまず、aを一定にしbを変化させたときに体積がどうなるか
を考えるために、式Vをbで微分して、
aが一定の場合に体積の極大値を与えるb=f(a)を求める
で、そのb=f(a)を式Vにぶちこむと式Vはaだけで表される。
それをさらにaで微分し、
体積の極大値を求めればよい

最初aを一定としbを変化させるときにa>bの条件を
忘れないように。
807732:02/02/11 12:59
関係ないけど>>770の上のページのイラストは自筆?
>>807
取り込み画像っぽいね。
>>774
与式は (x e^(-2x) y)'=e^(-x) と変形できるので、これを積分して
一般解は y=( -e^x +C e^(2x)) /x となります。
初期条件より C=1/e あので 求める答えは
y= (e^(2x-1) -e^x)/x
810132人目の素数さん:02/02/11 14:22
教えて下さい。
等比数列の問題で、

初項が正の数である等比数列の第二項と第四項の和が60で、
第四項と第六項の和が540である。
この数列の初項から第八項までの和を求めよ。

という問題なのですが、
持っている参考書などを見ても同じような問題がなく
困っています。
よろしくお願いします。
811132人目の素数さん:02/02/11 14:27
関係式が2つ出るから初項と公比が求まりそうだなぁ。
求まらなかったとしてもその2つの式をいじって出来そうだなぁ。


と、思う。
>>810
初項がa、公比がrの等比数列の第n項をa(n)とすると
a(n)=a*r^(n-1)

使う公式はこれだけでも可
813132人目の素数さん:02/02/11 14:42
初項a公比rとすると、
ar+ar^3=60
ar^3+ar^5=540
ar、ar^3でそれぞれくくって割ると、
r=±3これを最初の式にいれたら初項が正ってことから
初項公比が一つに決まって…
あとは自分でお願いします。

814132人目の素数さん:02/02/11 14:50
>>811
>>812
>>813
ありがとうございました!
なんとか解く事ができました。
本当にありがとうございます。
815132人目の素数さん:02/02/11 15:20
一般的に曲線の定義って何ですか?1次元多様体?

816質問です。:02/02/11 16:12
基本的な質問で申し訳ないのですが、「座標平面上に原点O(オウ)を中心とする
半径2の固定円と、それの外側に接しながら回転する半径1の円C’がある。
円C’の中心が(3,0)にあるときのC'側の接点に印Pをつけて、円C'を円Cに
接しながら滑らずに回転させる。点Pの描く曲線で囲まれた領域の面積
を求めよ。」という問題で、
まずベクトルOPをθ関数で表してx関数をdyで積分すると言うやり方は
わかるのですが、(答え12π)この場合は
おうぎ型分割は出来ませんか?先ほどのθ関数で表された、
OPを長さに直し、1/2OP^2θとして扇形積分をすると、10πとなり、
あわないのですがどこに不備がありますか?ご指摘ください。お願いします。
 

817132人目の素数さん:02/02/11 16:14
500円、100円、50円、10円の効果で1000円を支払う
@)それぞれの硬貨を何枚使ってもよく、使わなくてもいいものもある場合の数
A)何枚使ってもいいが、すべての硬貨を使う場合の数
B)合計20枚以内とし、使わなくていい硬貨があってもいいとする場合の数

解答も聞きたいのですが、こういう問題はどう解くのでしょうか?
全部数えるのはちょと大変かと…
818132人目の素数さん:02/02/11 16:35
◆2002年東京大学前期入試◆
問1・Gスポットの座標を求めよ。
問2・右の乳首から左の乳首まで、A君が時速0.5kmで舐める時、何秒かかるか求めよ。
問3・A君が毎秒三回の速さで手マンをした時、Bさんが192秒後にイクことを証明せよ。なお、A君は中指を使ったものとする。
問4・A君がB子さんに手マンを一時間し続けた時の仕事率を求めよ。
問5・BさんがA君の上で上下運動をしている時の重力を求めよ。なお、円周率は3.14とする。

赤本にも無いし・・・
誰か解き方教えて下さい・・・
来年受験なんですよ。
819算法処女:02/02/11 16:41
「わからない問題」じゃなくて、「問題がわからない」んだけど、いいかな。
同志社大工学部Bの入試で、高校生には解けない問題が出たらしいんだけど、
2ちゃんねらーには解けるよね。その「問題」だれか教えて!
>>816 極座標で積分する場合、OPの長さを
OPとx軸のなす角 θ であらわし、∫1/2 OP^2 dθ として面積が求まります。
~~~~~~
10 π となるのは、Oと円C'の中心を結ぶ直線が x軸となす角を θ としているから
と考えられます。
821質問です。:02/02/11 16:50
そうですか!そうですよね。そうかなと思ったんですが何か確証がもてなくて・・
助かりました!わざわざ計算してくださりありがとうございます。
822厨房:02/02/11 18:13
誰か教えてください!!

∫1/sinx^2 dx

簡単だったらスマソ・・・
>>822
-cot x
824質問です:02/02/11 18:26
Cを連続の濃度を持つ集合とし、DをCの有限部分集合とする。
このときC-Dも連続の濃度を持つことを証明せよ。
825822:02/02/11 18:52
>>823

答えは-1/tanx となってるんですが・・・(高2なんでcotとかは分かんないです・・・)
証明教えてください!!m(_ _)m
826132人目の素数さん:02/02/11 19:13
500円、100円、50円、10円の効果で1000円を支払う
@)それぞれの硬貨を何枚使ってもよく、使わなくてもいいものもある場合の数
A)何枚使ってもいいが、すべての硬貨を使う場合の数
B)合計20枚以内とし、使わなくていい硬貨があってもいいとする場合の数

解答も聞きたいのですが、こういう問題はどう解くのでしょうか?
全部数えるのはちょと大変かと…
>>825
cot x = 1/tan x

cos x/sin x を微分せよ。微分のことはび(じ)ぶんでせよ!って
いうらしいぞ。
>>825
答えが-1/tanxになるってわかっているなら
それを微分してみればよいのでは?

逆をたどれば積分の証明になる
829ぺぺぺぺ:02/02/11 19:45
斉次多項式って何ですか?
教えて下さい。
最近,パソコンのやり過ぎで生活が乱れているとすごく怒られ,,
親からパソコン禁止命令が出て,パソコン,携帯,コミパルすべてを取り上げられて
しまいました。あしたから,2chどころか,メール交換も出来ない身分になってしまいました・・
テレビ見るだけの原始生活に強制突入です。
>798
すいませんでした・・。もう書きこまない(=書きこめない)身分になったので,安心してください。
いろいろ,楽しかった数学板,,いろいろとありがとうございました。
>>824
自然数全体の集合 N が C の部分集合であるとしてよい。
D と N の和集合を X とするとこれは N と同じ濃度。
D = { x_1, x_2, ...,x_n } として X = { x_1,x_2,...}
とする。f:C -> C-D を f(x_i) = x_(i+n), f(y)=y
ただし y は C-X の要素、とすれば f が全単射。
832132人目の素数さん:02/02/11 20:04
exp[iθ]=cos(θ)+isin(θ) の公式を使えばexp[iπ]=-1っていうのは
すぐわかるでしょ。でもこれをマクローリン展開した形から上手に
計算して求めるの。まーお暇でしたらちょっと考えて味噌。

この問題はハーディーがコーヒーを作っている数分の間に
ラマヌジャンが近似ですがやってしまった問題らしいです。

頼む!だえか考えてくれー!
833741:02/02/11 20:16
trさんありがとうございます。
上限は左下の点でよかったんですね。自分は計算ミスしてたみたいです。
また何かあったらよろしくお願いします。
834774:02/02/11 20:47
>809
わざわざありがとうございます。でも
(x e^(-2x) y)'=e^(-x) への変形の過程が
一番わからない所なんですけども(厨房
教えてもらえると有難いです…
835132人目の素数さん:02/02/11 20:53
>829
全ての項の次数が等しい多項式です。
836132人目の素数さん:02/02/11 21:15
ローマンの定理の証明を教えて下さい
837809:02/02/11 21:26
>>774 >>834 一般に
(d/dx + f(x))y = exp(-∫f(x)dx) d/dx ((exp(∫f(x)dx)y) … [i]
が成り立つので、 これを使いました。
詳しく書くと
xy'+(1-2x)y=e^x を x で わります。
y' + (1/x-2) y = e^x/x …[ii]
ここで、[i] において f(x) = 1/x -2 とすると exp(∫f(x)dx)=x e^(-2x) となるので [ii] は
e^(2x)/x (x e^(-2x) y)' = e^x /x と変形できるで ここから
(x e^(-2x) y)'=e^(-x) を得ます。

定数変化法と呼ばれる方法と本質的には変わらないので 検索してみてください。

>>830 ◆FHB7Ku.g さん
いつも回答楽しみにしてました。もう書き込まれないのは残念ですが
また net できるようになったら 復活してください。
838132人目の素数さん:02/02/11 22:05
スレ立てたら教育的?指導をされたので、こちらで質問させてください。

一辺の長さが1の正方形ABCDで、頂点Aから辺BCに向かって直線を
引き、辺BCとの交点Eを∠BAE=15°となるように取る。以下同様に、
頂点B、C、Dから辺CD、DA、ABに向かって直線を引き、各辺との
交点F、G、Hを∠CBF=∠DCG=∠ADH=15°となるように取る。

このとき、直線AE、BF、CG、BHの交点を、頂点Aに近いほうから、
時計回りにP、Q、R、Sとすると、正方形PQRSの面積はいくつか。

この問題は、三角関数(というか、三角比)を使えば簡単に、
1/2という答えが出せるのですが、小学生にわかるように説明する
方法がどうも思いつきません。どなたか初等的な解法をご教示
いただけますか?

これは、子供(小5)が塾で先生に出された問題として持ち帰った
もので、どこかの中学入試に出たそうです。
839高校生:02/02/11 22:14
>756
ありがとうございました
感謝感謝です!
840非通知さん:02/02/11 22:17
直線y=ax+bが2点P(1,-1)Q(2,1)の間を通るとき定数a,bの関係を求めよ。
お願いします。意味がわかりません。
>840
線分PQと交わる時(ただしPとQを通る時は除く)のa,bの条件を求めよ

って意味

PQは、直線y=2x-3の上にあるから、この直線との交点を求めて
交点が1<x<2にあれば、PとQの間を通っていることになる
842809:02/02/11 22:30
>>840
直線 y= ax+b によって xy平面は
y> ax+b を満たす領域 (直線の上側)と
y< ax+b を満たす領域 (直線の下側) に分けられます。

よって 直線が 2点 P Q の間を通るとは 2点のうち一方が 直線の上側、他方が下側にあることを
意味するので y-ax-b に PQの座標を代入した値の積が 負になってればいいわけです。

今の場合、 (1+a-b)(2-a-b) < 0 が求める関係となります
843非通知さん:02/02/11 22:30
>>841
できたら式と答えもおねがいっす。
844非通知さん:02/02/11 22:31
スマソ。
かぶった
>>840
f(x,y)=ax−y+b とするとき
f(1,-1)・f(2,1)<0
意味は自分で考えまそ
>>838
30度、60度、90度の三角形において
斜辺と直角を挟む辺のうち短い方の比は2:1である・・・ア

これをアリとすればできる。

真ん中の小さい正方形=元の正方形−(15度、75度、90度の直角三角形)×4・・・イ
15度、75度、90度の直角三角形×2=30度、75度、75度の二等辺三角形・・・ウ

等辺の長さが1の30度、75度、75度の二等辺三角形は、
等辺のうち一辺を底辺と見ると高さは1/2(∵ア)なので面積は1/4。・・・エ

イ、ウ、エより
真ん中の小さい正方形=1-(1/4)×2=1/2
アは要するにcos60°=1/2ってことです。
中学入試ではアリなはず。
>>847
アリもアリ、モハメド大アリっしょ
>>846
838です。ありがとうございます。言われてみれば単純な話ですね。
でも、この単純なことがなかなか見えない。頭が固くなったように
思います。
851132人目の素数さん:02/02/11 23:24
トーラスが単連結に縮むことを正式にはなんというのでしょう?
あとトーラスのplane partってどういう意味でしょう?
852774=834:02/02/11 23:34
>>837=809
親切丁寧にどうもありがとうございました。
線形微分方程式って奴ですね。
公式として覚えやすい形でしか理解していなかったので
こういう場面にくると弱かったです。
精進してきます
853824:02/02/11 23:59
>>831さん
ありがとうございました。
854132人目の素数さん:02/02/12 00:16
東京電気大学の問題(大学への数学VCより)なんですが、次の問題で
どうしても判らない部分が出たので博学な皆様のお力でお願いします。

---------------------------------------------------------------
次に挙げる2式の交点を出す計算にて・・・・

(x^2/a^2)+(y^2/b^2)=1 a>0 b>0 これを@とする

y^2=x これをAとする

@A式の交点を出すために連立させる
A式を@式に代入して二次方程式の形に変形すると
次の式になる。
(b*x)^2+(a^2)-(a*b)^2=0 これをBとする
@Aのグラフは楕円と放物線となりx軸に対称になるので
B式の解は重解になると仮定しその解をx=αと置くと
解と係数の関係より

2α=-(a^2/b^2) となりα<0となる
しかしA式よりx>0なので矛盾するグラフを見ても明らかに矛盾。
またB式の判別式DをとるとD>0となるので重解が否定される。これまた矛盾

---------------------------------------------------------------
この考え方にのおかしなところをご指摘いただければありがたいです。
どうか皆様よろしくお願いします。
855132人目の素数さん:02/02/12 00:30
周期T パルス幅τ パルス振幅A のフーリエ変換は?
                             ^^^^
856132人目の素数さん:02/02/12 00:50
そもそもグラフが接してもいないのに重解を持つというのが誤り。
Bを解の公式で解けば正の解と負の解が出るがx>0(y^2=x) より
解が1つに決まる。(条件より解が1つに絞られるだけ)
857132人目の素数さん:02/02/12 01:02
@式とA式が二点で共有点を持つグラフでその二点のx座標を出すつもりで
連立したのですが。この考え方は違うのでしょうか?
858132人目の素数さん:02/02/12 01:07
>>857
連立するのは正解。重解を持つのは間違いってこと。
例えば、
x^2+y^2=2・・・@(円)
y^2=x・・・・A(放物線)
の交点は代入して連立方程式を解くと
x=1,-2
と求まるけど、グラフからも、もちろん
@かAの式を見ても
x=-2は不適と分かる。
859132人目の素数さん:02/02/12 01:09
考え方は正しいのですが方程式のxの解が1つであることが
イコール重解を持つことにはならないということです。
860132人目の素数さん:02/02/12 01:23
確かに、貴殿の示された式を同じ方法で解くと-1となりおかしいですね。
よって私のB式も解の公式に放り込まないといけない訳ですね。ありがとう
ございました。これからは注意したいと思います。ところでなぜウソの解が
出るのですか?
ウソの解というよりは虚数解なのではないかな
862KARL ◆gjHKPQSQ :02/02/12 01:49
>>854
>>857
 同値関係に注意しながら考えていけば、分ると思います。
@かつA⇔AかつB ですが 実はここでx,yは実数という条件が存在してい
ます。Bの解をα,βとすると(αとβは重解の可能性あり)
B⇔x=αまたはx=βですから
AかつB
⇔ Aかつ(x=αまたはx=β)
⇔ y=x^2かつ(x=αまたはx=β)
⇔ (y=±√αかつx=α)または(y=±√βかつx=β)
最終的にxはひとつに決まるはずですが、それはx,yが実数であるという条件
つきです。つまり重解である場合も一つに決まりますが、α,βのどちらかが
負の数でその平方根であるyが虚数となって条件に合わない場合も考えられ
るわけです。実際には後者があてはまっているというわけです。つまり重解
である、と決め付けたのが間違いということになります。
 ところでBの式は
(b*x)^2+(a^2)x-(a*b)^2=0
~~~
の間違いですね。
863KARL ◆gjHKPQSQ :02/02/12 01:56
↑やっぱりアンダーラインずれた。ごめんなさい。
>>863
スレ違いネタ&既にご存知ならスマソですが、
・行頭の半角スペースはカット
・2個以上連続する半角スペースは1個にカット
されてしまうので、基本的には全角スペースで調整するとよいです。
865KARL ◆gjHKPQSQ :02/02/12 02:10
>>860
ほんとになぜ出てくるんでしょう。ポンスレという数学者はその意味を
追求した結果、射影幾何学を作ったとかいう話を聞いたことがあります
が...
866質問です:02/02/12 05:49
i^(i+1) :iは虚数単位

を x+iy (x,y∈R) で表すとどうなりますか?
867132人目の素数さん:02/02/12 07:15
i^(i+1) =exp(−π/2)iでしょ
868教えてください:02/02/12 13:51
3x^2+y^2+3z^2-2xy-2xz-2yz の実2次形式の標準形を求めてください。
869教えてください:02/02/12 13:52
3x^2+y^2+3z^2-2xy-2xz-2yz の実2次形式の標準形を求めてください。
870検定統計量について:02/02/12 14:10
検定統計量の分布というのは、どのようにして作成しているのでしょうか?
漸近分布は、標本の数を実際に多く取ってシミュレーションで求めているのでしょうか?
理論的に導く場合は、統計量の定義式を定義に従って積分し、それを1とするところから導くと言うことでよろしいでしょうか?
その場合、うまく積分できないようなケースはどうするのでしょうか?
871マルコフ過程について:02/02/12 14:22
stability と Ergodic の違いって何ですか?
872 :02/02/12 14:26
>>870
検定統計量って何を指してるの?
873質問です:02/02/12 14:28
∫t^2*(1-t^2)^2*exp(-t^2) dt
を∞からー∞まで積分です。
∫t^2*exp(-t^2)dt がわかればわかりそうなのですが
お願いします
874870:02/02/12 14:32
>>872
例えば、正規分布なら規準正規変数、t分布ならt統計量、F分布ならF統計量等々というように、検定を行うときに判定基準とする分布から生みだされる(と考える)統計量のことです。
875XTORT:02/02/12 14:37
>>622でaとbを用いた体積が求まる(これを式Vとする)
そしたらまず、aを一定にしbを変化させたときに体積がどうなるか
を考えるために、式Vをbで微分して、
aが一定の場合に体積の極大値を与えるb=f(a)を求める
で、そのb=f(a)を式Vにぶちこむと式Vはaだけで表される。
それをさらにaで微分し、
体積の極大値を求めればよい

最初aを一定としbを変化させるときにa>bの条件を
忘れないように。

>>実際どうすればいいんでしょうか意見ください・・・
>873
∫t^2*exp(-t^2)dt=∫t*(t* exp(-t^2))dt
として部分積分すれば
=(1/2)∫exp(-t^2)dt
877甲陽学院高:02/02/12 14:44
今年の甲陽の解答発表されてないんで教えてほしいんですけど、
 
放物線y=1/2x^2(X≧0)と直線y=−3/2x+2(x≧0)とX軸とで
囲まれる図形をFとする。
@Fに含まれる正方形のうち、1辺をx軸上にもち、面積が最大のものの
1辺の長さを求めよ。

A@で求めた正方形の左側にあって、Fに含まれる正方形のうち、面積が
最大のものの1辺の長さを求めよ。
 
Aがどうしても分かりません。教えてください
878 :02/02/12 14:54
集合 A={-1,0,1} は除法について閉じているか?
どうも閉じているらしい。
こういうときって0で割ることはあらかじめ考えないのか?
意見キボンヌ
>>873
∫t^2*(1-t^2)^2*exp(-t^2) dt
= -(1+d/da)^2 d/da∫exp(-a t^2) dt |_[a=1]
= -(1+d/da)^2 d/da (√(π/a)) |_[a=1]
= 7√(π)/8
88020:02/02/12 15:20
1辺が1の正六角形の面積をお願いします。
1辺が1の正三角形の面積の6倍では?
882132人目の素数さん:02/02/12 15:27
z=√1-x^2-y^2をxy平面で囲まれる図形の体積を求めてください。
お願いします。これが解けないと卒業できません。
883880:02/02/12 15:31
>>881
 ふう〜良かった!!
884873質問です:02/02/12 15:33
>>876
(1/2)∫exp(-t^2)dt=[-1/(2t)*exp(-t^2)]で
∞からー∞いれたら0ということでしょうか?
>>879
= -(1+d/da)^2 d/da∫exp(-a t^2) dt |_[a=1]
= -(1+d/da)^2 d/da (√(π/a)) |_[a=1]
= 7√(π)/8
この式はどういうことでしょか?
π←は パイですか?
885873質問です:02/02/12 16:31
>>876
∫exp( t^2) dt=√Π ガウス変換?でしたっけ思い出しました。
>>879
ガウス変換?を使って
= -(1+d/da)^2 d/da∫exp(-a t^2) dt |_[a=1]
= -(1+d/da)^2 d/da (√(π/a)) |_[a=1]
= 7√(π)/8
なるのはわかりましたが
∫t^2*(1-t^2)^2*exp(-t^2) dtをどうやって↓に変換しているのですか?
= -(1+d/da)^2 d/da∫exp(-a t^2) dt |_[a=1]

886数学特訓者:02/02/12 16:38
すいません
誰か745をお願いします。
887解けるもんなら解いてみろ(できるだけ詳しく):02/02/12 16:47
xy平面上に点A(5,2)と、tを媒介変数とする方程式
      x=2+t
      y=1+2t
によって表された直線Lが与えられている。
(1)点Aからの距離が最小となるようなL上の点のP座標と、距離の最小値
APを求めよ。
(2)点Aからの距離が2√5(2ルート5)となるようなL上の2点Q、R
   の座標を求めよ。また、線分AQとARのなす角θを求めよ。ただし、Qの
   x座標はPのx座標より小さいとし、0゜≦θ≦180゜とする。
(3)L上の2点S、Tを、三角形ASTが正三角形となるように定める。Sと
   Tの座標を求めよ。ただし、Sのx座標はTのx座標より小さいとする。
888132人目の素数さん:02/02/12 17:05
>>818が解けたら新しい数学の域だろう・・・?
しかし何で厨房はプライドが傷つけられるとセックスの話に持ってくのかね。
大抵の人間は両親がセックスした結果生まれているのだから、
その方面で先天的に才能がないって事はないはずなんだけどね。
>>878
1.(-5+√41)/2
2.(-3+√41)/2-√(-1+√41)
891132人目の素数さん:02/02/12 17:22
xy平面に関して同じ側にある2点A(2,1,3)、B(-1,-3,2)に対して、距離の和PA+PBが最小となるxy平面上の点をPとする。
PA+PBの最小値を求めよ。

ベクトルを使って説明していただくよう、お願いします。
>>887
(1)P(3,3)AP=√5
(2)Q(3-√3,3-2√3) R(3+√3,3+2√3) θ=120°
(3)S((9-√3)/3,(9-2√3)/3),T((9+√3)/3,(9+2√3)/3)
>>891
5√2
>>882
半径1の半球だろ?
それさえ分かれば残りは中学生レベル。
895873質問です:02/02/12 17:47
>>885
これに続きの問題があるんですが
f(t)=(1-t^2)^2*exp(-t^2) dt
とおいてフーリエ変換↓
φ(ω)=1/(√(2Π))∫f(t)*exp(-iωt) [∞、-∞]
してさらに
∫ω^2*(φ(ω))^2*dω  [∞、-∞]
という問題ですどなたか解けますか?
>>882
2π/3
897873質問です:02/02/12 17:49
>>895
間違えました
これに続きの問題があるんですが
f(t)=(1-t^2)^2*exp(-t^2)
とおいてフーリエ変換↓
φ(ω)=1/(√(2Π))∫f(t)*exp(-iωt)*dt[∞、-∞]

∫ω^2*(φ(ω))^2*dω  [∞、-∞]
という問題ですどなたか解けますか?
898132人目の素数さん:02/02/12 17:52
10進法が世界で愛用されてる理由を教えて下さい
899たぶん:02/02/12 18:30
指の本数
900
>897
やることは同じ
φの計算はexpの中身をtについて完全平方式にしてtを平行移動すればよし
その下のもωを定数倍すれば計算は同じ
902132人目の素数さん:02/02/12 19:35
すみません くだらない問題ですがおしえてください

 ○○○○○
− ○○○○
-------------
 33333

1から9の数字 一度だけしか使えないそうです
誰か教えてください
903873質問です:02/02/12 19:42
>>901
ほんとに何度も申し訳ないのですが
具体的に式にして書いてもらえないでしょうか?
お願いします。
>903
レポート?
905132人目の素数さん:02/02/12 19:53
W={x∈C^5|x1+x2=x3+x4=0}⊂C^5
でWの正規直交基底を求めるには、基をどのようにして求めればいいのか教えてもらえませんか?
お願いします。
906132人目の素数さん:02/02/12 19:58
>>902
どうでもいいが、まず検索サイトで調べれ。

41268 - 7935 = 33333
41286 - 7953 = 33333
907質問:02/02/12 20:02
0°≦θ<360°の時次の方程式、不等式の解き方を教えて下さい

@ 2(sinθ+cosθ)=ルート6
A sinθ+ルート3cosθ=−1
B sinθ−ルート3cosθ>−1
C cosθ≧ルート3sinθ

                    
>905
{x∈C^5|x1+x2=x3+x4=0}⊂{x∈C^5|x1+x2=0}⊂C^5

{x∈C^5|x1+x2=0}の正規直交基底を求める

C^5の自然な正規直交基底をとる。
(1,0,0,0,0),(0,1,0,0,0),(0,0,1,0,0),(0,0,0,1,0),(0,0,0,0,1)
このうちx1+x2=0の制約を受けるのは
最初の2つだけなのでこれを条件を満たすものに変えればよい

(1,-1,0,0,0),(0,0,1,0,0),(0,0,0,1,0),(0,0,0,0,1)

が互いに直交しているのは自明
あとは長さが1で無いものは1にするために定数倍すればよい
Wの正規直交基底も同様に求まる。
909ぺぺぺぺ:02/02/12 20:11
>>835
有難う御座います!
>>907
(1) 2(sin θ + cos θ)=√6 の解き方

ベクトル V=(2,2), W=(cos θ,sin θ) を定義すれば問題の式は内積使って
V・W=√6 ---(A)
と書ける。一方二つのベクトルの内積は二つのベクトルの成す角をφとすれば
V の長さ × W の長さ × cos φ = 2√2 cos φ ---(B)
(A) と (B) は等しいから
cos φ = (√3)/2
φ = ±30度
よって θ=45±30度

(2)(3)(4) も同様にできる。
これは三角関数の加法定理を使った解き方と本質的に
同じであることに気づくこと。
911132人目の素数さん:02/02/12 20:19
>908さん
ありがとうございます。
Wの場合は
(1,-1,0,0,0)(0,0,1,-1,0)(0,0,0,0,1)以外のもう一つはどうすればいでしょうか?
>897
∫f(t)*exp(-iωt)*dt
=∫(1-t^2)^2*exp(-t^2) *exp(-iωt)*dt
=∫(1-t^2)^2*exp(-t^2-iωt)*dt
=∫(1-t^2)^2*exp(-(t-(iω/2))^2 + (iω/2)^2)*dt
=exp((iω/2)^2) ∫(1-t^2)^2*exp(-(t-(iω/2))^2)*dt
 s=t-(iω/2)とおいて
=exp(-(ω/2)^2) ∫(1-(s+(iω/2))^2)^2*exp(-s^2)*ds

後は、
∫exp(-s^2)ds、∫s*exp(-s^2)ds、∫s^2*exp(-s^2)ds
の積分の計算は分かっているので終わり
>911
もう一つって?
914132人目の素数さん:02/02/12 20:44
Wの基は4つ必要ですよね?
条件を満たす基はどのようにすればいいですか?
915732:02/02/12 20:49
>>875
個人的にはこれが解けないなら数学オリンピックは
夢のまた夢だと思うが・・・

>>622さんの式から
V=(a^2+ab+b^2)√(1-2b-a^2+2ab)/6
で、この式をまずbで微分する(aは定数とみなす)
すると
dV/db=(1-a)(a+2b-ab-5b^2)/√(1-2b-a^2+2ab))・6
になる
分母は無視して分子=0になるところを考えると、
1-a>0であることから a+2b-ab-5b^2=0になるbを
0<b<aの範囲で見つければそのbが極大値を与えるbである
極大値を与えるbは
a≦1/2で  b=a
a>1/2で b=(1-2+√(a^2*16a*4))/(-10)
である.
このbを式Vにぶち込むと
a≦1/2において最大値を与えるaはa=1/2でV=1/48
当然こっちが最大値じゃなくて
a>1/2の範囲に最大値を与えるVがある。
あとは自力で計算してくれ、さすがにしんどい・・・。
>914
関係式1つにつき1次元下がる
Wは関係式がx1+x2=0とx3+x4=0の2つだから
3次元となり基底は3つ
917132人目の素数さん:02/02/12 21:00
くだらねぇ問題へ書いたら、こちらへ行けといわれたので…

xy平面上の2点 A(sinθ, 2-cosθ)、 B(-2-cosθ, -sinθ)に対し、線分ABの傾きをf(θ)とする。
ただし0≦θ≦2πとする。

f(θ)の最大値と最小値を求めよ。
で、問題集の解答が、
f(θ)=(sinθ-cosθ+2)/(sinθ+cosθ+2)-------------------(A)
cosθ=p、sinθ=qとおくと0≦θ≦2πであるから、
点(p, q)は、円 p^2+q^2=1の円周すべての点を取る。--------(B)
f(θ)=(q-p+2)/(q+p+2) = kとおくと、(k+1)p+(k-1)q+2k-2=0
この直線と円p^2+q^2=1の距離が半径1以下であるから
k^2-4k+1≦0 故に 2-√3≦k≦2+√3
よって最大値2+√3 最小値2-√3
となっているのですが、突然(B)の考えが出てくるのがわかりません。
p,qとおくのは分かりますが、だからといってその後でp^2+q^2=1で計算するのが理解できません。

私はこの問題は
(A)を三角関数の合成を使い、変形してから解いたのですが、答えは一致しませんでした。
なぜこのとき方では、いけないかも併せてお願いいたします。

当方リア工ですので、その程度で理解できるようにお願いいたします。
918132人目の素数さん:02/02/12 21:01
>914さん
本当にありがとうございました。
そういえばそうでしたね。
919132人目の素数さん:02/02/12 21:02
x>0のとき
f(x)=(3x+2)/(x^2+2x+2)
の値を求めよ

という問題がわかりません
よろしくおねがいします
>917
p^2+q^2=1 はsinθ^2+cosθ^2=1からきている

三角関数の合成をつかってといた・・・ってのはよくわからないな
具体的にはどうやったのだろうか
>>915
b=(2-a+√(4+16a+a^2))/10 ですよね
これを V に代入して a で微分して整理してやると結局
21a^3-29a^2+6a+3=0 …[i]
の解であることが 必要条件としてでてきます。
この方程式には 1/2<a<1を満たす解が2つあって,
そのうちの小さい方がVの最大を実現します。
数値は a=0.782305..., b= 0.53564...
くらいですが 解析的表現はきたないです。(複素数の3乗根とか入ってて)
mathematica にやらしてみたんですが
うまく工夫すれば[i]の解をもっと奇麗に表現できるかもしれません.
922132人目の素数さん:02/02/12 21:22
>917
>当方リア工ですので、その程度で理解できるようにお願いいたします。

大学生なら氏ねとしか言いようがないレベル
923名無し募集中。。。:02/02/12 21:23
>919
なんで問題をちゃんと書かないのだろうか
>919
問題を正確に書いてくれ
何を求めたいのか?
925132人目の素数さん:02/02/12 21:59
919みたいなのって厨房板からのネタ
工房までの問題の9割はネタ
ネタにも3種類あり
(1)問題をわざと正確に書かない(相手を混乱させイライラさせる)
(2)誤解した振りをして意味を聞いてみる(厨房を演じる)
(3)相手の学力を調べてみる
厨房板でdat落ちしたスレにそういうネタスレがあった。html化されたら読んでね
926132人目の素数さん:02/02/12 22:01
問題をわざと正確に書かないで相手に答えさせ、
その後で「違ってました」というのが流行っていたらしい
こういうのは完全放置してください
問題出す人(質問者)はトリップ必須に
してはどう?
928質問です!:02/02/12 22:08
f(x)=1/(1-2a*cosx+a^2)
このフーリエ級数展開のしかたを
おしえてください。
おねがいします。
>>877
(1)8/25 (2)(9-√65)/5 計算ミスしてそうだ

(1)
y=x^2/2,y=(3x/2)+2(x≧0)の交点は(1,1/2)
直線y=k(0<k<1/2)によってFを切断したとき
切り口の長さ(=正方形の一辺)がkになればよい
k=x^2/2よりx=√(2k),k=(3x/2)+2よりx=(4-2k)/3
切り口の長さがk ⇔ (4-2k)/3-√(2k)=k ⇔ k=8/25(答)

(2)
正方形の左側の辺のx座標は√(2k)=4/5
直線x+y=4/5とy=x^2/2(x≧0)の交点と(4/5,0)を結ぶ線分を
対角線に持つ正方形が最大
-x+4/5=x^2/2,x≧0 ⇔ x=(-5+√65)/5
一辺の長さ=4/5-(-5+√65)/5=(9-√65)/5(答)
>>927
あまり意味はないかと。
トリップ変えればいいわけだし・・・
ネタ?だったらスマソ。
931質問です!:02/02/12 22:17
線形代数のもんだいなんですけど、
    
    211
行列A=121 の固有方程式は  
    112

λ−2  −1   −1
 −1 λ−2   −1  =(λ−1)^2(λ−4)=0
 −1  −1  λ−2

となるらしいのですが全くわからないので教えて下さい。
932ハマコー:02/02/12 22:20
ハマコー
>931
いくらなんでもそれはネタだろ
934132人目の素数さん:02/02/12 22:22
xy平面に関して同じ側にある2点A(2,1,3)、B(-1,-3,2)に対して、距離の和PA+PBが最小となるxy平面上の点をPとする。
PA+PBの最小値を求めよ。

ベクトルを使って詳しく説明していただくよう、お願いします。

935931:02/02/12 22:22
  211
A=121 
  112   です。

すいません・・・
936名無し募集中。。。:02/02/12 22:24
>934
なんでベクトルを使わないといけないのか理由を言え
937132人目の素数さん:02/02/12 22:26
>933
ネタってどういう意味??
>>934
頻出パターンなので覚えれ
xy平面に関するBの対称点をB'(-1,-3,-2)とすると
|PA|+|PB|=|PA|+|PB'|≧|AB'|=5√2
939132人目の素数さん:02/02/12 22:48
直線y=−3/2x+2
となってるよ
マイナス見落としてる
>>890
と同じ数値になったが
940132人目の素数さん:02/02/12 22:57
x+y=1
xy=1
のとき
x/y^3+y^3/xの値を求めよ
お願いします

941929:02/02/12 22:57
>>939
スマソ。でもここに書くときにマイナス忘れただけなので…他の計算で間違ってるのか
942非通知さん:02/02/12 22:57
x+y≦1、2x+y≧1、y≧0のときP=x^2+y
Pの最大値最小値を求めよ。
お願いします。
943132人目の素数さん:02/02/12 22:59
940と942はネタ
氏ね
944132人目の素数さん:02/02/12 23:02
929は計算あってるんでは?
945非通知さん:02/02/12 23:05
>>943
マジなんだけど。
946132人目の素数さん:02/02/12 23:09
y=sinx^2

計算の過程を分かりやすく解説してください。
947132人目の素数さん:02/02/12 23:10
>946に追記
微分するときです
948質問です:02/02/12 23:13
こういう問題でもいいんでしょうか?

適切な述語を定義し、以下の述語論理式を表現せよ。
(1)学生には好きな科目があるが、その科目が良い成績であるとは限らない。
(2)自分が金持ちでないなら、誰とも結婚しない男がいる。
>>946
dy/dx=(ds/dx)inx^2+(di/dx)snx^2+(dn/dx)six^2+(d^/dx)sinx2+sin^2
950『質問です』:02/02/13 00:28
整数a,bがa^2+b^2=13 a^3+b^3=19 を満たすとき、a+bの値を求めよ。

高校2年の者ですが、かなり複素数を苦手としています。どなたかぜひ解答手順
を教えてください。お願いします。
>>949
激しくワラタ
^まで文字かよ。
>>950
a^2+b^2=13, a,bは整数なので
a=±2 b=±3 又は a=±3 b=±2
a^3+b^3=19 なので a=-2 b=3 又は a=3 b=-2
よってa+b=1

複素数は関係ないぜよ。
>>950
よくわからんが複素数で解くものなのか?
|a|≦|b|として13=a^2+b^2≧2a^2より|a|=0,1,2
|b|も整数になるのは|a|=2のときだけでこのとき|b|=3
以下略
954中3:02/02/13 00:43
回答、解説お願いします。

関数y=x^2のグラフ上で点Aは(-2,16/5)
点Bはx座標が3の点である。Bからx軸に引いた垂線とx軸との交点をcとする。

(1)2点A,Bを通る直線の式y=mx+nとするとき、m,nの値を求めなさい。
(2)線分BC上にBP:PC=5:4となるように点Pを取ります。このとき、△AOBと△APB
の面積比を出来るだけ簡単に求めなさい。
図  http://bekkan.omosiro.com/img-box/img20020213003332.jpg

お願いします!!!



http://bekkan.omosiro.com/img-box/img20020213003332.jpg

955中3:02/02/13 00:45
ん、2こも。垂線が斜めってます。すみません。
956873質問です:02/02/13 00:54
>>912
最後に∫s*exp(-s^2)ds これを教えてください
>>950
13がもっと大きいと大変なので
汎用性がありそうな解法を・・・

a+b=k
a^2+b^2=13
a^3+b^3=19

a,bが整数なのでkも整数
k^2=(a+b)^2=a^2+b^2+2ab=13+2ab
k^3-13k=2kab

13k=(a+b)(a^2+b^2)=a^3+b^3+ab(a+b)=19+kab
26k-38=2kab

∴k^3-39k+38=0 ⇔ (k-1)(k^2+k-38)=0
よってa,bの整数解が存在するときk=1が必要

# 13がもっと大きいと三次方程式の解を絞るのが大変になるのか :-P
>>954
y=Ax^2とかじゃねーの?
959●FHB7Ku.g:02/02/13 01:20
(1)m=29/25 n=138/25
(2)138:125
━━━━━━━━━━━━━━━━━━━━━━━━━━━━━━

             新しいスレッドが出来ましたので、
    新たに質問をする方はこちらで質問して頂けると嬉しいですわ

         ◆ わからない問題はここに書いてね 23 ◆
    http://cheese.2ch.net/test/read.cgi/math/1013530562/l50

━━━━━━━━━━━━━━━━━━━━━━━━━━━━━━
961●FHB7Ku.g:02/02/13 01:22
>>954
もしそうなら、ネタにまんまとひっかかったというわけか
許せんな
962132人目の素数さん:02/02/13 01:26
3変数x、y、zの間に、次の関係式が成り立っている。
       x+y+z=6
       xy+yz+zx=9
(1)V=xyzをxだけの式で表せ。
(2)Vが極大値をとるxの値を求めよ。また、そのとき
   のyとzの値を求めよ。
(3)(2)で求めたyとzの値のうちで大きい方をmと
   する。0≦x≦mにおいて、Vが最大値をとるとき
   のxの値を求めよ。
新たな質問はこっちでしてくれ。

◆ わからない問題はここに書いてね 23 ◆
http://cheese.2ch.net/test/read.cgi/math/1013530562/l50
>>959 >>961
偽者
埋め立てるよ。
973中3:02/02/13 01:32
>>958
そです!y=ax^2でした。
>>961
天然でございます。
>>962
(1) xy+yz+zx=9 より yz=9-x(y+z)=9-x(6-x)=x^2-6x+9
よって V=x^3-6x^2+9x=x(x-3)^2
(2) x=1, (y,z)=(1,4),(4,1)
(3) x=1,4
993●FHB7Ku.g:02/02/13 01:41
>>962
(1)V=x^3-6x^2+9x
(2)x=1 (y、z)=(1,4)(4,1)
(3)x=1,4
>>993
騙りですか?
10001000:02/02/13 01:43
10011001
このスレッドは1000を超えました。
もう書けないので、新しいスレッドを立ててくださいです。。。